Você está na página 1de 92

8 72 30

Vol. XIX No. 8 August 2017


Corporate Office:
Plot 99, Sector 44 Institutional Area,
Gurgaon -122 003 (HR), Tel : 0124-6601200
e-mail : info@mtg.in website : www.mtg.in 81 46
Regd. Office:
406, Taj Apartment, Near Safdarjung Hospital,
Ring Road, New Delhi - 110029.
Managing Editor : Mahabir Singh
Editor : Anil Ahlawat
50 77

CONTENTS 82

38

8 Success Story 12

55
12 High Yield Facts-Zoology
Neural Control and Coordination

30 NEET Foundation
Class XI

Subscribe online at www.mtg.in

38 High Yield Facts-Botany Individual Subscription Rates


Biomolecules-II
1 yr. 2 yrs. 3 yrs.
Mathematics Today 330 600 775
50 MPP-4 Chemistry Today 330 600 775
Physics For You 330 600 775
46 Concept Map Biology Today 330 600 775

55 NEET Essential Combined Subscription Rates

1 yr. 2 yrs. 3 yrs.


Class XII

Applications of Biotechnology
PCM 900 1500 1900
72 NCERT Xtract PCB 900 1500 1900
PCMB 1000 1800 2300
Principles of Inheritance and Variation
Send D.D/M.O in favour of MTG Learning Media (P) Ltd.
Payments should be made directly to : MTG Learning Media (P) Ltd,
77 MPP-4 Plot 99, Sector 44 Institutional Area, Gurgaon - 122 003, Haryana.
We have not appointed any subscription agent.
Owned, Printed and Published by MTG Learning Media Pvt. Ltd. 406, Taj Apartment, New Delhi - 29
Competition Edge

81 Biogram and printed by HT Media Ltd., B-2, Sector-63, Noida, UP-201307. Readers are adviced to make
appropriate thorough enquiries before acting upon any advertisements published in this magazine.
Focus/Infocus features are marketing incentives. MTG does not vouch or subscribe to the claims and

82 Biology Olympiad Problems representations made by advertisers. All disputes are subject to Delhi jurisdiction only.
Editor : Anil Ahlawat
Copyright© MTG Learning Media (P) Ltd.
85 Crossword All rights reserved. Reproduction in any form is prohibited.

MT BIOLOGY TODAY | AUGUST ‘17 7


Cracking the

Rank

EXAM
Nishita Purohit
• MTG : Why did you choose medical entrance? Nishita : All the papers I appeared in, were of different
Nishita Purohit : From childhood, I had a dream of serving formats and difficulty levels. So, it is very important to find
the mankind. So, I thought it was the best occupation out all the details regarding the test. On a personal note,
satisfying my wish. Therefore, I decided to appear for the difficulty level of AIIMS was at the peak followed by JIPMER
medical entrance examinations. and finally NEET.
• MTG : What exams you • MTG : How many hours in a
The ideal preparation plan is nothing as
have appeared for and day did you study to prepare
such. Each person has a different way
what is your rank in these for the examination?
exams?
of preparation. But the basic theme is
Nishita : I studied for around
Nishita : I also appeared for same, relevance of material, frequency 5 hours in my Allen classroom and
NEET and my rank was 11. of revisions and question practice. for about 4-5 hours of self-study.

• MTG : Any other achievements? (Please mention • MTG : On which topic and chapters you laid more
the name of exams and rank) stress in each subject?

Nishita : In the past two years, I had thoroughly focussed Nishita : For AIIMS, specially I laid more stress on Physics
on AIIMS but still I qualified for the second stage of Physics followed by Chemistry. Biology is easy and is more application
Olympiad after clearing the first stage. based. Some chapters such as ‘POC’, ‘Semiconductors’, ‘States
of matter’, ‘Surface chemistry’, ‘Biomolecules’, ‘Polymers’,
• MTG : How did you prepare for AIIMS and other
‘Chemistry in everyday life’ were my main topics.
medical exams?
Nishita : The main idea was to focus on all the topics • MTG : How much time does one require for serious
as taught in the class with utmost attention and follow preparation for this exam?
guidelines given in my coaching classes by Allen Career Nishita : Quantity of time is no factor. Qualitative and
Institute, Kota. smart study is of more importance.

• MTG : What basic difference you found in various • MTG : Any extra coaching?
papers you cleared? Nishita : No extra coaching.

8 MT BIOLOGY TODAY | AUGUST ‘17


• MTG : Which Subjects/Topics were you strong/ • MTG : What mistake you think you shouldn’t have
weak at? made?
Nishita : I was strong at Physics and weak in Chemistry. Nishita : I did not focus on NCERT textbooks earlier which
I regretted later and corrected my mistake. Also, I hesitated
• MTG : Which Books/Magazines/Tutorial/Coaching
a lot in asking questions from my teachers. Students should
classes did you follow? avoid this.
Nishita : I primarily followed Allen classroom material • MTG : How did you de-stress yourself during the
and solved few books like HC Verma, MTG-magazines and preparation? Share your hobbies and how often
NCERT. could you pursue them?
• MTG : In your words what are the components of Nishita : I used to perform pranayam and meditation to
an ideal preparation plan? de-stress myself. I enjoyed listening to songs whenever I got
bored.
Nishita : The ideal preparation plan is nothing as such.
Each person has a different way of preparation. But the basic • MTG : How have various MTG products like
Explorer, Books and Magazines helped you in
theme is same, relevance of material, frequency of revisions
your preparation?
and question practice.
Nishita : I did follow a few magazines but not regularly.
• MTG : What role did the following play in your However, whatever I read. I found it informative.
success:
• MTG : Was this your first attempt?
(a) parents
Nishita : Yes, this was my first attempt.
(b) teachers
• MTG : Had you not been selected then what would
(c) school?
have been your future plan?
Nishita : I firstly thank my coaching institute for nourishing
Nishita : If I hadn’t been selected, I would have switched
me. I thank my parents, my brother, relatives and teachers for on to administration and civil service side.
their continuous support.
• MTG : What do you think is the secret of your
• MTG : Your family background? success?
Nishita : My family background is towards engineering Nishita : The secret was nothing but regular study and
side. My dad and my brother are engineers and my mother practice questions along with NCERT books.
is a pharmacist. All the Best!

10 MT BIOLOGY TODAY | AUGUST ‘17


MT BIOLOGY TODAY | AUGUST ‘17 11
Class XI
Neural Control and Coordination
• Nervous system (Neural system) is a system of highly specialised nerve cells, nerve
fibres and organs (called receptors), that coordinate and control the activities of
different parts of the body. This is done by converting the internal and external
stimuli of environmental changes into the form of electrical impulses (a process
called transduction) and transmit them to other specialised cells called effectors
(muscles and glands) to react accordingly. Between the receptors and effectors are
present the conducting cells of the nervous system, the neurons. 2017



3


• Overall function of nervous system is to collect information about the external
conditions in relation to the body’s internal state, to analyse this information and
to initiate appropriate responses to satisfy certain needs (maintain homeostasis).
The most important need is survival. The nerves do not form one single system,
2016


4


2

rather they form several systems, which are interrelated. Some of these are physically
Analysis of various PMTs from 2013-2017

separate, others are different in function only.


NEURAL SYSTEM
2015

• The neural system provides an organised network of point-to-point connections


1
1


4

for a quick coordination between various body functions. A brief classification of


human neural system is given below.

Human Neural System


2014

1
2

Central neural system Peripheral neural system


2013


3

Brain
Somatic neural system Autonomic neural system
AIPMT/NEET

Spinal cord Parasympathetic neural system


Kerala
AIIMS

K-CET

J&K

Sympathetic neural system


AMU

12 MT BIOLOGY TODAY | AUGUST ‘17 12


Neurons
• Neurons are the structural and functional unit of neural system. A neuron structurally comprises of three major parts : cell
body, dendrites and axon.
Neuroplasm Dendrites
Abundant cytoplasm of cell Short fibres which branch repeatedly and project
body with relatively large out of the cell body are called dendrites. These
typical cell organelles and certain granular
The cell body contains cytoplasm with

central nucleus. fibres transmit impulses towards the cell body,


Nucleus and are called afferent process.
Nissil’s granuala
Nissl’s granules
Irregular masses of rough ER
bodies called Nissl’s granules.

Neurofibrils
Cyton or Soma

with numerous ribosomes Characteristic to all neurons, play role in trans-


and polysomes, synthesise mission of impulses.
proteins for cell. Axon hillock
Neurilemma
Axon
Single, very long process of Schwann cell nucleus
uniform thickness, conducts Medullary sheath
nerve impulses away from cell Axis cylinder with
body, called as efferent process. neurofibrils
Collateral fibre
Node of Ranvier Side branches arising from axon, much finer
The gaps between two than the main axonal process.
adjacent myelinated sheaths
are called nodes of Ranvier. Terminal arborisation
Group of branches into which axon ends.

Synaptic knob
Contain mitochondria and secretory vesicles.

Fig.: Neuron

Types of Neurons

On the basis of polarity On the basis of structure

Afferent neurons Unipolar


• Carry impulses from receptors to CNS. • When neuron has only one process, i.e., one axon
• They are sensory in nature. extends from the cell body, e.g., in embryonic stages.
• The terminals of dendrites become modified to form
receptors.
Pseudounipolar
• When one process arises from the cell body and then
Efferent neurons divides into two i.e., axon and dendrite, e.g., dorsal
• Carry impulses from CNS to effectors like muscles root ganglion of spinal nerves.
or glands.
• They are motor in nature.
• The axon terminals come in contact with the motor Bipolar
end plate to form neuromuscular junction. • When one axon and one dendrite come out from the
soma, e.g., retina of eye.

Non-polar
• When all processes of soma are equal and nerve Multipolar
impulse can be conducted in any direction, e.g., • When the axon and several dendrites extend from the
neurons of coelenterates. soma, e.g., motor neurons and interneurons.

MT BIOLOGY TODAY | AUGUST ‘17 13


NERVE IMPULSE
• Nerve impulse is a wave of bioelectric/electrochemical disturbance that passes along a neuron during conduction of an
excitation. The nerve impulse travels along a neuron or across synapse between one neuron and another, or between neuron
and an effector.
• Nature of nerve impulse or conduction of nerve impulse is an electrochemical process. It has been found that impulse
conduction depends upon permeability of axon membrane (axolemma) and osmotic equilibrium and electrical equivalence
between the axoplasm and extracellular fluid (ECF) present outside the axon.
Generation and Conduction of Nerve Impulse
• When a neuron is not conducting any impulse, i.e., resting, the axonal membrane is comparatively more permeable to potassium
ions (K+) and nearly impermeable to sodium ions (Na+). Similarly, the membrane is impermeable to negatively charged proteins
present in the axoplasm. Consequently, the axoplasm inside the axon contains high concentration of K+ and negatively charged
proteins and low concentration of Na+. In contrast, the fluid outside the axon contains a low concentration of K+, a high
concentration of Na+ and thus forms a concentration gradient. The ionic gradients are maintained by active transport of ions by
sodium-potassium pump which transports 3 Na+ outwards for 2 K+ into the cell. As a result, the outer surface of the axonal
membrane possesses a positive charge while its inner surface becomes negatively charged and therefore is said to be polarised.
The electrical potential difference across the resting plasma membrane is called as the resting potential.
• When a stimulus is applied, the permeability of the membrane to Na+ ions is greatly increased at the point of stimulation.
It is due to the fact that the Na+ channels open and the K+ channels remain closed.
• As a result, sodium channels permit the influx of Na+ ions by diffusion. This results in the positive charge inside and negative
charge outside. The change in polarity across the plasma membrane is known as action potential and the membrane is said
to be depolarised.
• With the increase of Na+ ions inside the nerve fibre, the membrane becomes less permeable to Na+ ions and more permeable
to K+ ions. Na+ influx stops and K+ outflow begins until the original resting state of ionic concentration is achieved. Thus,
resting potential is restored which is called repolarisation of the membrane.
• When an impulse travels along a myelinated neuron, depolarisation occurs only at nodes. The action potential jumps from
node to node and impulse transmission along the myelinated axon is faster than the series of smaller local currents in a
non-myelinated axon. This type of conduction is called saltatory conduction.

Na+ channel
Depolarisation opens
Voltage gated Na+ channel is K+ channel Na+ channel
in resting state and voltage
gated K+ channel is closed.
Na+ flows inward

(a) (b)

Resting potential Depolarisation begins

Repolarisation
K+ ions exit and Na+ begins, K+
channel begins to open channel opens
again. and Na+
(d) (c) channel gate
closese.

Repolarisation completes action potential peaks


Fig.: Stages in axon membrane potential during resting, depolarisation, action potential and repolarisation

14 MT BIOLOGY TODAY | AUGUST ‘17


Transmission Through Synapse
• Synapse is the junction between two neurons, across which the impulse passes from one neuron to the next.
• A synapse is formed by the membranes of a pre-synaptic neuron and a post-synaptic neuron, which may or may not be
separated by a gap called synaptic cleft.

Types of Synapse

Electrical synapse Chemical synapse


• At electrical synapse there is continuity between the pre-synaptic • At a chemical synapse, when an impulse arrives at a pre-synaptic
and post-synaptic neurons. The continuity is provided by the gap knob, calcium ions from the synaptic cleft enter the cytoplasm of
junctions between the two neurons. In electrical synapse there the pre-synaptic knob. The calcium ions cause the movement of
is minimal synaptic delay because of the direct flow of electrical
synaptic vesicles to the surface of the knob. The synaptic vesicles
current from one neuron to the other through gap junctions. Thus,
impulse transmission across an electrical synapse is always faster are fused with the pre-synaptic membrane and get ruptured to
than that across a chemical synapse. Impulse transfer through discharge their content (neurotransmitters) into the synaptic cleft.
electrical synapse do not involve any neurotransmitter. Electrical • The neurotransmitter of the synaptic cleft binds with protein
synapses are rare, it is found in the cardiac muscle fibres, smooth receptor molecules on the post-synaptic membrane. This binding
muscle fibres of intestine and the epithelial cells of lens. a c t i o n c h a n g e s
Presynaptic t h e m e m b r a n e
neuron Microtubule potential of the post
Cytoplasm synaptic membrane, Presynaptic Synaptic vesicle
opening channels in neuron
Mitochondrion
the membrane and
allowing sodium
ions to enter the
cell. This causes Postsynaptic
Gap Postsynaptic neuron
neuron the depolarisation Synaptic
junction Ions flow through gap
Neurotransmitter released
Presynaptic junction channels and generation of vesicle fusing Presynaptic membrane
membrane action potential in
the post-synaptic
Synaptic
cleft

membrane. Thus,
t h e i m p u l s e i s Postsynaptic
neurotransmitter Ions flow through Postsynaptic
transferred to the receptor postsynaptic channels membrane
Postsynaptic Gap junction channels
membrane next neuron.

TYPES OF NERVOUS SYSTEMS


Central Nervous System (CNS)
The central nervous system (CNS) is the processing centre of the nervous
system. It receives and sends information to peripheral nervous system.
The brain and spinal cord are continuous structures, which together form
the CNS. They are both protected by hard bony coverings-the brain
by the cranium and the spinal cord by the vertebral column and by
a continuous, tough, three layered membrane called the meninges.
Both the brain and spinal cord are bathed in cerebrospinal fluid (CSF),
which is continuously circulated through the cerebral ventricles, the
cavity of the spinal cord and the sub-arachnoid space. It serves as a pad
to cushion the CNS from shocks and mechanical injuries. The CSF also
carries oxygen and nutrients from the blood to the neurons and neuroglia
of the CNS.
MT BIOLOGY TODAY | AUGUST ‘17 15
Cranium
Cerebrum

Fore brain

Brain
Thalamus Cerebrospinal
Hypothalamus fluid (CSF)
Midbrain Ventricles
Pons
Hind brain

Cerebellum

Medulla Meninges

Spinal cord
Spinal cord cavity

Vertebral column

Fig.: Overview of central nervous system

Human Brain
• The human brain is the command centre for the human neural system. It receives input from the sensory organs and sends
output to the effectors. It processes and interprets sensory information sent from the spinal cord.
• The brain is covered by three protective layers of connective tissue called meninges.
Veins Duramater
Outermost, tough, fibrous membrane adhering
Arachnoidmater closely to the inside of the skull
Thin “spider webby” structure
Subdural space
Sub-arachnoid space Space between arachnoid membrane and
Space between arachnoid duramater
membrane and piamater and is Arachnoid granulation villi
filled with CSF
Longitudinal fissure
Piamater Separates two cerebral hemispheres
Innermost, thin, very delicate
and vascular membrane Cerebral cortex
Cerebral vein

Fig.: Meninges of brain

Fore brain
(Prosencephalon)

Cerebrum Diencephalon
Olfactory lobes
• Largest and most complex part of human brain,
Epithalamus (Roof)
• Paired club shaped consists of left and right hemispheres, connected by
• Forms anterior choroid plexus.
structure, forming corpus callosum (large bands of myelinated fibres).
• Short pineal stalk, have pineal body, secretes hormone
anterior part of brain, • Cerebral cortex forms outermost portion of
melatonin.
concerned with sense cerebrum and makes up the grey matter.
of smell. • The surface of cortex is highly folded, forming gyri Thalamus (sides)
(upward folds) alternating with sulci (downward • Lies superior to mid-brain.
• Each lobe consists of grooves).
two parts - anterior • Composed primarily of grey matter.
• Beneath the grey matter, millions of medullated
olfactory bulb and nerve fibres are present, which give opaque white Hypothalamus (floor)
posterior olfactory appearance and comprise of white matter. • Hypothalamus lies above the pituitary gland and is
tract. • Each hemisphere consists of frontal, parietal, attached to it by stalk called infundibulum.
temporal and occipital lobes. • Maintains homeostasis, provides anatomical connections
• Contains sensory, motor and association areas. between nervous and endocrine systems.

16 MT BIOLOGY TODAY | AUGUST ‘17


Mid-brain (Mesencephalon)
• Passage to hind brain.
• Contains narrow canal called iter/aqueduct of Sylvius (mesocoel).
Corpora quadrigemina Cerebral peduncles (crura cerebri)
• 2 pairs of rounded protrusions present on upper or superior surface

Mid-brain
of mid brain are called corpora quadrigemina.
• Crura cerebri are two bundles of fibres
present on lower or inferior surface of the
• One pair is called superior colliculi (concerned with sense of mid brain.
sight) and other pair is inferior colliculi (concerned with hearing).
• The superior and inferior colliculi of each side are termed corpora • They relay impulses back and forth between
bigemina. the cerebrum, cerebellum, pons and medulla.

Hind brain
(Rhombencephalon)

Cerebellum Pons varolii Medulla oblongata

• Second largest part of the human brain, consists of 3 • It is situated in front of cerebellum • Pyramid shaped, extends
layers, outer layer of cells, cerebellar cortex and deeper below the mid brain and above the from pons varolii above and
cell clusters. Middle layer contains characteristically medulla oblongata. is continuous with spinal cord
large flask shaped Purkinje cells. below.
• Functions: It relays impulses
• Cerebellum has two lateral cerebellar hemispheres • It has thin, non-vascular folded
between the medulla oblongata
and central worm shaped part, vermis. structure on its lower side called
and superior part of brain, between
• It contains branching tree like arrangement of grey posterior choroid plexus.
hemispheres of cerebellum and
and white matter called arbor vitae (tree of life). • Functions: Regulatory centres
between cerebrum and cerebellum.
• Functions: Controls posture, balance and mainly for heart rate, blood pressure,
• Pneumotaxic centre present in pons breathing, sneezing, salivation,
involuntary activities, receives information from varolii limits inspiration.
muscles, joints, skin, eyes. coughing, etc.

Motor Area Parietal Lobe


• Lies in the frontal lobe • Sensations
• Transmit impulses to effectors • Language
• Control of voluntary muscles • Perception
Sensory Area • Body awareness
• Lies in the parietal lobe • Attention
• Receive impulses from receptors
Occipital Lobe
• Perceive skin sensations
• Vision
(temperature, pressure, pain)
• Perception
Frontal Lobe
• Muscle movement Wernicke’s Area
• Control intellectual ability to • Lies in the left temporal lobe
problem solving, reasoning • Language comprehension
• Concentrating, thinking Cerebellum
• Behaviour, personality, mood
• Posture
Broca’s Area
• Balance
• Lies in the frontal lobe
• Coordination of movement
• Speech control
Temporal Lobe Brain Stem
• Hearing and smell • Consciousness
comprehension • Breathing
• Language • Heart rate
• Memory Fig.: Functional areas of the brain
• Emotions

MT BIOLOGY TODAY | AUGUST ‘17 17


Human Spinal Cord
• Spinal cord (42 to 45 cm long) extends from the medulla oblongata and is continuous to the level of second lumbar
vertebra. It conducts impulses to and from the brain and controls most of the reflex activities.
• Inner butterfly shaped area of spinal cord is the grey matter as it lacks
myelin and surrounding grey matter is the bundles of myelinated nerve
fibres, which form white matter.
• The area within the vertebral column below the second lumbar vertebra
contains spinal nerves that are collectively called, the cauda equina. Skull bone Skull bone
• The spinal cord ends as the conus medullaris from which a fine filament Foramen magnum C1 nerve
called filum terminale arises that anchors spinal cord within the vertebral C1 vertebra C1 C1 vertebra
column. Dorsal side Ventral side
• The spinal cord is surrounded by three protective membranes; innermost Duramater
piamater, middle arachnoid mater and outer, tough duramater. The sub- Arachnoid mater
arachnoid space is filled with cerebrospinal fluid and the additional epidural Piamater
space, above duramater contains fatty and connective tissues and veins. C8 C7 vertebra
C7 vertebra
T1
Functions T1 vertebra T1 vertebra
T1 nerve
• Spinal cord performs two main functions:
(i) The stimuli are passed from and to the brain through the spinal cord. Fig. : Coverings of human spinal cord
(ii) It is the centre of spinal reflex action.
REFLEX ACTION
• Reflex action (Marshal Hall, 1833) is an immediate involuntary action of any organ or part of the body in response to a particular stimulus.
• The nervous pathway taken by nerve impulses in a reflex action is called reflex arc. The components that mediate a reflex,
usually includes a receptor, afferent pathway, integrating centre, efferent pathway and effector. Hence, the entire impulse circuit
of a reflex response is -

Stimulus Receptors Sensory or afferent nerve fibres CNS


(External or internal (A tissue/organ/cell which (Take sensory impulse (May be spinal cord or brain)
stimuli) receive an external or generated by receptor to CNS)
internal stimulus, e.g.,
skin, eye, ear)
Response Effector Motor or efferent nerve fibres
(Movement, secretion, (May be organ/muscle/gland which on being (Carry motor impulse generated
behaviour) activated produces work or substances in response in CNS to the effector organs)
to specific stimulus)
Fig.: Reflex arc
Dorsal root
ganglion Synaptic
(3) Sensory neuron Cell body knob Synapse
Direction Dorsal horn
Dorsal
of impulse
Sensory nerve root White matter (axons
(1) Pin (stimulus) ending in skin running to and from brain)
Receptor is a pain receptor (2) Receptor Interneuron
or a mechanoreceptor sets up nerve Central canal (contains cerebrospinal fluid)
(sensitive to pressure impulse Grey matter (cell bodies
touch). (4) Motor and interneurons)
neuron
Ventral horn
(5) Biceps muscle- T.S spinal cord
(6) Finger pulled away Synapse
from pin (response) contracts (effector)
(5) Triceps muscle-relaxes (effector)
Fig.: Reflex action and reflex arc

18 MT BIOLOGY TODAY | AUGUST ‘17


Types of Reflexes

Simple or Unconditioned reflex Conditioned reflex


• A simple reflex is inborn, unlearned response to a stimulus or any change in • Conditioned reflexes are not inborn but are acquired
the environment. Most reflex actions are protective in nature. during the life time of an individual. It was first
• In this reflex, the nerve impulse is initiated when the sensory nerve endings demonstrated by I.P. Pavlov. He presented a hungry
in the receptor organ are stimulated. The impulse travels along the afferent dog with food, which elicited a reflex of salivation by
or sensory neuron until it reaches the terminal branches of the axon forming the stimulation of the taste buds in the mouth. He then
a synapse with the dendrites of the connector neuron. In the spinal cord, rang a bell each time the dog was given the food to
the impulse proceeds along the length of this connector neuron to its associate the unconditioned response (salivation) with
axon endings which form another synapse with the dendrites of the motor an additional stimulus (bell). After a few trials, he was
neuron. At this synapse an impulse is initiated in the efferent or motor able to demonstrate that the dog salivated at the sound
neuron. The terminal axon branches of the motor neuron end in the effector of the bell without the sight or smell of food. This is thus
organ. The reflex or response made by the effector organ as a result of the known as conditioned reflex.
arriving nerve impulse is either muscular contraction or glandular secretion, • These are very important for studying the process of
depending on the effector involved. learning and behaviour in animals.

Inhibition of Reflex Action


• In the infant the act of micturition and defecation are carried out by reflex action and there is no voluntary control over these
processes. As the child grows, the sensory nerve tracts of the spinal cord become fully developed, the impulses from the bladder
and rectum reach the brain and can be controlled. Hence, now the act of micturition and defaecation become a conscious
voluntary act in the adult.
Peripheral Nervous System
• The nerves running outside brain and spinal cord constitute peripheral neural system. These nerves are of two types: cranial nerves
(nerves originating from brain) and spinal nerves (nerves originating from spinal cord).
Cranial Nerves
• There are 12 pairs of cranial nerves. They are called so as they pass through various openings in cranial bone.
• Some cranial nerves are sensory (carry impulses from receptor to CNS), a few are motor (conduct impulses from CNS to effector)
and others are mixed (have both sensory and motor neurons).
Table: 12 pairs of cranial nerves in man
Name Origin Distribution Nature Functions

1. Olfactory Olfactory epithelium in Olfactory bulb of olfactory lobes of Sensory Smell


nasal cavity brain

2. Optic Retina of eye Optic lobe of midbrain Sensory Sight (Retina of eye)

3. Oculomotor Floor of midbrain Eye, 4 muscles of eyeball Motor Movements of eye-ball, iris, lens,
eyelid and constriction of pupil

4. Trochlear Floor of midbrain Superior oblique muscles of eyeball Motor Rotation of eye-ball
(Smallest and
thinnest cranial
nerve)

5. Trigeminal (Largest Ventral surface of pons Mixed


cranial nerve) varolii
Divided into three
branches:
(i) Opthalmic • Lacrimal glands, conjunctiva of eye, Sensory Skin sensations
skin of forehead, eyelids, etc.
(ii) Maxillary • Cheeks, upper gums, upper teeth Sensory Sensations
and lower eyelids
(iii) Mandibular • Teeth and gums of lower jaw, pinna Mixed Sensations, tongue movements,
of the ear, lower lip and tongue mastication

MT BIOLOGY TODAY | AUGUST ‘17 19


6. Abducens Pons varolii External rectus muscle of eye-ball Motor Rotation of eye-ball

7. Facial (bears Lower part of pons Anterior 2/3 tongue (taste buds), Mixed Taste, facial expression,
geniculate ganglion) varolii muscles of face, neck and salivary chewing, movement of neck
glands

8. Auditory (also called Comes from internal Organ of Corti in cochlea, semicircular Sensory Hearing and equilibrium
vestibulo-cochlear) ear and joins lateral canals
side of pons varolii

9. Glosso-pharyngeal Lateral side of medulla Posterior 1/3rd of tongue, soft palate Mixed Taste and touch, movements
and muscles of pharynx (swallowing) of pharynx,
salivation

10. Vagus (Pneumogas- Lateral side and floor Muscles of pharynx, vocal cords, Mixed Vocal cords (sound production),
tric) (Longest of medulla lungs, heart, oesophagus, stomach. lungs, respiratory reflexes,
cranial nerve) Also called wandering nerve as peristaltic intestine movements,
(i) Superior laryngeal it has maximum branches speech, swallowing, secretion
(ii) Recurrent laryngeal of gastric glands, inhibition of
(iii) Cardiac heart beat
(iv) Pneumogastric
(v) Depressor

11. Spinal accessory or Both medulla and Muscles of palate, larynx, vocal cords, Motor Movement of muscles of
accessory nerve spinal cord neck, shoulder pharynx, larynx, neck, shoulder

12. Hypoglossal Ventral side of medulla Muscles of tongue, hyoid apparatus Motor Movement of tongue
oblongata

Spinal Nerves
• There are 31 pairs of spinal nerves, named and numbered according to the vertebrae with which they are associated.

Thoracic nerves Sacral nerves


(12 pairs) (5 pairs)
31 pairs of
spinal nerves
Cervical nerves Lumbar nerves Coccygeal nerves
(8 pairs) (5 pairs) (1 pair)

• Structure - The spinal nerves are formed by union of dorsal and ventral roots shortly after they leave spinal cord. Each
spinal nerve has afferent (sensory) and efferent (motor) fibres, in general, efferents come from ventral root and afferents go
into dorsal root. Thus, all spinal nerves are mixed nerves because they carry both sensory and motor impulses.
• After leaving vertebral column, each spinal nerve divides into:
– Posterior branch- innervates muscles and skin of the posterior portion of the body.
– Anterior branch- innervates limbs and the lateral and anterior portions of the body.
Cervical plexus Lumbar plexus
The main spinal Innervates neck and diaphragm Innervates the legs
nerves join to
form plexus Coccygeal plexus Brachial plexus Sacral plexus
Innervates skin and pelvic region Innervates chest and arm Connects pelvic region

Autonomic Neural System


• Autonomic neural system (ANS) is a special self governed autofunctioning system of ganglia and peripheral motor nerve fibres
(efferent) which innervate various organs and glands of the body to stimulate, accelerate, slow down or inhibit their functions
without directly consulting the will. ANS is made of two opposing divisions, sympathetic and parasympathetic neural system.

20 MT BIOLOGY TODAY | AUGUST ‘17


Parasympathetic division Sympathetic division

Eye
Constricts pupil Dilates pupil
Salivary
glands
Stimulates saliva secretion Inhibits saliva secretion

Lung
Constricts bronchi Dilates bronchi

Slows heart beat Accelerates heart beat


Heart
Adrenal gland
Stimulates epinephrine and
Promotes glycogen formation Liver
Stomach norepinephrine release

Pan
c rea Stimulates glucose
Stimulates stomach, pancreas, s
and intestine secretion Intestines Inhibits stomach, pancreas, and
intestine secretion
Bladder
Constricts bladder Relaxes bladder

Promotes erection of genitals Promotes ejaculation and vaginal


Genitalia contractions
Fig.: Functions of autonomic neural system

Table: Differences between sympathetic and parasympathetic neural system


Sympathetic neural system Parasympathetic neural system
Anatomical differences
(i) It has paired chains of ganglia and other visceral ganglia. Chains of ganglia are absent. It has ganglia very close to
the organ supplied.
(ii) Its preganglionic fibres originate from the spinal cord. Its preganglionic fibres originate from the brain and spinal
cord. Those preganglionic fibres which come from the brain
run along with the III, VII, IX and X cranial nerves while
those coming from spinal cord pass through 2,3 and 4 sacral
spinal nerves.
(iii) Its preganglionic fibres are shorter than the postganglionic Its preganglionic fibres are much longer than the
fibres. postganglionic fibres.
(iv) Each preganglionic fibre forms synapses with many Each preganglionic fibre synapses with only a few
postganglionic fibres, so many organs are affected. postganglionic fibres, thus, only one organ may be affected.
Physiological differences
(v) Its postganglionic fibres are adrenergic i.e., release the Its postganglionic fibres are cholinergic i.e., release the
neurotransmitter noradrenaline (norepinephrine). neurotransmitter acetylcholine.
(vi) It stimulates the medulla of the adrenal glands to release None.
epinephrine and norepinephrine.

SENSORY RECEPTORS
• The principle function of the special sensory receptors is to detect environmental stimuli and transduce their energy into
electrical impulses. These are then conveyed along sensory neurons to the central nervous system, where they are integrated
and processed, and a response is produced.

MT BIOLOGY TODAY | AUGUST ‘17 21


Types of receptors
On the basis of stimuli they receive

Mechanoreceptors Chemoreceptors
Respond to mechanical forces such as touch, pressure, vibration Respond to chemical molecules. Include olfactory cells in the nasal
and itch by generating nerve impulses. Includes the receptors mucosa and taste buds.
such as free nerve endings of sensory neurons, Merkel’s discs, hair
follicle endings, Pacinian corpuscles, muscle spindles and Golgi Nociceptors
tendon organs. Respond to painful stimuli which may be damaging to tissues. Include
free nerve endings.
Thermoreceptors
Respond to temperature changes. Include free nerve endings and Photoreceptors
Krause’s end bulbs. Respond to light energy. Include cones and rods in the retina.

• The most complex sensory receptors, are called sensory organs. They consist of numerous sense cells, sensory neurons
and associated accessory structures, e.g., eye and ear have a level of complexity of sense organs.
Eyes (Organs of Sight)
• Human eyes are spherical structures present in the bony sockets of the skull. Each eye is about 2.5 cm in diameter and
consists of tissues present in three concentric layers:
(i) Outer most layer consists of sclera and cornea.
(ii) Middle vascular layer consists of choroid, ciliary body and iris (also called uvea).
(iii) Inner most layer consists of retina.

DAVID BALTIMORE
D
avid Baltimore is an American attention to two RNA tumour viruses– Rauscher murine leukemia virus
biologist, University administrator and Rous sarcoma virus. It was through these experiments that he
and a Nobel laureate in Physiology discovered reverse transcriptase. This discovery proved an exception
or Medicine. From 1997 - 2006, he served to the “central dogma” of genetic theory. Since its discovery reverse
as President of California Institute of transcriptase has become an invaluable tool in recombinant DNA
Technology (Caltech) and is currently the technology. Reverse transcriptase is essential for the production of
President Emeritus and Robert Andrews retroviruses, allowing such viruses to turn viral RNA strands into DNA
Millikan Professor of Biology at Caltech. strands. They published these findings in the prestigious journal Nature.
He was born on March 7, 1938 in New York In 1974, Baltimore was honored as a Fellow of the American Academy of
City to Gertrude and Richard Baltimore. He Arts and Sciences. In 1975, he shared the Nobel Prize for Physiology or
graduated from Great Neck High School Medicine with Howard Temin and Renato Dulbecco. In 1980, Baltimore
in 1956 and earned his Bachelor’s degree and his group published a paper regarding Abolson murine leukemia
in Chemistry from Swarthmore College
virus (AMuLV) and showed it was a member of new class of protein
Pennsylvania in 1960. He developed his interest in molecular biology
kinase that used the amino acid tyrosine as a phospho-acceptor. In
at cold spring harbor laboratory. He obtained his PhD in 1964 from
1999, Former U.S. President Bill Clinton awarded Baltimore, the National
Rockefeller University, New York. He made fundamental discoveries
Medal of Science for his contribution to the scientific world. In 2005, he
on virus replication and its effect on cell metabolism, including the first
became member of the Encyclopedia Britannica Editorial Board Advisor
description of RNA replicase.
and was elected in 2006 as President of American Association for
In February, 1965, Baltimore was recruited by Renato Dulbecco to the advancement in science (AAAS). His laboratory at Caltech focus on two
newly established Salk Institute for Biological Studies in La Jolla, there major research areas understanding the mammalian immune system
he investigated poliovirus RNA replication. He joined the faculty of MIT and creating viral vectors to make the immune system more effective
in 1968, accompanied by Alice S. Huang, a post doctoral fellow and the in resisting cancer. In recent research led by Jimmy Zhao, Baltimore’s
two worked together on vesicular stomatitis virus (VSV). Baltimore and team has discovered a small RNA molecule called microRNA-146a and
Huang showed that vesicular stomatitis virus, an RNA virus reproduced bred a strain of mice that lacks miR146a. Their results suggest that
itself by means of an enzyme RNA-dependent RNA polymerase that microRNA-146a protects HSCs during chronic inflammation and that its
copies RNA by a process not involving DNA. In 1972, he was awarded lack may contribute to blood cancers and bone marrow failure.
tenure as a professor of Biology at MIT. In 1973, he became a professor
of Microbiology at an American Cancer Society. He then turned his 

22 MT BIOLOGY TODAY | AUGUST ‘17


MT BIOLOGY TODAY | AUGUST ‘17 23
Structure of Human Eye
Sclera Choroid
Tough, outermost, protective, Middle vascular, dark, pigmented layer, which absorbs light
Ora serrata fibrous, opaque coat made entering the eye and stop it from reflecting back within eyeball.
Ciliary body Demarcates sensitive up of dense connective Blood vessels supply nutrients and oxygen to other tissues,
Made up of smooth muscle, part of retina from its tissue. It is white in colour especially retina.
changes the shape of the lens non sensory part. and gives shape to eyeball.
depending upon distance of Retina
object to bring images into Innermost neural and sensory layer
focus. containing light sensitive cells, which
send impulses through the optic nerve
Ciliary zonule to brain.
(Suspensory ligament)
Holds the lens in place and connects Macula lutea (Yellow spot)
it to the ciliary muscles. A small, oval, yellowish area on retina
lying exactly opposite the centre of the
Cornea cornea.
Anterior transparent area of the
sclera, which admits and focuses Fovea centralis
light into eyeball. The cornea is A shallow depression in the
avascular and absorbs oxygen from middle of yellow spot, has
the air. cone cells only, devoid of rods
and blood vessels. It is the place
Iris of most distinct vision.
Pigmented, opaque, muscular
structure of eye, which gives colour Optic nerve
to the eye and regulates intensity of Carries image impulses
light entering the eye either by to brain.
constriction or dilation of pupil. Central artery and vein of
the retina
Pupil
Supply blood to the eyeball.
A hole in the centre of the iris,
through which light enters the eye. Blind spot
Pupil dilates in dark to permit more Point where optic nerve leaves
light in and contracts in bright light the eyeball, devoid of light
to reduce light falling in. sensitive cells to detect image.

Conjunctiva
Aqueous humor Lens Vitreous humor
Thin, clear, protective
Watery liquid, formed by capillaries of ciliary Tra n s p a r e n t , b i c o n v e x , Clear jelly-like fluid that fills space
front layer over the
processes, that fills space between the cornea and circular body lying immedia- between the lens and retina,
surface of the eye and
lens. It provides nutrition to avascular structures of tely behind pupil. It forms which helps to maintain the
lining the eyelids. An
the eye, i.e., cornea and lens and maintains image on retina. It separates shape and inner pressure of
infection of eye called
intraocular pressure and helps in image forming aqueous and vitreous eyeball. It allows undistorted light
conjunctivitis occurs
mechanisms. humors. to fall on retina.
here.

Mechanism of Vision

Light from object Light is focussed Light induces dissociation


pass through on retina, where of retinene from opsin
cornea, aqueous it is converted into which activates transducin, JUNE 2017
humor, lens and potentials in rods thereby causing potential 1-c- ENDOMETRIUM 2-a- MONGOLISM
vitreous humor. and cones. generation.
3-b- ANGIOPLASTY 4-g- TROPHALLAXIS
5-h- ERYTHROPOIETIN 6-e- FUCOXANTHIN
Potential generated 7-d- INTERDIGITATION 8-f- PHYTOPHAGOUS
Neural impulses Action potential is in photoreceptor
transmitted by optic
9-j- APOSPORY 10-i- RAMENTA
are analysed and c e l l s t r i g g e r s
Winners : Chaitanya Vattem (Hyderabad), Aswini Mura (Purba Medinipur), Pranjal
erect image is nerve to visual area action potential in
Meshram (Chattisgarh), Prakriti Saini (Rohtak), Jennifer Minz (Bengaluru)
recognised. of the brain. ganglion cells.

24 MT BIOLOGY TODAY | AUGUST ‘17


External limiting membrane
Formed by the glial tissues, it is the continuation of internal limiting
membrane and is pierced by the rods and cones.
Outer nuclear layer
Formed by the cell bodies and nuclei of rods and cones.
Pigmented epithelium
It contains melanin pigment Rods
which along with the Layers of Retina Contain photosensitive pigment rhodopsin.
pigmented choroid absorbs The rods mainly enable to see in darkness,
light and prevents the therefore are present in large numbers in
reflection of rays back nocturnal animals.
within eyeball which may
distort the image formation. Cones
Contain photosensitive pigment photopsin,
Nuclei involved in colour vision.

Rod end bulb Outer plexiform layer


Axons of rods and cones synapse here with
Cone plate
dendrites of bipolar cells and horizontal cell
Horizontal cell processes.
Connect one receptor cell to
another receptor cell. Inner nuclear layer
Contains cell bodies and nuclei of bipolar,
Bipolar cells neurons, horizontal neurons and amacrine
Amacrine cell cells.
Processes make synaptic
contacts with dendrites of
both ganglion and bipolar cells
and connect ganglion cells to
one another.
Inner limiting membrane
Ganglion cell It separates the retina from the
vitreous humor. It is formed by the Inner plexiform layer
It gives rise to optic nerve fibres and sends It is the site of major processing
impulses to brain. glial tissues.
of the visual image. Axons of
Optic nerve fibres bipolar neurons and amacrine
Ganglion cell layer Formed by joining the axon of ganglion cells synapse with the dendrites
A single layer of cell containing cell bodies cells; here all the axons run parallel. of ganglion cells.
of ganglion cells.

Fig.: Schematic diagram to show layers of retina and main structures therein

Ear (Organs of Hearing)


• The ear is the organ that detects sound. It not only acts as a receiver for sound, but plays a major role in the balance and
position of body. It contains receptors that respond to movements of the head and receptors that convert sound waves into
nerve impulses.
• Anatomically, the ear can be divided into three major sections:
external, middle and inner ear.
• External ear comprises of two parts, pinna and external auditory
canal.
• Pinna is an oval, somewhat funnel-shaped, skin-covered flap of
elastic cartilage and muscles. It collects sound waves and directs
them into the external auditory canal.
• Middle ear is formed of an air filled cavity called tympanic cavity
in the temporal bone that opens via the auditory (Eustachian) tube
into the nasopharynx.
• The tympanic membrane separates the tympanic cavity from
external auditory canal.

MT BIOLOGY TODAY | AUGUST ‘17 25


• Ear ossicles are present which are a flexible chain of three small bones: the malleus, the incus and the stapes. The
function of the ossicles is to transmit and amplify sound waves across the tympanic cavity from the tympanic membrane to
the oval window of the inner ear (fenestra ovalis).
• Inner ear is a delicate, irregular organ called membranous labyrinth surrounded by bony labyrinth and separated from
it by a narrow perilymphatic space. This space contains a watery fluid called perilymph. The membranous labyrinth is
filled with another fluid called endolymph.
• The coiled portion of the labyrinth is called cochlea. It is the main hearing organ connected to the saccule. Internally, it
consists of three fluid-filled chambers or canals. The middle chamber, called scala media, bears an upper membrane, the
Reissner’s membrane, and lower membrane, basilar membrane. On the basilar membrane a sensory ridge, the organ
of Corti is present. Organ of Corti consists of hair cells (phonoreceptors), which bear ‘hair’ at the free surface and have
synaptic contacts with the dendrites of neurons at the bases. The tips of ‘hair’ are embedded in a thin elastic membrane
called tectorial membrane that determines the vibrations of sound waves.
• The inner ear also contains a complex system called vestibular apparatus. The vestibular apparatus is responsible for
maintenance of balance of the body and posture.

Pinna

Helix

Cartilage

Sound waves are collected from Tympanic membrane (eardrum) Tympanic membrane
an external source by the help of stretches; and as the air molecules bows inwards and
Mechanism ear pinna and are transmitted to push the membrane, they cause it transmits the sound
of Hearing tympanic membrane through an to vibrate at the same frequency waves to the ear
external auditory meatus. as the sound wave. ossicles.

Hair cells receive the impulses Movements in fluid (endolymph) Vibrations are further transferred Perilymph of inner
and transmit it to brain through of scala media and tectorial to scala vestibuli and then to ear receives the
auditory nerve and finally sound membrane stimulate the sensory scala media through Reissner’s vibrations through
is detected by the brain. hair of the organ of Corti. membrane of cochlea. fenestra ovalis.

MPP-4 CLASS XII ANSWER KEY


1.
6.
(a)
(b)
2.
7.
(d)
(d)
3.
8.
(c)
(a)
4.
9.
(d)
(a)
5.
10.
(c)
(c)
Winners
11. (c) 12. (c) 13. (a) 14. (d) 15. (c)
16. (b) 17. (a) 18. (b) 19. (b) 20. (b) June-2017
21. (d) 22. (c) 23. (d) 24. (b) 25. (c) 1. Nidhi Agarwal, Dwarka (New Delhi)
26. (d) 27. (a) 28. (b) 29. (a) 30. (a)
31. (d) 32. (b) 33. (d) 34. (c) 35. (a) 2. Shobhit Singh (Meerut)
36. (c) 37. (d) 38. (a) 39. (b) 40. (c)

26 MT BIOLOGY TODAY | AUGUST ‘17


1. Increased olfactory sensation refers to 7. Which part of human eye corresponds to light hole of
(a) hyperosmia (b) ageusia camera?
(c) diplopia (d) alexia. (a) Lens (b) Pupil
2. Select correct sequence of conduction of nerve impulse. (c) Retina (d) Cornea
(a) High concentration of K+ in axoplasm → Threshold 8. Identify A, B, C and D in the given A
stimulus → Influx of Na+ → Action potential figure showing T.S. of cochlea and
generation → Efflux of K+ select the incorrect option. B
(b) High concentration of Na+ in axoplasm → (a) A and D are connected with
Depolarisation → Threshold stimulus → Repolarisation other through helicotrema.
→ Action potential generation (b) B determines the patterns C
(c) High concentration of K+ in the fluid outside axon → of vibration of sound waves. D
Threshold stimulus → Influx of K+ → Action potential (c) C is a sensory ridge present on
→ Efflux of Na+ the Reissner’s membrane.
(d) Resting potential → Depolarisation → Threshold (d) D is filled with endolymph.
stimulus → Efflux of Na+ → Action potential → 9. Match the following and select the correct option.
Repolarisation
Column I Column II
3. Which of the following pair is incorrect? A. Pons varolii (i) Thirst and centres satiety
Cerebral lobe Function
B. Cerebellum (ii) Cardiac centre
(a) Parietal lobe – Register and communicate
information from environment C. Hypothalamus (iii) Maintains body posture
(b) Occipital lobe – Interpretation of shape and colour D. Medulla oblongata (iv) Limits inspiration
(c) Frontal lobe – Judgement and decision making (a) A-(iv), B-(iii), C-(i), D-(ii)
(d) Temporal lobe – Controls intellectual ability (b) A-(iii), B-(i), C-(ii), D-(iv)
4. Which of the following pair is an example of unconditioned (c) A-(ii), B-(i), C-(iii), D-(iv)
reflex? (d) A-(iv), B-(i), C-(iii), D-(ii)
(a) Breastfeeding and watering of mouth on seeing food 10. Which of the following functions is performed by the
(b) Blinking of eyes and cycling cerebrospinal fluid?
(c) Swallowing in new born baby and blinking of eyes (a) Excretion of harmful metabolic wastes
(d) Salivation on seeing food and cycling (b) Transport of hormones to the brain
5. The part of the brain associated with regulation of body (c) Provides buoyancy to brain
temperature is (d) All of these
(a) corpus callosum (b) cerebellum 11. Select the incorrect difference between sympathetic and
(c) hypothalamus (d) corpora bigemina.
parasympathetic neural system.
6. Select an option that correctly fills the blanks.
I. Abducens nerve controls the movement of ______. Sympathetic Neural Parasympathetic
II. ______ is the longest cranial nerve. system Neural system
III. The _____ nerves are motor nerves. (a) Inhibits salivary gland Stimulates salivary
IV. All spinal nerves are of ______ type. secretion. gland secretion.
I II III IV (b) Promotes glycogen Promotes glycogen
(a) shoulder Trochlear optical and motor breakdown. formation.
muscle oculomotor (c) Constricts urinary Dilates urinary bladder.
(b) pharynx Mandibular facial and olfactory sensory bladder.
(c) tongue Trigeminal facial and vagus mixed (d) Postganglionic fibres Postganglionic fibres are
(d) eye ball Vagus hypoglossal mixed are adrenergic. cholinergic.
and oculomotor
MT BIOLOGY TODAY | AUGUST ‘17 27
12. Read the given statements and select the correct option. 18. Read the given statements and select an option stating
I. The continuity between the pre-synaptic and post- which ones are true (T) or false (F).
synaptic neurons is provided by gap junctions in I. Ageing neurons contain a pigment made up of a
chemical synapses. residual bodies derived from lysosomes.
II. Release of calcium ions from pre-synaptic knob
II. Axon and cyton contains numerous mitochondria,
stimulates the release of neurotransmitter into synaptic
Golgi apparatus, lysosomes, etc.
cleft.
III. More energy is required for impulse conduction through III. Cyton depends on axon for supply of proteins
myelinated neurons due to saltatory conduction. synthesised by Nissl’s granules.
IV. After transmission of impulse, neurotransmitter is I II III
hydrolysed by an enzyme present at synapse. (a) T T F
(a) I and II are correct but III and IV are incorrect. (b) T F F
(b) I and IV are correct but II and III are incorrect. (c) F T T
(c) I, II and III are correct but IV is incorrect. (d) F F T
(d) I, II and III are incorrect, but IV is correct. 19. Read the following statements and select the correct
13. Which receptors respond to the heat? option.
(a) Nociceptors (b) Algesireceptors Statement A : Sweat secretion is under the control of
(c) Caloreceptors (d) Tangoreceptors autonomic neural system.
14. Meninx piamater of brain is Statement B : Involuntary activities of various organs are
(a) innermost, vascular layer under control of autonomic neural system.
(b) innermost layer with spider webby structure (a) Both statements A and B are correct.
(c) middle, tough fibrous layer (b) Statement A is correct but statement B is incorrect.
(d) outermost, avascular layer. (c) Statement A is incorrect but statement B is correct.
15. Read the given statements and select the incorrect one. (d) Both statements A and B are incorrect.
(a) Fenestra rotunda, an opening of middle ear into inner 20. Which plexus formed by spinal nerves innervates the pelvic
ear is covered by foot plate of the stapes. region?
(b) The iris controls the amount of light entering the eye by (a) Cervical plexus (b) Sacral plexus
contracting the radial and circular muscles in dim and (c) Lumbar plexus (d) Coccygeal plexus
bright light respectively.
21. Scattered masses of grey matter present in limbic system are
(c) Glands of Moll and glands of Zeis in eye are modified
(a) septal nuclei (b) amygdala nuclei
sebaceous glands which produce oily secretion to
(c) basal ganglia (d) mammillary bodies.
lubricate the corneal surface.
(d) Transformation of the vibrations into nerve impulses 22. Which branch of largest cranial nerve have both sensory and
for relay to brains takes place in the internal ear. motor fibres?
(a) Maxillary (b) Ophthalmic
16. Photosensitive pigment found in cones
(c) Mandibular (d) Both (b) and (c)
(a) scotopsin (b) photopsin
(c) rhodopsin (d) porpyrosin. 23. Which of the following statements is incorrect regarding
vitreous humour present in an eye?
17. Read the given passage and select the correct option to fill
(a) It is semi-solid jelly like substance present between
the blanks.
cornea and lens.
Neurons are the structural and functional unit of neural
(b) It maintains shape of an eyeball and provides nutrients
tissue and are longest cells in the body. Fully formed neurons
to lens and cornea.
remain in (i) of cell cycle throughout the life. Neurons with
(c) It is continuously secreted by ciliary processes and is
flask shaped cytons are called (ii) and are located in
drained into canal of schlemm.
(iii) part of the brain.
(d) None of these
(i) (ii) (iii)
(a) G0 phase Purkinje cells Cerebrum 24. Action of which neurotransmitter is inactivated by an
(b) Interphase Purkinje cells Cerebellum enzyme monoamine oxidase?
(c) Interphase Perikaryon Pons varolli (a) Acetylcholine (b) Norepinephrine
(d) G1 phase Schwann cells Diencephalon (c) GABA (d) Serotonin

28 MT BIOLOGY TODAY | AUGUST ‘17


25. Refer to the given figure and identify the correct option. 27. During nerve impulse conduction, a membrane gets
Receptor depolarised at
(a) 0 mV (b) + 30 mV
Y
(c) –70 mV (d) –55 mV.
W
28. The neurotransmitter released at all neuromuscular junctions
between motor neurons and skeletal muscle cells is
(a) norepinephrine (b) dopamine
Z
(c) acetylcholine (d) serotonin.
X 29. Which disease is caused due to destruction of neurons of
basal ganglia that produce dopamine?
Transverse section
through spinal cord
(a) Neuralgia
Motor nerve fibre
(b) Multiple sclerosis
(c) Parkinson’s disease
Effector (muscle)
(d) Korsakoff’s syndrome
(a) W is the grey matter containing bundles of myelinated 30. ______ is continuous with central canal of the spinal cord.
nerve fibres. (a) Cerebral aqueduct (b) Fourth ventricle
(b) X represents white matter of spinal cord and is so (c) Foramen of Monro (d) Third ventricle
named as it lacks myelin.
(c) Y is dorsal root ganglion containing cell bodies of ANSWER KEY
sensory neurons. 1. (a) 2. (a) 3. (d) 4. (c) 5. (c)
(d) Z is interneuron that forms synapse with cell bodies of 6. (d) 7. (b) 8. (d) 9. (a) 10. (d)
motor neurons confined to grey matter. 11. (c) 12. (d) 13. (c) 14. (a) 15. (a)
26. The area of the cerebral cortex responsible for understanding 16. (d) 17. (b) 18. (b) 19. (a) 20. (d)
speech and writing words is 21. (c) 22. (c) 23. (d) 24. (b) 25. (d)
(a) parietal association area 26. (c) 27. (a) 28. (c) 29. (c) 30. (b)
(b) Broca’s area
(c) Wernicke’s area (d) Frontal association area. 

UN S C R A M B L E M E
Unscramble the words given in column I and match them with their explanations in column II.
Column I Column II
1. ALIMANINR (a) A layer of connective tissue that surrounds the myelinated nerve fibre.
2. TEMEGAENISS (b) Food reserve found in Sargassum.
3. ONEDENRUUMI (c) Alternation of asexual and sexual phases in hydrozoans.
4. SULGETINL (d) Drug obtained as by-product of heroin synthesis.
5. MECITINRS (e) Parasites that are completely dependent on the host for their nutrition.
6. LAPONYGOYL (f) Secretion from mammary glands of a mother for the first 2-3 days after
child birth.
7. MSAKC (g) Disorder in infants caused due to deficiency of thyroid hormone.
8. LOHAPOARISETS (h) Natural insecticide obtained from margosa extract.
9. HDZTAIARIANC (i) Group of storage proteins found in cereal grains.
10. OCOLTSUMR ( j) Study of external morphology of mature pollen grains.

Readers can send their responses at editor@mtg.in or post us with complete address by 25th of every month to win exciting prizes.
Winners’ names will be published in next issue.

MT BIOLOGY TODAY | AUGUST ‘17 29


Maximise your chance of success in NEET by reading this article. This section is specially designed to optimise your preparation
by practising more and more. It is a unitwise series having chapterwise question bank, allowing you to prepare systematically
and become more competent.
Recall question or single concept question – indicated by a single finger.
Application question or question which requires 2 or 3 concepts - indicated by 2 fingers.
Application question or question which requires 3 or more concepts - indicated by 3 fingers.

UNIT-IV : HUMAN PHYSIOLOGY

CHAPTER-16 : DIGESTION AND ABSORPTION 4. The process by which liver cells synthesise glycogen by the
action of insulin is called
Multiple Choice Questions (a) glyconeogenesis (b) glycolysis
1. Which of the following options correctly depicts the flow (c) glycogenolysis (d) glycogenesis.
of bile from liver? 5. The salivary amylase is inactivated in stomach due to
(a) Liver → cystic duct → hepatic duct → gall bladder → (a) absence of thiocyanate ions
bile duct → hepatopancreatic ampulla → duodenum (b) presence of somatostatin
(b) Liver → hepatic duct → gall bladder → cystic duct → (c) denaturation of enzyme by acidic pH in stomach
bile duct → hepatopancreatic ampulla → duodenum (d) inhibition by enterokinase.
(c) Liver → hepatic duct → gall bladder → cystic duct
→ bile duct → hepatopancreatic ampulla → ileum 6. Which of the following options correctly describes the
(d) Liver → cystic duct → gall bladder → bile duct → sequence of enzymes involved in digestion of proteins?
hepatic duct → hepatopancreatic ampulla → duodenum (i) (ii) (iii)
Proteins Peptones Dipeptides Amino acids
2. Milk casein Y Paracasein + M (a) (i) Pepsin ; (ii) Carboxypeptidase ; (iii) Trypsin
(b) (i) Pepsin; (ii) Chymotrypsin; (iii) Trypsin
Calcium paracaseinate (c) (i) Pepsin; (ii) Chymotrypsin; (iii) Dipeptidase
(Curdling of milk) (d) (i) Trypsin; (ii) Pepsin; (iii) Dipeptidase
In the above question letter ‘Y’ and ‘M’ denote 7. Which of the following statements is correct regarding
(a) rennin and Ca++ respectively absorption of different nutrients?
(b) Ca++ and renin respectively (a) Fructose is absorbed by active transport in stomach
(c) rennin, HCl and Ca++ respectively and small intestine.
(d) renin and Ca++ respectively. (b) Amino acids are absorbed in small intestine by passive
transport.
3. The opening of bile duct before it unites with pancreatic
(c) Fatty acids and fat soluble vitamins are absorbed via
duct is guarded by
facilitated transport in small intestine.
(a) ampulla of Vater (b) sphincter of Oddi
(d) Water soluble vitamins are absorbed by simple diffusion
(c) Thebasian valve (d) sphincter of Boyden.
in small intestine.

30 MT BIOLOGY TODAY | AUGUST ‘17


8. Which of the following options is incorrect regarding (viii) Decreases gastric secretion
digestion of fats? (ix) Increases secretion of bile
(a) Lipase is present in pancreatic juice but absent in (x) Secreted by G-cell
intestinal juice.
(b) Emulsification of fat increases the action of lipase on fat. Passage Based Question
(c) Most of the fat is digested by pancreatic lipase in the 13. Complete the given passage with appropriate words or
small intestine. phrases.
(d) Bile salts convert fat droplets into smaller ones by The enzymatic digestion of food starts in (i) where
reducing their surface tension. digestion of starch is carried out by (ii) in a slightly
9. Which one of the following is the correct matching of the (iii) environment. Presence of (iv) in stomach inhibits
action of gastric amylase on starch. When chyme reaches
site of action on the given substrate, the enzyme acting
duodenum, Brunner's glands secrete (v) to withstand
upon it and the end product ? the acidity under the influence of vagus nerve and (vi) .
Pepsin
(a) Small intestine : Proteins Amino acids (vii) of pancreatic juice digests the remaining starch into
Lipase maltose, isomaltose and (viii) . These are further broken
(b) Stomach : Fats Micelles
Trypsin down by enzymatic action and converted into (ix) . These
(c) Duodenum : Triglycerides Monoglycerides are absorbed in stomach and (x) by active and facilitated
(d) Small intestine : Starch -amylase Disaccharide (maltose) transport.
10. A typical tooth consists of crown, neck and root regions. Assertion & Reason
Which of the following parts come under the crown region?
(a) Enamel, Dentine, Periodontal ligament In each of the following questions, a statement of Assertion (A)
is given and a corresponding statement of Reason (R) is given
(b) Enamel, Dentine, Cement
just below it. Of the statements, mark the correct answer as :
(c) Enamel, Dentine, Odontoblasts
(a) if both A and R are true and R is the correct explanation of A
(d) Enamel, Dentine, Gum
(b) if both A and R are true but R is not the correct explanation
of A
Match The Columns
(c) if A is true but R is false
11. Match Column I with Column II. (d) if both A and R are false.
Column I Column II 14. Assertion : Many babies experience neonatal jaundice.
A. Valves of Kerkring (i) Controls peristalsis Reason : Neonatal jaundice is caused due to poor
functioning of liver for a week after birth.
B. Auerbach's plexus (ii) Controls intestinal secretion 15. Assertion : Fatty acids are converted to chylomicrons
C. Lacteal (iii) Increase surface area of and transferred into lymph capillaries called lacteals.
absorption Reason : Fatty acids cannot be directly absorbed into blood
D. Meissner's plexus (iv) Absorb fatty acids capillaries by simple diffusion.
16. Assertion : Sodium glycocholate present in bile activates
12. Match Column I with Column II. (There can be more than proelastase.
one match for items in Column I). Reason : Sodium ions of sodium glycocholate act as cofactor
Column I Column II of proelastase.
A. Gastrin (i) Inhibits the release of 17. Assertion : Protein digestion starts in the stomach, not
pancreatic juice in the buccal cavity.
B. Pancreatic polypeptide (ii) Stimulates gastric mobility Reason : An absence of acidic environment in buccal
cavity fails to activate proteases and peptidases.
C. Somatostatin (iii) Secreted by F-cell
18. Assertion : E.coli inhabits the colon but is not killed by
D. Glucagon (iv) Stimulates glycogenolysis body defense system.
E. Secretin (v) Slows down absorption of Reason : E.coli produces cobalamin, thiamine and riboflavin.
nutrients from gastroin-
testinal tract Figure Based Questions
(vi) Secreted by -cells
19. Refer to the following figure showing a longitudinal section
(vii) Inhibits secretion of of small intestinal mucosa and answer the following
glucagon and insulin questions.

MT BIOLOGY TODAY | AUGUST ‘17 31


(b) External nares → Nasal cavities → Internal nares
→ Pharynx → Larynx → Bronchi → Trachea →
A Bronchioles → Alveolar duct → Alveoli
B
(c) External nares → Nasal cavities → Internal nares
→ Pharynx → Larynx → Trachea → Bronchi →
C Bronchioles → Alveolar duct → Alveoli
(d) External nares → Nasal cavities → Internal nares
→ Larynx → Pharynx → Trachea → Bronchi →
D Bronchioles → Alveolar duct → Alveoli
4. Select the incorrectly matched pair.
(a) Identify the structures A, B, C and D. (a) Respiratory membrane - Capillary endothelium
(b) State the importance of finger-like foldings A. (b) Trachea - Ciliated cuboidal epithelium
(c) Name the cells which are present in structure D. What (c) Alveoli - Squamous epithelium
are the functions of these cells? (d) Larynx - Ciliated columnar epithelium
20. Observe the figure showing salivary glands and their ducts 5. Bohr's effect is the phenomenon in which
in human and answer the following questions. (a) low pH of blood due to increase in CO2 concentration
results in decreased dissociation of oxyhaemoglobin
(b) high pH of blood due to increase in O2 concentration
A
results in increased dissociation of oxyhaemoglobin
Q P (c) low pH of blood due to increase in CO2 concentration
results in increased dissociation of oxyhaemoglobin
B (d) high pH of blood due to increase in CO2 concentration
C
R
results in decreased dissociation of oxyhaemoglobin.
(a) Write the name of glands A, B and C. What are the 6. Read the following statements and select the correct one.
functions of these glands? I. The volume of gas that diffuses through the
membrane per minute for a pressure difference of 1
(b) These glands release their secretions into the oral mm Hg is defined as diffusing capacity.
cavity through ducts P, Q and R. Name these ducts. II. The high pCO2 in deoxygenated blood allows the
CHAPTER-17 : BREATHING AND EXCHANGE OF GASES release of CO2 from blood into the alveoli.
III. Diffusion of oxygen is 20 times faster than CO2 and
Multiple Choice Questions that of CO2 is two times faster than nitrogen at the
particular pressure difference.
1. Select the incorrect statements.
IV. Binding of oxygen with haemoglobin is important in
(a) 97% of oxygen is carried as oxyhaemoglobin in the
promoting CO2 transport and its exchange in tissues
blood.
and lungs.
(b) 70% of carbon dioxide is transported as bicarbonate
(a) I, II and IV (b) II, III and IV
ions in blood plasma.
(c) III and IV (d) All of these
(c) 7% of carbon dioxide is transported in dissolved form
in blood plasma. 7. The respiratory disorder which leads to irreversible
(d) 5% of oxygen is transported by blood in dissolved form. distension and loss of elasticity of alveoli of lungs is
(a) bronchitis (b) bronchial asthma
2. Which of the following set of conditions will shift the
(c) emphysema (d) SARS.
oxygen haemoglobin dissociation curve to left?
(a) Low pH, high temperature, increase in pCO2 MPP-4 CLASS XI ANSWER KEY
(b) High pH, low temperature, decrease in pCO2
1. (d) 2. (c) 3. (b) 4. (c) 5. (c)
(c) High pH, high temperature, decrease in pCO2
6. (a) 7. (c) 8. (c) 9. (c) 10. (b)
(d) Low pH, low temperature, increase in pCO2
11. (c) 12. (d) 13. (b) 14. (a) 15. (a)
3. Which of the following shows the correct sequence of 16. (c) 17. (c) 18. (a) 19. (a) 20. (b)
airflow from atmosphere to alveoli of lungs? 21. (b) 22. (b) 23. (c) 24. (a) 25. (a)
(a) External nares → Internal nares → Nasal cavities 26. (c) 27. (a) 28. (a) 29. (d) 30. (a)
→ Pharynx → Larynx → Trachea → Bronchi → 31. (c) 32. (a) 33. (d) 34. (d) 35. (d)
Bronchioles → Alveolar duct → Alveoli 36. (c) 37. (d) 38. (b) 39. (c) 40. (c)

32 MT BIOLOGY TODAY | AUGUST ‘17


8. Hamburger's phenomenon can be described as the (ix) Initiates both inspiration
transport of and expiration
(a) chloride ions from RBCs into plasma (x) Controls depth of inspiration.
(b) bicarbonate ions from RBCs into plasma
(c) chloride ions from plasma into RBCs
Passage Based Question
(d) bicarbonate ions from plasma into RBCs. 13. Complete the given passage with appropriate words or
9. The functional residual capacity can be represented by phrases.
(a) IC + ERV (b) TLC – FRC The exchange of gases between lung alveoli and
(c) ERV + RV (d) IC + RV. pulmonary capillaries is called (i) . It occurs through
(ii) . This membrane consists of alveolar epithelium,
10. During the exchange of gases between lung alveoli and
(iii) , a thin interstitial space, (iv) and (v) . All these layers
pulmonary capillaries, if the partial pressure of oxygen in
form a membrane of (vi) mm thickness. The respiratory
alveolar capillaries is 95 mm Hg and the partial pressure
membrane has a limit of gaseous exchange between
of CO2 is 45 mm Hg in deoxygenated blood. Based on this
alveoli and pulmonary blood called (vii) . The diffusing
identify the correct statement.
(a) The partial pressure of oxygen will be highest, i.e., capacity is dependent on (viii) of the diffusing gases. The
104 mm Hg in alveoli. diffusion of gases from a higher to lower concentration
(b) The partial pressure of nitrogen will remain same in leads to the movement of (ix) from alveoli to (x) .
both blood and alveoli.
(c) The partial pressure of CO2 is less in pulmonary veins Assertion & Reason
as compared to pulmonary arteries. In each of the following questions, a statement of Assertion (A)
(d) All of these is given and a corresponding statement of Reason (R) is given
just below it. Of the statements, mark the correct answer as :
Match The Columns
(a) if both A and R are true and R is the correct explanation of A
11. Match Column I with Column II. (b) if both A and R are true but R is not the correct explanation
Column I Column II of A
A. Bronchial asthma (i) Pulmonary oedema (c) if A is true but R is false
B. Mountain sickness (ii) Fibrosis (d) if both A and R are false.
C. Pneumonia (iii) Release of histamine 14. Assertion : The conversion of CO2 into bicarbonate ions
is thousand times faster in RBCs.
D. Occupational (iv) Inflammation of bronchi
Reason : RBCs contain carbonic anhydrase enzyme.
respiratory disorders
15. Assertion : Exhalation becomes more difficult in patients
E. Bronchitis (v) Inflammation of alveoli
of emphysema.
12. Match Column I with Column II. (There can be more than Reason : In emphysema patients, the surface area for gas
one match for items in Column I). exchange is greatly reduced.
Column I Column II
16. Assertion : External intercostal muscle is considered as
A. Dorsal respiratory (i) Dorsal part of pons Varolii the principle muscle of inspiration.
group Reason : The external intercostal muscle contract to pull
B. Ventral respiratory (ii) Dorsal part of medulla the ribs downward and inward increasing the volume of
group oblongata thoracic cavity.
C. Pneumotoxic centre (iii) Limits inspiration 17. Assertion : In women, thoracic breathing is predominant.
D. Apneustic centre (iv) Ventrolateral part of Reason : Breathing in women occurs through lateral
medulla oblongata movement of thorax.
E. Larynx (v) Adam's apple 18. Assertion : Mammals show positive pressure breathing.
(vi) Lower part of pons Varolii Reason : Positive pressure breathing reduces the chances
(vii) Initiates inspiration of blocking the trachea to allow eating and breathing
(viii) Vocal cords simultaneously.

MT BIOLOGY TODAY | AUGUST ‘17 33


Figure Based Questions (c) Identify E and give the formula for calculating it.
Suggest an example where volume of 'E' is found to
19. Refer to the given figure and answer the following be higher than normal.
questions.
Inspired air Expired air CHAPTER-18 : BODY FLUIDS AND CIRCULATION

Multiple Choice Questions


1. A person with blood group O is considered as universal
CO2 O2 donor because he has
X (a) no antibodies in the plasma but both A and B
CO2 O2 antigens on RBCs.
A B
(b) both anti-A and anti-B antibodies in the plasma but
no antigens on RBCs
(c) only anti-A antibodies in the plasma but no antigens
on RBCs
(d) both anti-A and anti-B antibodies in the plasma but
C D only A antigen on RBCs.
2. Which of the following shows correct sequence of
CO2 O2 conduction of action potential in the heart?
Y (a) SA node → Bundle of His → AV node → Right and
CO2 O2 left branches of bundle of His → Purkinje fibre
(b) SA node → AV node → Purkinje fibre → Bundle of
Body tissues His → Right and left branches of bundle of His
(a) Identify the labelled parts A, B, C and D. (c) SA node → AV node → Bundle of His → Right and
left branches of bundle of His → Purkinje fibre
(b) Mention the partial pressure of O2 and CO2 at A and B. (d) AV node → SA node → Bundle of His → Right and
(c) Which processes are represented by 'X' and 'Y'? Explain. left branches of bundle of His → Purkinje fibre
20. Refer to the given graph and answer the following 3. Select the incorrectly matched pair.
questions. (a) Leukopenia - Fall in WBCs count
(b) Polycythemia - Abnormal rise in RBCs count
(c) Thrombocytosis - Decrease in number of platelets
(d) Erythrocytopenia - Decrease in number of RBCs
4. Which of the following statements is incorrect?
Inspiratory capacity

A E (a) The fibrous cords attached to the flaps of the bicuspid


and tricuspid valves are chordae tendinae.
(b) The opening of inferior vena cava is guarded by
Total lung capacity

Eustachian valve.
(c) The right atrio-ventricular opening is guarded by the
B mitral valve
(d) Impulses received from carotid sinuses decreases
C heart rate whereas impulses received from vena cava
Functional residual

increases heart rate.


capacitiy

5. Which of the following cells secretes histamine and


D heparin?
(a) Basophils (b) Eosinophils
(c) Acidophils (d) Neutrophils
Lung volumes Lung capacities
6. When the ECG of a person was analysed, the Q and R
(a) Identify A, B, C and D. waves was found to be enlarged. It could be due to
(b) Define D mentioned in the above figure and state its (a) myocardial infarction (b) hypertension
volume. (c) cardiac arrest (d) bradycardia.

34 MT BIOLOGY TODAY | AUGUST ‘17


7. Which of the following statements is correct for cardiac cycle? C. AV node (iii) Pacemaker
(a) The ventricular systole is stimulated by the SA node. D. Parasympathetic nerve (iv) Decreases cardiac output
(b) The first heart sound coincides with the closure of (v) Pacesetter
semilunar valves and relaxation of ventricles. (vi) Decreases the rate of heart
(c) The blood flows from auricles to ventricles during the beat
atrial systole via bicuspid and tricuspid valves.
(vii) Generates action potential
(d) The fall in ventricular pressure causes the closure of
semilunar valve to prevent the back-flow of blood (viii) Increases the rate of heart
into ventricles. beat

8. Select important functions of lymph. Passage Based Question


(i) It absorbs and transports fat and fat soluble vitamins
from intestine. 13. Complete the given passage with appropriate words or
(ii) It regulates the pH of the body. phrases.
(iii) It transports nutrients, hormones, etc., to the body cell. Cardiac cycle is a regular sequence of three events namely
(iv) It destroys invading microorganisms. auricular systole, (i) and (ii) . The number of cardiac
(a) (i) only (b) (i), (iii) and (iv) cycles performed per minute are (iii) . and the duration
(c) (i) and (ii) only (d) (i), (ii), (iii) and (iv) of one cardiac cycle is (iv) . During a cardiac cycle, each
ventricle pumps out approximately 70 mL of blood which
9. If the heart rate of a person has increased but the stroke is called (v) . The amount of blood pumped by heart per
volume is same. The cardiac output will minute is called (vi) which is approximately (vii) in a
(a) increase (b) decrease healthy individual. In an (viii) it will be higher than that
(c) remains same (d) first increase then decrease. of an ordinary man.
10. Which of the following statements is correct? Assertion & Reason
(a) Peripheral circulation is the flow of deoxygenated
blood from the right ventricle to the lungs and return In each of the following questions, a statement of Assertion (A)
of oxygenated blood from the lungs to the left atrium. is given and a corresponding statement of Reason (R) is given
(b) Pulmonary circulation carries deoxygenated blood just below it. Of the statements, mark the correct answer as :
from pulmonary and bronchial arteries. (a) if both A and R are true and R is the correct explanation of A
(c) Hepatic portal system is a unique vascular connection (b) if both A and R are true but R is not the correct explanation of A
which exists between digestive tract and heart. (c) if A is true but R is false
(d) Coronary circulation is the flow of oxygenated blood (d) if both A and R are false.
from ascending aorta to heart muscle and return of 14. Assertion : Blood clot can be prevented in a test tube by
deoxygenated blood from heart muscle to the right adding a little oxalate.
atrium. Reason : Oxalate ions bind with Mg++ ions and prevent
them from forming prothrombinase.
Match The Columns 15. Assertion : Atherosclerosis occurs when there is fat
deposition in tunica interna of large arteries.
11. Match Column I with Column II.
Reason : Atherosclerosis is indicated by a flat T-wave in
Column I Column II
an electrocardiogram.
(Types of (Amount present in
leucocytes) total leucocytes) 16. Assertion : The pressure of blood in ascending aorta is
more as compared to that in descending aorta.
A. Lymphocytes (i) 2-3%
Reason : The diameter of lumen of ascending aorta is
B. Basophils (ii) 60-65% slightly less than that of descending aorta.
C. Eosinophils (iii) 20-25% 17. Assertion : Foramen ovale is the opening between two
D. Neutrophils (iv) 0.5-1% atria in fetus.
12. Match Column I with Column II (There can be more than Reason : Foramen ovale compensates for the underdevelo-
one match for items in Column I). ped pulmonary circulation.
Column I Column II 18. Assertion : Aortic semilunar and pulmonary semilunar
A. SA node (i) Transmits excitatory valves are open during joint diastole stage.
impulses to the ventricles Reason : The opening of these valves results in filling the
B. Sympathetic nerve (ii) Increases cardiac output right and left ventricles with blood.

MT BIOLOGY TODAY | AUGUST ‘17 35


Figure Based Questions (b) Identify enzyme C formed from A and B.
19. Study the given flow chart of blood clotting and answer (c) Thrombin acts as enzyme and brings depolymerisation
the following questions. of D. Identify D and state its significance.
Injured tissue Blood platelets 20. Refer to the given figure and answer the following question.
Releases Disintegrate
and release
A B X
Ca++, Proteins Y
Ca++, Proteins Z
C
A B
Prothrombin Thrombin catalyses
Ca++ (a) What do the figures A and B represent?
D Fibrin (b) Identify the labels X, Y, Z and mention their components.
(Fibrin + blood cells) Clot (c) State any two significant differences between A and B
(a) What does A and B represent? with respect to their contribution in blood circulation.

SOLUTIONS

CHAPTER-16 : DIGESTION AND ABSORPTION (iii) Paneth cells - secrete lysozyme and are capable of
1. (b) 2. (a) 3. (d) 4. (d) phagocytosis.
20. (a) A is parotid gland, B is sublingual gland and C is
5. (c) 6. (c) 7. (d) 8. (a)
submandibular gland. The parotid glands secrete
9. (d) 10. (c) much of salivary amylase or -amylase (= ptyalin).
11. A - (iii); B-(i); C-(iv); D-(ii) Sub-lingual and sub mandibular glands secrete salivary
amylase and mucus. The fluids secreted by the salivary
12. A - (ii), (x); B-(i), (iii); C- (v), (vii); D-(iv), (vi), E-(viii), (ix) glands constitute saliva.
13. (i) buccal cavity (ii) salivary amylase (b) P - Stenson's duct
(iii) acidic (iv) HCl Q - Ducts of Rivinus
(v) mucoid fluid (vi) secretin R - Wharton's duct
(vii) -amylase (viii) -dextrins CHAPTER-17 : BREATHING AND EXCHANGE OF GASES
(ix) monosaccharides (x) jejunum 1. (d) 2. (b) 3. (c) 4. (b)
14. (a) 15. (c) 16. (d) 17. (c) 5. (c) 6. (a) 7. (c) 8. (c)
18. (b) 9. (c) 10. (d)
19. (a) A is villi, B is lacteal, C are capillaries and D is crypt of 11. A - (iii), B-(i), C-(v), D-(ii), E-(iv)
Lieberkuhn. 12. A - (ii), (vii), B-(iv), (ix), C-(i), (iii); D-(vi), (x); E- (v), (viii)
(b) The innermost lining of alimentary canal mucosa have 13. (i) external respiration (ii) respiratory membrane
small finger like foldings called villi. These are covered
(iii) epithelial basement (iv) capillary basement
by a layer of columnar cells which have a 'brush
membrane membrane
border' appearance due to presence of microvilli.
These modifications increase the absorptive surface (v) capillary endothelium (vi) 0.2
of intestine. (vii) diffusing capacity (viii) solubility
(c) Following cells are present in crypts of Lieberkuhn- (ix) oxygen (x) blood
(i) Goblet cells - secrete mucus that protects the duodenal 14. (a) 15. (b) 16. (c) 17. (a)
wall from getting digested. 18. (d)
(ii) Argentaffin cells - synthesise secretin hormone and 19. (a) A - Pulmonary artery B - Pulmonary vein
5-hydroxytryptamine (5-HT). C - Systemic veins D - Systemic arteries

36 MT BIOLOGY TODAY | AUGUST ‘17


(b) The partial pressure of O2 and CO2 19. (a) A - Thromboplastin B - Platelet factor - 3
At A pO2 = 40mm Hg (b) Enzyme prothrombinase is formed by the combination
pCO2 = 45mm Hg of factor A and B with Ca2+ ions.
At B pO2 = 95mm Hg (c) D is fibrinogen. Fibrinogen is a plasma protein that
undergoes polymerisation to form insoluble polymer
pCO2 = 40mm Hg. called fibrin that results in clot formation.
(c) X represents the exchange of gases between lung 20. (a) Figure A represents T.S. of artery while figure B-
alveoli and pulmonary capillaries which is called represents T.S. of vein.
external respiration. Here the gases diffuse from a
higher to lower concentration, thus oxygen enters (b) In the given figures,
blood from alveoli and CO2 diffuses into alveoli from X is tunica adventitia or tunica externa, made up of
blood. Y represents the exchange of gases between connective tissue, Y is tunica media, made up of elastic
blood capillaries and tissue cells, known as internal connective tissue and smooth muscle fibres and Z
respiration. Here also, due to high partial pressure, is tunica interna, formed of elastic tissue of yellow
oxygen enters tissue cells from the capillaries and CO2 fibres, i.e., elastic membrane and flattened squamous
diffuses out from cells into the capillaries via tissue fluid. epithelial cells, i.e., endothelium.
20. (a) A - Inspiratory reserve volume (IRV) (c) The two significant differences between A (artery) and
B - Tidal volume (TV) B (vein) are :
C - Expiratory reserve volume (ERV) (i) Arteries carry oxygenated blood (except pulmonary
D - Residual volume (RV) artery) and distribute blood away from heart to
different parts of body, while veins carry deoxygenated
(b) Residual volume is the volume of air which still remains blood (except pulmonary vein) and collect blood from
in the lungs, after the most forceful expiration. Its different parts of body into the heart.
volume is about 1100 ml to 1200 ml.
(ii) The flow of blood is fast and under great pressure
(c) E is vital capacity. It is defined as the maximum volume in arteries, therefore, they possess thick and muscular
of air a person can breathe in after a forced expiration walls. On the other hand the flow of blood is not fast
or maximum volume of air a person can breathe out and under low pressure, therefore, veins have thin, non
after forced inspiration. muscular walls. They also possess valves to prevent
It can be calculated as ⇒ VC = TV + IRV + ERV. backward flow of blood.

(Tidal Volume + Inspiratory Reserve Volume +
Expiratory Reserve Volume)
Its capacity varies from 3400 mL to 4800 mL.
Vital capacity is observed to be higher in athletes and
mountain dwellers.
CHAPTER-18 : BODY FLUIDS AND CIRCULATION
1. (b) 2. (c) 3. (c) 4. (c)
5. (a) 6. (a) 7. (c) 8. (b)
9. (a) 10. (d)
11. A - (iii), B-(iv), C-(i), D-(ii)
12. A - (iii), (vii); B- (ii), (viii); C- (i), (v); D- (iv), (vi)
13. (i) ventricular systole (ii) joint diastole
(iii) 72 (iv) 0.8 seconds
(v) stroke volume (vi) cardiac output
(vii) 5 litres (viii) athlete
14. (c) 15. (b) 16. (c) 17. (a)
18. (d)

MT BIOLOGY TODAY | AUGUST ‘17 37


H GH
YIELD
FACTS Class XI

Biomolecules-II
• In the previous issue, we have discussed most of the macromolecules comprising
the cellular pool i.e., carbohydrates, lipids, proteins, nucleic acids, etc.
• The remaining important biomolecules such as enzymes are discussed here in

2017
2


continuation.

ENZYMES
• Enzymes are usually the most remarkable and highly specialised proteins having 2016
6


an extraordinary catalytic power and high degree of specificity for their substrates.
• Enzymes are core to every biochemical process as they catalyse the stepwise reactions
Analysis of various PMTs from 2013-2017

involving degradation of nutrient molecules, conservation and transformation of


chemical energy as well as synthesis of biological macromolecules from simple
precursors.
2015
2

2
• Since enzymes are capable of catalysing reactions of biological origin and are
synthesised by living cells only they are called biocatalysts.
• The term ‘enzyme’ was first coined by Kuhne (1877) for catalytically active substances.
• However the enzymes were first discovered and isolated by Buchner (1897). For
2014

10
2

this he was awarded Nobel Prize in 1903.


• Depending on their functioning inside or outside the cells, enzymes may be
Functional outside living cells, e.g., enzymes of digestive juices,
lysozymes of tears etc.
– Functional inside living cells, e.g., enzymes of Krebs’ cycle (inside
2013
4

mitochondria) and glycolysis (in cytoplasm).


Chemical Nature of Enzymes
• With the exception of a small group of catalytic RNA molecules, e.g., ribozymes and
AIPMT/NEET

ribonuclease P, all enzymes are globular proteins.


• Some enzymes may also contain non-proteinaceous molecules and metal ions that
Kerala
AIIMS

K-CET

J&K
AMU

participate directly in substrate binding or catalysis.

38 MT BIOLOGY TODAY | AUGUST ‘17


• Based on presence or absence of non-protein group, enzymes are of two types:
Enzymes


Apoenzyme (Protein part)
• Made up of protein only. • Made up of two parts:
• Active site comprises of specific grouping of its own Co-factor (Non-Protein part)
amino acids. • The complete enzyme is called holoenzyme.
• E.g., pepsin, trypsin etc. • The active site is formed jointly by apoenzyme and co-factor.

• Co-factor is small, heat stable and dialysable part of conjugate enzyme which may be organic or inorganic in nature.
• Organic co-factors are of two types: Co-enzymes and .
• are easily separable non-protein organic co-factors that act as transient carriers of specific functional groups.
Co-enzymes require two apoenzymes one for picking up the group and second for transferring the group e.g., NAD+,
NADP+, CoA.
• Most of the co-enzymes are made up of water soluble vitamins B and C, e.g., thiamine, riboflavin, nicotinamide, pyridoxine, etc.
• are non protein organic co-factors firmly attached to apoenzymes and take part in group transfer
reactions, e.g., heme, biotin, pyridoxal phosphate.
• include ions of a variety of minerals, e.g., calcium, iron, copper, zinc, nickel, molybdenum etc. They
usually function as activators by forming one or more coordination bonds with both the substrate and active site of enzyme.

Apoenzyme Coenzyme
(i) It is protein part of the holoenzyme or conjugate Coenzyme is the nonprotein organic group which gets attached
enzyme. to the apoenzyme to form holoenzyme or conjugate enzyme.
(ii) It is large in size. It is small in size.
(iii) An apoenzyme is specific for an enzyme. A coenzyme can function as a cofactor for a number of enzymes
carrying out that particular type of reaction.
(iv) It takes part only in catalytic activity of the enzyme. It helps in removing a product of chemical reaction besides
bringing contact between the substrate and the enzyme.
(v) Apoenzyme does not help in group transfer. Coenzyme takes part in group transfer.
(vi) Apoenzyme is thermolabile. Coenzyme is heat stable.
• of enzyme is the small portion of enzyme that takes part in catalysing biochemical reaction by attracting and
holding particular substrate molecule by its specific charge, size or shape.
• Active site consists of a few amino acids and their side groups which are brought together in a particular fashion due to
secondary and tertiary folding of a protein molecule and its association with the cofactor, if any. An enzyme may have more
than one active site.
• The molecule or biochemical acted upon by an enzyme is known as substrate and if more than one biochemical are involved
in a reaction, they are called reactants. The chemicals formed after the completion of reaction are called products.

Coenzyme
transferred
Biocytin CO2 Biotin
Co-enzyme A Acyl groups Pantothenic acid and other compounds
5 - Deoxyadenosylcobalamin (coenzyme B12) H atoms and acyl groups Vitamin B12
Flavin adenine dinucleotide Electrons Riboflavin (vitamin B2)
Lipoate Electrons and acyl groups Not required in diet

MT BIOLOGY TODAY | AUGUST ‘17 39


Nicotinamide adenine dinucleotide Hydride ion ( • •H–) Nicotinic acid (niacin)
Pyridoxal phosphate Amino groups Pyridoxine (vitamin B6)
Tetrahydrofolate One-carbon groups Folate
Thiamine pyrophosphate Aldehydes Thiamine (vitamin B1)


Ions Enzymes
Cu2+ Cytochrome oxidase
Fe2+ or Fe3+ Cytochrome oxidase, catalase, peroxidase
K+ Pyruvate kinase
“I bind with enzyme at a site
Mg2+ Hexokinase, glucose 6-phosphatase, pyruvate kinase other than its active site thereby
Mn2+ Arginase, ribonucleotide reductase distorting its conformation
which prevents its catalytic
Mo Dinitrogenase activity.”
Ni2+ Urease
Se Glutathione peroxidase
Zn2+ Carbonic anhydrase, alcohol dehydrogenase,
carboxypeptidases A and B

Classification of Enzymes
• According to modern system of enzyme classification, introduced by
in 1961, enzymes are classified into six classes, based on the type of reaction catalysed.

Oxidoreductases Lyases
These enzymes catalyse the oxidation-reduction reactions by These enzymes catalyse the breakdown of specific covalent bonds
transfer of electrons. These enzymes are of three types - Oxidases, and removal of groups without hydrolysis producing double bonds or
e.g., cytochrome oxidase, dehydrogenases, e.g., succinate removal of double bonds by adding groups.
dehydrogenase and e.g., nitrate reductase. Aldolase
1 Cytochrome Fructose 1,6-diphosphate
NADH + H+ + O2 NAD++ H2O 3-phosphoglyceraldehyde + Dihydroxyacetone phosphate
2 oxidase

Isomerases
Transferases These enzymes catalyse the change of a substrate into a related
These enzymes catalyse the transfer of specific group other than isomeric form by rearrangement of molecules.
hydrogen from one substrate to another. The group transfer does not These enzymes too are of three types: Isomerases (change aldose
occur in free state. to ketose group or vice-versa), Epimerases (change in position of
Glutamic acid + Oxaloacetic acid Transferase -ketoglutaric acid + Aspartic acid one constituent or carbon group), (shift the position of side
(amino acid) (organic acid) (organic acid) (amino acid) group like Glucose-6-phosphate → Glucose-1-phosphate).
Phosphohexose
Glucose 6-phosphate Fructose 6-phosphate
isomerase
Hydrolases
These enzymes catalyse splitting of larger molecules into smaller
ones by addition of water (hydrolysis of bonds), i.e., with the help of Ligases
hydrogen and hydroxyl groups. Usually bonds formed by dehydration These enzymes catalyse the joining of two substrate molecules by
condensation, i.e., ester, ether, glycosidic are catalysed by these utilising the energy from hydrolysis of ATP.
enzymes, e.g., amylase, sucrase, lactase.
Pyruvate
Amylase Pyruvate + CO2 + ATP+ H2O Oxaloacetate + ADP + Pi
Starch Maltose + Isomaltose + -Dextrins carboxylase

40 MT BIOLOGY TODAY | AUGUST ‘17


Characteristics of Enzymes

Enzymes are usually globular proteins except two RNA enzymes. Enzymes are hydrophillic and form hydrosol in free state.

Efficiency Reversibility
The efficiency of enzyme depends upon the number of active sites present Almost all enzymatic reactions are reversible. However, reversibility
over an enzyme. The number of substrate molecules changed per minute is dependent upon energy requirements, availability of reactants,
by an enzyme is called . The higher the turn over concentration of end products and pH.
number, the more efficient an enzyme is.
Thermolabile
Enzymes are heat sensitive or thermolabile and operate between
Enzymes are highly specific in their action. Different enzymes act on same 25-35°C (optimum temperature range). They become inactive at
substrate but give rise to different products. Similarly, an enzyme may act freezing temperatures and denature at 50-55°C.
on different substrates to produce different end products.

The active sites of enzymes have crevices or conformations that fit in
Enzymatic reactions are not isolated, they occur in quick succession or a the substrates in a complementary fashion forming a complex known
team of enzymes (complex) work together to perform multistep reactions. as enzyme substrate complex. The latter are changed into products.
As soon as the products are released, the active sites become free to
perform another catalytic act.

MECHANISM OF ENZYME ACTION


• The mechanism of enzyme action can be studied under two steps :

Mechanism of Enzyme Action

Most of the chemical reactions have an energy barrier separating the reactants and
products. Non
Therefore an external supply of energy is needed for the start of chemical reaction.
This energy is called which is quite high for the non-catalysed
reactions and is lowered by the enzyme. The activation energy increases the kinetic
energy of the system and brings about forceful collisions between the reactants.
Enzyme lowers activation energy of a reaction by : taking the reactants out of solution (Reactant)
state (desolvation), establishing weak bonds between reactants and enzymes,
(Product)
bringing reactant molecules, close to one another in the region of active sites and
developing strain in reactants bonds by electrophillic and nucleophillic attack.

All enzymes have a specific three-dimensional structure and a part of it is known as into which substrate fits to proceed chemical
reactions. The point where substrate is bound on the active site is known as the The first step in an enzymatic
reaction is that the enzyme forms a temporary association with its substrate called (E – S complex). Following
two models have been proposed to explain the formation of E – S complex.


(1894) proposed that as a particular lock can be opened by Koshland (1959) proposed that the active sites of an
enzyme are not rigid. When the substrate binds to an
only a particular key, similarly a substrate of specific shape fits into a specific
enzyme, it may induce conformational change in enzyme
active site on the enzyme surface. This suggests that both enzyme and
molecule in such a way that it is fit for the substrate-
substrate molecules have specific geometrical shapes and form an enzyme
enzyme interaction.
substrate (ES) complex. This also explains the of enzyme action.
Active site
Active site
Substrate Reactive site

+
Products
Enzyme Enzyme-substrate
Pr oducts complex (Enzyme
(Larger than substrate)
Enzyme Enzyme substrate has changed its shape
complex to accommodate the
substrate)

MT BIOLOGY TODAY | AUGUST ‘17 41




(i) Active site is a single entity. Active site is made of two components.
(ii) There is no separate catalytic group. A separate catalytic group is visualised.
(iii) Active site is static. In contact with substratum, the buttressing group undergoes
conformational change.
(iv) Development of transition state is not considered. It considers the development of transition state before the
reactants undergo change.
(v) It does not visualise the weakening of substrate Catalytic group is believed to weaken the substrate bonds by
bonds. nucleophilic and electrophilic attack.
(vi) It does not explain the mechanism of non-activity in It gives a mechanism for non-action over competitive inhibitor.
case of competitive inhibitor.

Temperature
An enzyme is active within a narrow range of

Reaction velocity
temperature.The temperature at which an enzyme The rate of enzymatic reaction
Reaction velocity

shows its highest activity is called increases with the increase in Limiting
temperature. The optimum temperature for enzyme concentration upto effect
most of the enzymes is between 25°C - 35°C. Optimum
a point called
Temperature above and below this range point. Above this limit, there
affects the enzyme activity. High temperature Temperature is little effect on enzyme Enzyme concentration
above 50°C destroy enzymes by causing their Fig .: Relation between temperature and
denaturation and very low temperature preserves enzyme controlled reaction velocity activity. Fig.: Effect of enzyme concentration
the enzymes in the inactive state. on the rate of biochemical reaction

Factors Affecting Enzyme Activity

pH
Enzymes work at their own optimum pH. A Initially the rate of enzymatic reaction increases with Some chemical substances or molecules
rise or fall in pH reduces enzyme activity by the increase in substrate concentration. In beginning, increase activity of enzymes such as
changing the degree of ionisation of its side the velocity of reaction is high but later it does not co-factors, e.g., K+, Mn2+ etc. On the other
chains. Most of the intracellular enzymes increase progressively with the increase in substrate hand, salts of heavy metals and compounds
function near neutral pH with the exception concentration. It happens because enzyme molecules such as cyanides, azides, iodoacetate
of several digestive enzymes that are active get fully saturated and no active site is left free to bind destroy tertiary structure of enzymes thus
either at acidic or alkaline pH range. additional substrate. affecting enzyme activty.

Michaelis-Menten Equation
• Michaelis-Menten equation mathematically illustrates the relationship between
initial reaction velocity and substrate concentration. Maximum rate
Reaction Velocity (V)

• Here, the Km (Michaelis Menten constant) is a mathematical derivative which Vmax


indicates the substrate concentration at which the chemical reaction catalysed Reaction with
enzyme
by an enzyme attains half its maximum velocity.
1/2Vmax
• Km indicates affinity of the enzyme for its substrate. A high Km indicates low
affinity while a low Km indicates strong affinity. Reaction
• The Km value generally lies between 10–1 to 10–6 M. without enzyme
• Km, the Michaelis-Menten constant (or Brig’s and Haldane’s constant), is given Km
k2 +k3 Concentration of substrate (S)
by the formula Km = , here k1, k2 and k3 represent the velocity constants Fig .: Effect of substrate concentration on
k1
reaction velocity with or without enzyme.
for the respective reactions as shown below.

42 MT BIOLOGY TODAY | AUGUST ‘17


k k
E + S 
← 
1
→ ES 
3
→ E + P
(Enzyme) (Substrate) k
2 (Enzyme-substrate complex) (Enzyme) (Product)

Vmax [S]
The Michaelis-Menten equation obtained after suitable algebraic manipulation is V= where V = Measured velocity,
Km +[S]
Vmax = Maximum velocity, S = Substrate concentration, Km = Michaelis-Menten constant.

INHIBITION OF ENZYME ACTION


• The reduction or cessation of enzyme activity due to presence of certain adverse conditions or chemicals is called enzyme
inhibition.

On the basis of nature of inhibition


• Temporary inhibition that can be overcome by withdrawal of inhibition. • Permanent inhibition of enzyme due to the change
• It occurs due to blocking of active site or binding to linkages required or destruction in conformation of enzyme.
for maintenance of active site. • It occurs due to covalent binding of heavy metals
• It is characterised by rapid dissociation of enzyme inhibitor complex. or other inhibitors with the enzymes, resulting in
• Dilution and dialysis reduces or eliminates the effect of reversible inhibition. their inactivation.

On the basis of causes of inhibition



• The inhibitor (I) closely resembles the real substrate (S), and is regarded as S
S ES complex
a . The inhibitor competes with substrate and binds
at the active site of the enzyme, but does not undergo any catalysis. As
long as the competitive inhibitor holds the active site, the enzyme is not
available for the substrate to bind. I I EI complex
• Competitive inhibition is usually reversible.
• The classical example of competitive inhibition is enzyme
with succinic acid as its substrate. Malonate, oxaloacetate are its inhibitor which resemble succinic acid.


• The inhibitor binds at a site other than the active site on the enzyme
S
surface. This binding impairs the enzyme function. The inhibitor has no E+S ES E+P
structural resemblance with the substrate. It does not interfere with the + + S

enzyme-substrate binding but the catalysis is prevented, possibly due to I I


a distortion in the enzyme conformation. I
• Non-competitive inhibition is usually irreversible because it cannot be
EI + S EIS
overcome by increasing substrate concentration.
I
• The inhibitor (I) generally binds with the enzyme as well as the ES complex.
• Common example of non-competitive inhibition is inhibition of respiratory S

enzyme cytochrome oxidase by cyanide that combines with its metallic


ions. It has no structural similarity with the substrate of the enzyme cytochrome c.


• A rarely encountered inhibition called uncompetitive inhibition is also known where inhibitor binds only to the ES complex.
• Uncompetitive inhibition cannot be overcome as the ESI complex cannot form product.


• It is a type of reversible inhibition found in allosteric enzymes. The inhibitor is non-competitive
and is usually a low molecular weight intermediate or product of a metabolic pathway
having a chain of reactions involving a number of enzymes. It is, therefore, also called end
product or .
• The inhibitor is also called modulator. Modulator is a substance that attaches with an
allosteric enzyme at a site other than catalytic one but influences the latter, either inhibiting
or activating the same.
• An example of feed back or allosteric inhibition is stoppage of activity of enzyme hexokinase
(glucokinase) by glucose-6-phosphate, the product of reaction catalysed by it.

MT BIOLOGY TODAY | AUGUST ‘17 43




(i) The inhibitor binds to the active site of enzyme. The inhibitor attaches to an area other than the active site.
(ii) It does not change conformation of enzymes. Conformation of enzyme is changed.
(iii) The active site is swamped by inhibitor. Conformation of active site is changed so that substrate cannot
combine with it.
(iv) The inhibitor resembles the substrate in its broad The inhibitor has no structural similarity with the substrate.
structure.
(v) The inhibitor is not connected by metabolic pathway Inhibitor is a product or intermediate of the metabolic pathway
catalysed by the enzyme. connected with that enzyme.
(vi) It does not have a regulatory function. It has a regulatory function as it stops the excess formation of
a product.
Significance of Enzymes
Biological Importance

Enzymes are highly regulated by activation and repression Operate under favourable conditions, enabling Mediates thousands of chemical
of genes, allowing some enzymes to be functional or non the organism to live under favourable reactions occurring inside living
functional as per requirement. environment only. cells.

Economic Importance


Enzyme complex zymase obtained from yeast is used in brewing or Several enzymes such as diastase are prescribed to patients with
fermentation of alcoholic drinks. deficient digestive juices.
Detergents Cheese
Amylases are used as detergents for machine, dishwashing to remove Enzymes such as rennet and lactase are employed in preparation of
resistant starch residues. Lipases are used to assist in removal of fatty cheese and to impart consistency and flavour to it.
and oily stains.
Diagnostics
ELISA (Enzyme linked immunosorbent assay) is an enzyme based technology
Enzyme trypsin is used to partially predigest baby foods. used for detection of diseases such as AIDS.

METABOLITES
• Metabolites are biomolecules or organic compounds that are constantly utilised (catabolism) or formed (anabolism) in various
metabolic reactions occurring inside the cells.
• Plants and microbes produce several organic compounds that may be required for their basic metabolic process while at the
same time they also synthesise compounds that do not participate in metabolic pathways.
• On this basis, metabolites are of two types :

Types of Metabolites

Some of the organic compounds like carbohydrates, fats, proteins, nucleic acids, etc., are required for basic metabolic processes and are
directly involved in normal growth, development and reproduction. These organic compounds are called primary metabolites.


Many plants, fungi and microbes of certain genera synthesise a number of organic compounds which are not involved in primary metabolism
(photosynthesis, respiration, protein and lipid metabolism) and seem to have no direct function in growth and development of plants. These
compounds are called secondary metabolites e.g., gum, resin, rubber etc.


e.g., rubber, steroids, carotenoids. e.g., alkaloids, glycosides, glucosinolates e.g., tannins, flavonoids, lignin etc.

44 MT BIOLOGY TODAY | AUGUST ‘17


Some of them attract animals for
pollination and seed dispersal. (i) Pigments Carotenoids, anthocyanins, etc.
(ii) Alkaloids Morphine, codeine, etc.
They act as agents of plant-plant
competition. (iii) Terpenoides Monoterpenes, diterpenes etc.

(iv) Essential oils Lemon grass oil, etc.


They help plants in defense against (v) Toxins Abrin, ricin
herbivores and pathogens.
(vi) Lectins Concanavalin A

They are also used in making drugs, (vii) Drugs Vinblastin, curcumin, etc.
flavours, gums, resins, insecticides etc. (viii) Polymeric substances Rubber, gums, cellulose



(i) These are biomolecules required for basic metabolic These are derivatives of primary metabolites which are not
processes. involved in bast.
(ii) These are produced in generous quantities and can easily be These are produced in small quantities and their extraction from
extracted from the plant. the plant is difficult.
(iii) These are found throughout the plant kingdom. Particular secondary metabolites are found in one plant species
or families and not all.
(iv) These are part of the basic molecular structure of the cell. These are not part of the basic molecular structure of the cell.
(v) They are highly useful to plant. They have limited role in plant.
(vi) They are found from the beginning of plant life. They are found at particular stages of development.

Study the given table and identify A, B, C and D. Match column I with column II and select the correct
option from the given codes.


A Transfer of electrons 2+
A. Catalase (i) Zn
Transferases B
B. Salivary amylase (ii) Cu2+
C Transfer of functional group to water C. Carboxypeptidase (iii) Fe2+
Isomerases D D. Cytochrome oxidase (iv) Cl–
(a) A-(iv), B-(iii), C-(ii), D-(i)
A B D
(b) A-(iii), B-(iv), C-(i), D-(ii)
(a) Ligases Transfer of Oxido- Transfer of
hydride ion reductases H atom (c) A-(iii), B-(ii), C-(i), D-(iv)
(d) A-(i), B-(ii), C-(iii), D-(iv)
(b) Hydrolases Hydrolysis Lyases Transfer of
reaction ion A low Km indicates
(c) Oxido- Group Hydrolases Intramolec (a) low affinity of the enzyme for its substrate
reductases transfer -lar rearra- (b) high affinity of the enzyme for its substrate
reactions ngement (c) substrate concentration decreases at which
(d) Lyases Transfer of Ligases Transfer of chemical reaction attains maximal velocity
H atom electron (d) negligible affinity of the enzyme for its substrate.

MT BIOLOGY TODAY | AUGUST ‘17 45


Fumarase catalyses conversion of fumarate to malate at Read the following statements and fill up the blanks with
pH 6.2. What will be the effect on this reaction if the pH correct options.
is increased to 7.5? (i) The complete conjugate enzyme consists of a
(a) Krebs’ cycle will stop. protein portion called ________.
(b) Fumarate will change directly to oxaloacetate. (ii) ________ is a non-protein cofactor which is
(c) Fumarate will convert to succinate. loosely bound to an apoenzyme.
(d) Malate will convert to fumarate. (iii) ________ enzyme is used for removing blood
Refer to the given list of secondary metabolites and select clots inside blood vessels.
the correct option.
(i) Concanavalin A (ii) Vinblastin (a) holoenzyme Prosthetic group Trypsin
(iii) Morphine (iv) Ricin (b) coenzyme Apoenzyme Diastase
(v) Carotenoid (vi) Anthocyanin (c) apoenzyme Coenzyme Streptokinase
(vii) Codeine (viii) Abrin (d) apoenzyme Holoenzyme Trypsin
(ix) Curcumin Read the following statements and select the incorrect
ones.
(a) (v), (vi) (iii), (vii) (iv), (viii) (i) (ii), (ix) I. Proenzyme is the active precursor of an enzyme.
(b) (i), (vi) (ii), (v) (vii) (iii), (ix) (iv), (viii) II. Enzymes are mostly produced in the zymogen
(c) (v), (vi) (iii), (vii) (i), (viii) (iv), (ii) (ix) state.
(d) (iii), (vi) (v), (i) (vii), (viii) (iv), (ii) (ix) III. The enzymes become reactive only at their
Consider the following statements and select the correct optimum temperature.
option. IV. An enzyme catalyses specific reaction of a single
I. All the enzymes are proteinaceous in nature. or only a few substrates.
II. The enzymatic reactions may be reversible or (a) I and II (b) III and IV
irreversible. (c) I and III (d) I, II and III
III. Enzymes denature at both low and high
Which of the following is not a function of secondary
temperatures.
metabolite?
IV. All enzymes are active at optimum pH of
(a) Acts as agents of competition between plants
7.0.
(b) Helps in defense against herbivores
(a) I and III (b) II only
(c) Used in degumming of silk
(c) IV only (d) II and IV
(d) Attract animals for pollination and seed dispersal
In induced fit theory,
(a) the active site is a single entity An enzyme such as alcohol dehydrogenase occurs in
(b) there is no separate catalytic group multiple molecular forms with similar substrate activity.
(c) the active site is static Such enzymes are called
(d) the development of transition state before the (a) allosteric enzymes (b) isozymes
reactants changes. (c) holoenzymes (d) proenzyme.
Di-isopropyl fluorophosphate (DFP), is a nerve gas that Read the following statements regarding enzyme
prevents impulse transfer by combining with amino acid inhibition and select the incorrect ones.
serine of acetylcholine esterase. It is an example of I. In allosteric inhibition, the inhibitor attaches to
(a) competitive inhibition active site.
(b) non-competitive inhibition II. Competitive inhibition changes the conformation
(c) protein denaturation of enzyme.
(d) allosteric inhibition. III. In competitive inhibition, the product of the
During an enzymatic reaction, the activation energy metabolic pathway itself acts as inhibitor.
(a) decreases to increase the kinetic energy of system IV. Allosteric inhibition has a regulatory function as
(b) increases to decrease the kinetic energy of system it stops the excess formation of a product.
(c) required is higher in case of exothermic reactions (a) I, II and III (b) I and IV
(d) all of these. (c) III and IV (d) II, III and IV

48 MT BIOLOGY TODAY | AUGUST ‘17


Michaelis Menten constant is a mathematical derivative binding of various modulators that may alter the
that indicates the conformation of active site.
(a) concentration of products formed at the end of (c) Allosteric activator binds to the active site so as
enzymatic reaction to make it active in absence of substrate.
(b) substrate concentration at which the reaction (d) Lactate dehydrogenase is an example of allosteric
catalysed by an enzyme attains half of its enzyme.
maximum velocity The role of a coenzyme during enzymatic reaction is to
(c) substrate concentration at which the reaction (a) pick up product of the reaction
catalysed by an enzyme attains its maximum (b) transfer the product picked up to another reactant
velocity (c) bring substrate in contact with the enzyme
(d) rate of increase in velocity of enzyme reaction due (d) all of these.
to increase in substrate concentration. Select the correctly matched pair.
Select the incorrectly matched pair for the type of enzyme (a) Amylase - Preparation of cheese
inhibition and their respective inhibitors involved. (b) Protease - Added in baby foods
(a) Competitive inhibition - Sulpha drugs (c) Pectinase - Preparation of green coffee
(b) Non-competitive inhibition - Cyanide (d) Lipase - Softening of meat
(c) Allosteric inhibition - Iodoacetamide The turn over number of an enzyme refers to
(d) Irreversible inhibition - Iodoacetic acid (a) number of substrate molecules changed per
Select the correct statement regarding allosteric enzymes. minute by an enzyme
(a) These are molecular variants of a single enzyme (b) the efficiency of enzyme in terms of number of
showing same active site for binding of various active sites present over it
substrates. (c) increase in rate of reaction
(b) These are enzymes having separate sites for (d) both (a) and (b).


Humans are bringing about the sixth mass extinction of life on Earth, according to scientist writing in a special edition of the leading journal Nature.
Mammals, birds and amphibians are currently becoming extinct at rates comparable to the previous five mass extinctions that occurred across the past
500 million years when “cataclysmic forces”, such as massive meteorite strikes and super volcano explosions, wiped out vast swathes of life, including
the dinosaurs.
The growing human population which has increased by 130 percent in the last 50 years and is set to rise to more than 10 billion by 2060 and our
increasing demand for resources as we become wealthier, ramping up the pressure on the natural world. Tens of thousands of species, including 25
percent of all mammals and 13 percent of birds, are now threatened with extinction because of over-hunting, poaching, pollution, loss of habitat, the
arrival of invasive species and other human-caused problems.
But the researchers said it was not “inevitable” that this process would continue. There is still time for humans to turn the situation around by protecting
habitats, changing our diets to less land-intensive food and taking other forms of conservation.
In one of a series of papers in Nature, a team of international scientists wrote:
“There is overwhelming evidence that habitat loss and fragmentation, over-exploitation of biological resources, pollution, species invasions and climate
change have increased rates of global species extinctions to levels that are much higher than those observed in the fossil records.” This loss of
biodiversity could “substantially diminish the benefits that people derive from nature”, they warned.
In order to preserve such “ecosystem services”, policies should be designed to “secure the valuable and often irreplaceable benefits of biodiversity for
future generations, even under conditions of rapid global change” the paper added.
Another paper painted a bleak picture of humans’ long history of wiping out other animals. Human-influenced extinctions began when modern humans
moved out of Africa it said. Successive waves of extinctions in Australia (50,000 years ago), North America and South America (10,000-11,000 years
ago) and Europe (3,000-12,000 years ago) were driven largely by a combination of hunting by humans and natural climate change. By 3,000 years
ago, Earth had lost half of all terrestrial mammalian megafauna species and 15 percent of all bird species.
The researchers said that since 1500 AD, human destruction of wildlife had accelerated. It said urgent action was needed to ensure that sufficient
habitats will remain to preserve the viability of species in the long term and to guarantee that such habitats are well managed.

Courtesy : The Times of India

MT BIOLOGY TODAY | AUGUST ‘17 49


-4 Practice Problems
Class XI

T his specially designed column enables students to self analyse their


extent of understanding of specified chapters. Give yourself four
marks for correct answer and deduct one mark for wrong answer.
Self check table given at the end will help you to check your
readiness.
• Structural Organisation in Animals
• Cell : The Unit of Life

Total Marks : 160 Duration : 40 Min.

1. The epithelium present in the descending limb of loop of 4. Bidder’s canal is found in
Henle is (a) kidney of mammal (b) testes of frog
(a) simple cuboidal epithelium (c) kidney of frog (d) ovary of mammal.
(b) simple columnar epithelium 5. In a sodium potassium exchange pump, when one ATP is
(c) simple ciliated epithelium
hydrolysed, the number of Na+ ions pumped outward and
(d) simple squamous epithelium.
K+ ions pumped inward, respectively are
2. Study the given diagram showing transverse section of (a) two, three (b) one, two
mammalian bone and select the incorrect option regarding (c) three, two (d) two, one.
P, Q, R and S.
6. Complete the following table by correctly identifying A, B
S
and C.
P
Connective Location Function
tissue
Q
Yellow elastic
Provides strength
(i) connective A
and elasticity
tissues
R Forms embryonic
(a) P forms an envelope around bone and contains Rings of
(ii) B skeleton in bony
osteoblasts which produce new bone material. trachea
vertebrates
(b) Q represents Haversian canals in bone matrix and are
White fibrous Intervertebral
interconnected by Volkmann’s canals. (iii) C
(c) R comprises of inactive bone cells called osteocytes cartilage discs
and white fibrous tissue.
(d) S represents canaliculi, which connects osteocyte with A B C
another. (a) Walls of blood Hyaline Acts as a cushion
3. Identify the tissue from the given functions. vessels cartilage and provides
(i) Collagen fibres are present in rows between many strength
parallel bundles of fibres. (b) Periosteum of Hyaline Provides articulation
(ii) Presence of this tissue at joints between skull bones bone cartilage
makes them immovable.
(c) Cornea of the Calcified Allows stretching of
(a) Loose connective tissue
eyeball cartilage pubic symphysis
(b) Dense regular connective tissue
(c) Dense irregular connective tissue (d) Nasal septum Calcified Provides flexibility to
(d) Hyaline cartilage cartilage organs

50 MT BIOLOGY TODAY | AUGUST ‘17


7. Select the incorrect statement with reference to cockroach. 14. Select the correct set of differences among the following
(a) Ecdysone secreted by prothoracic glands controls types of membrane transport.
moulting of body wall of the nymph. Pinocytosis Phagocytosis
(b) The atrial part of a spiracle possesses a valve and
bristly plate to filter out dust particles. (i) Bulk intake of fluid Intake of solid material
(c) The secretion of uricose glands form the outermost materials by a cell. from outside to the inside
layer of spermatophore. of the cell.
(d) Anal styles are present in the 9th sternum of male
(ii) Large vesicles are formed. Small vesicles are formed.
cockroach only.
8. Read the given statements and select the correct option. (iii) Receptor pits are present Receptor pits are absent.
Statement A : In the nymph of cockroach, neotinin is on membrane for receiving
secreted by a paired structure called corpora allata. the materials.
Statement B : Neotinin regulates the development and (a) (i) and (iii) (b) (ii) only
functioning of reproductive organs. (c) (i) and (ii) (d) (i), (ii) and (iii)
(a) Both statements A and B are correct and B is the
correct explanation of A. 15. Study the diagram of a mitochondrion given below and select
(b) Both statements A and B are correct but B is not the the option that correctly identifies the labelled parts A-E.
correct explanation of A. E
D
(c) Statement A is correct but statement B is incorrect. A
(d) Both statements A and B are incorrect.
9. Bone resorption is brought about by B
(a) osteocytes (b) osteoblasts
(c) osteoclasts (d) osteoprogenitor cells.
10. The ovaries in a female cockroach are located in the segments C
A B C D E
(a) 3rd to 5th (b) 2nd to 6th
th th (a) Oxysome 70S ribosome Matrix Crista Outer
(c) 6 to 8 (d) 4th to 8th. chamber
11. Identify the correct statements with reference to the (b) Vesicle 80S ribosome
Inner Crista Outer
ultrastructure of nucleus. chamber chamber
I. Perinuclear space separates the two membranes of the (c) Oxysome 70S ribosome Crista Inner Matrix
chamber
nuclear envelope.
(d) Vesicle 80S ribosome Crista Matrix Outer
II. Some proteins present in nucleoplasm are essential for chamber
spindle formation.
III. The lightly stained condensed fibrous part of chromatin
is called heterochromatin.
IV. Nucleolus is the main site for the development of WEST BENGAL at
ribosomal RNAs.
Progressive Book Centre - K haragpur
(a) I and II (b) II and IV
Ph: 03222-279956; Mob: 9932619526, 9434192998
(c) I, II and IV (d) III only International Book Trust - K olkata
12. The cell organelle which takes part in the storage and Ph: 033-22414947, 24792343; Mob: 9830360012
synthesis of lipids is Rukmani Agencies - K olkata Ph: 033-24666173, 224839473; Mob: 9830037811
Every Books - K olkata
(a) peroxisome (b) glyoxysome
Ph: 033-22418590, 22194699; Mob: 9830162977, 8599985926
(c) aleuroplast (d) sphaerosome.
K atha - K olkata Ph: 033-22196313; Mob: 9830257999
13. Cell membrane was discovered by (i) in year (ii) . Most recent Saraswati Book Store - K olkata Ph: 22198108, 22190784; Mob: 9831349473
fluid mosaic model of biomembrane was given by (iii) . Chhatra Pustak Bhawan - Medinipur Mob: 9609996099,9332341750
Novelty Books - Siliguri Ph: 0353-2525445; Mob: 7797938077
(i) (ii) (iii)
O m Traders - Siliguri Ph: 0353-6450299; Mob: 9434310035, 9749048104
(a) Plowe 1855 Danielli and Davson
(b) Schwann 1838 Singer and Nicolson
(c) Plowe 1831 David Robuston
(d) Nageli and Cramer 1844 Singer and Nicolson

MT BIOLOGY TODAY | AUGUST ‘17 51


16. Which of the following is not the similarity between 21. Softening of ripe fruits is caused due to partial solubilisation
mitochondria and chloroplast? of pectic compounds present in
(a) Both produce ATP. (a) primary cell wall (b) middle lamella
(b) Both possess double membrane envelope. (c) cell membrane (d) secondary cell wall.
(c) Both convert light energy into chemical energy.
22. Read the given statements and select the correct option.
(d) Both contain naked DNA.
Statement A : Lysosomes contain enzymes called acid
17. Match column I with column II and select the correct option
hydrolases in the form of crystalline or semi-crystalline
from the codes given below.
granules.
Column I Column II
A. Elaioplasts (i) Endosperm cells of castor Statement B : Digestive enzymes present in lysosomes
require a pH of 3-4 for proper functioning.
B. Aleuroplasts (ii) Plastids of red algae
(a) Both statements A and B are correct and B is the
C. Chromoplasts (iii) Synthesis of membrane lipid correct explanation of A.
D. Chloroplasts (iv) Colourless plastids (b) Both statements A and B are correct but B is not the
E. Rhodoplasts (v) Starch storage correct explanation of A.
(a) A-(iv), B-(i), C-(v), D-(iii), E-(ii) (c) Statement A is correct but statement B is incorrect.
(b) A-(ii), B-(iii),C-(v), D-(iv), E-(i) (d) Both statements A and B are incorrect.
(c) A-(iv), B-(i), C-(iii),D-(v), E-(ii)
23. Which of the following options is matched incorrectly
(d) A-(iii), B-(ii), C-(iv),D-(v), E-(i)
regarding the epithelium shown in the given figure?
18. Refer to the given table showing differences between Squamous layers
single-unit and multi-unit smooth muscles.
Single-unit smooth Multi-unit smooth Intermediate
layers
muscles muscles
Germinative
(i) Composed of muscle Composed of muscle layer
fibres not so closely fibres closely joined Basement
joined together. together. membrane
(a) It represents non-keratinised stratified squamous
(ii) All the fibres contract All the fibres contract
epithelium.
simultaneously as a independently as
(b) It lines the buccal cavity, vagina, cornea of eye, etc.
single unit. separate units.
(c) It provides well protection against abrasions and
(iii) Found in the walls of Found in the walls of the contains dead superficial cells.
gastrointestinal tract large blood vessels and (d) It is permeable to water.
and urinary bladder. iris muscles in the eye.
Which of the above cited differences is/are not correct?
(a) (i) only (b) (i) and (ii)
(c) (iii) only (d) (ii) and (iii)
19. Which of the following features does not belong to fluid 1. Make as many biological terms as possible
mosaic model of biomembrane system? using the given letters. Each word should
(a) -protein molecules are attached by electrostatic contain the letter given in circle.
2. Minimum 4 letter word should be made.
forces to phospholipid bilayer.
3. In making a word, a letter can be used as
(b) Proteins are held in their position by both polar and many times as it appears in the box.
non-polar side chains. 4. Make at least 1 seven letter word.
(c) Proteins provide the structural and functional specificity
to the membrane.
(d) Tunnel proteins are integral proteins that form channels D Z H Y P H A T M U
for passage of water.
B E O I S E R G L N C
20. The larger subunit of 70S ribosome is
(a) 60S (b) 50S Send your response at editor@mtg.in or post to us with complete address by 25th of
(c) 40S (d) 30S. every month to win exciting prizes. Winners’ name will be published in next issue.

52 MT BIOLOGY TODAY | AUGUST ‘17


24. Complete the following table by identifying X, Y and Z from Of the above statements
the given options. (a) II and III are correct (b) I and IV are correct
(c) I, II and IV are correct (d) I and III are correct.
Chromosome type Position of centromere
30. Match column I with column II and select the correct option.
(i) Acrocentric X
Column I Column II
(ii) Y Sub-median
A. Pseudo-stratified (i) Subcutaneous tissue
(iii) Telocentric Z epithelium around eye
X Y Z B. Adipose tissue (ii) Sternal parts of ribs
(a) Sub-terminal Sub-metacentric Terminal C. Hyaline cartilage (iii) Nephrons of kidneys
(b) Inner to telomere Metacentric Terminal D. Smooth muscle tissue (iv) Olfactory mucosa
(c) Sub-terminal Metacentric Median E. Ciliated cuboidal (v) Blood vessels
epithelium
(d) Terminal Sub-metacentric Sub-median
(a) A-(iv), B-(i), C-(ii), D-(v), E-(iii)
25. The component of the cell associated with synthesis of (b) A-(iii), B-(i), C-(ii), D-(v), E-(iv)
sterols and steroid hormones in the interstitial cells of testis (c) A-(i), B-(iii),C-(ii), D-(iv),E-(v)
and ovary is (d) A-(iv), B-(iii),C-(i), D-(ii), E-(v)
(a) smooth endoplasmic reticulum
(b) rough endoplasmic reticulum 31. Identify the correct option on the basis of type of flagellation
(c) Golgi vesicle in bacteria.
(d) mitochondrion. (a) Lophotrichous - Tuft of flagella at one end
(b) Cephalotrichous - Number of flagella distributed all
26. Select the mismatched pair from the given options. over the surface
(a) Cardiac muscle fibres - Branched fibres (c) Amphitrichous - Flagellum at each of the two ends
(b) Microfilaments - Actin (d) Atrichous - Single flagella present at one end
(c) Apocrine glands - Goblet cells
(d) Polytene chromosomes - Salivary chromosomes 32. Refer to the given figure of trachea and trancheoles in a
cockroach. Identify the parts labelled as A – D.
27. Nephrostome occurs in earthworm’s
(a) septal nephridia B C A
(b) integumentary nephridia
(c) pharyngeal and septal nephridia
(d) pharyngeal and integumentary nephridia.
28. Which option is correct with D
reference to the parts marked A
A, B and C in the given figure B A B C D
of section of flagellum? C
(a) Tracheoles Spiracle Atrial muscle Trachea
(a) A is a microtubule made of (b) Tracheae Atrium Tracheoles Atrial muscle
protein dynein. (c) Spiracles Tracheole Trachea Atrium
(b) A and B are interconnected (d) Atrial muscles Spiracle Tracheole cell Atrial muscle
by a linker called spoke. 33. Cell wall of ––––– contains long chain fatty acids of mycolic
(c) C is made up of protein hexin. acid.
(d) Part C is made up of 11 protofilaments. (a) Bacillus subtilis (b) E. coli
29. Consider the following statements. (c) Streptococcus (d) Mycobacterium
I. Golgi apparatus in plant cell consists of distinct units
called dictyosomes.
II. Glyoxysomes are absent in plant cells but present in
ANSWERS WHO AM I ...
animal cells. 1. Arbor vitae Pg. 15
III. Specialised vacuoles in plant cells are analogous to
lysosomes in animal cells. 2. Iris Pg. 25
IV. Absence of cell wall in animal cell results in its bursting 3. Non-competitive inhibitor Pg. 40
in hypertonic solution.

MT BIOLOGY TODAY | AUGUST ‘17 53


34. Study the given diagram of a typical plant cell and 37. A frog has
identify the option that correctly corresponds with the (a) eyes but no lids (b) jaws but no teeth
function of labelled parts A-D. (c) hands but no fingers (d) ears but no pinnae.
D 38. Match the column I containing various part of earthworm
with column II showing its body segments.
C
B Column I Column II
p. Typhlosole (i) 4th-6th segments
q. Stomach (ii) 9th-14th segments
r. Pharyngeal nephridia (iii) 6th-9th segments
A s. Spermathecae (iv) 26th-35th segments
(a) A-Site for synthesis of glycoproteins. (a) p-(ii), q-(iv), r-(iii), s-(i)
(b) B-Synthesis of acrosome found at the tip of animal sperms. (b) p-(iv), q-(ii), r-(i), s-(iii)
(c) C- Coupling factors found in the matrix and thylakoids (c) p-(i), q-(ii), r-(iv), s-(iii)
are responsible for ATP synthesis.
(d) p-(iv), q-(iii), r-(i), s-(ii)
(d) D-Concentrating and packaging of the secretory
substances inside a soluble protein coat. 39. Consider the following statements and select the option
35. Read the given statements and select the option stating with correct set of statements.
which ones are true (T) and which ones are false (F). (i) Oligodendrocytes are found in the central nervous
(i) Non-striated muscle fibres have nerve supply from system and have a function similar to Schwann cells.
autonomic neural system. (ii) Microglial cells engulf microbes and debris in the
(ii) Presence of Balbiani rings is a characteristic feature of nervous tissue.
lampbrush chromosomes. (iii) Ependymal cells play an important role in maintenance
(iii) Cell wall of cork cells contains a special fatty substance of the blood brain barrier.
called suberin.
(iv) The movement of astrocytes processes is responsible
(iv) In cockroach, anus lies in the 8th segment just below
for flow of cerebrospinal fluid.
the tergum.
(a) (i) and (iii) only
(i) (ii) (iii) (iv)
(b) (ii), (iii) and (iv) only
(a) T T T F
(b) T F T T (c) (i) and (ii) only
(c) T T F F (d) (ii) and (iv) only
(d) T F T F 40. Which option is correctly matched with its function?
36. Which of the following statements is incorrect? (a) Desmosome - Increases surface area of
(a) In cockroach, the auditory sensillae are located on the contact between cells
anal cerci. (b) Zonula adherens - Facilitates chemical exchange
(b) Gram –ve bacteria are more resistant to antibiotics between cells
than Gram +ve bacteria. (c) Tight junction - Checks flow of material
(c) Nucleoproteins are synthesised and stored in the between the cells
nucleolus.
(d) Gap junction - Joins epithelial cells with
(d) Mast cells produce serotonin that acts as a
basement membrane
vasoconstrictor.
Key is published in this issue. Search now! 

Check your score! If your score is


> 90% EXCELLENT WORK ! You are well prepared to take the challenge of final exam.

No. of questions attempted …… 90-75% GOOD WORK ! You can score good in the final exam.
No. of questions correct …… 74-60% SATISFACTORY ! You need to score more next time.
Marks scored in percentage …… < 60% NOT SATISFACTORY! Revise thoroughly and strengthen your concepts.

54 MT BIOLOGY TODAY | AUGUST ‘17


The syllabus for NEET is very vast which impedes students from
acquiring indepth knowledge and covering the entire syllabus
at the same time. An essential topic for NEET is therefore
presented here to enable students grasp the topic, analyse the
type of questions and SCORE HIGH.

APPLICATIONS OF BIOTECHNOLOGY
Biotechnology is the use of living systems and organisms to develop or make useful products. It harnessess cellular and biomolecular
processes to develop technologies and products that help to improve our health and lives.
Modern biotechnology provides breakthrough solutions and technologies to combat debilitating and rare diseases, reduce our
environmental foot-prints, support use of less and cleaner energy, contribute safer and more efficient industrial manufacturing
processes and improve areas of food production and health.
The diverse applications of biotechnology in various fields have led to emergence of different branches of biotechnology.

Red biotechnology White biotechnology


Refers to medical biotechnology, used in Refers to industrial biotechnology, used
designing organisms, treating and analysing and applied in industries and its various
genetic diseases, production of drugs and processes as production of biopolymers,
antibiotics, etc. biofuels, fermentation, etc.
Branches of
Green biotechnology Biotechnology Blue biotechnology
Refers to biotechnology in agricultural Refers to biotechnology related
sector, includes creation and designing of to marine and other aquatic
transgenic plants, development of various organisms. Least developed branch of
agricultural processes as micropropagation. biotechnology.

MT BIOLOGY TODAY | AUGUST ‘17 55


APPLICATIONS IN AGRICULTURE
Despite the increase in food production through green revolution, purpose is not served. Increasing population and new challenges
in agricultural sector, seek new solutions to overcome these problems.
One such solution is production of genetically modified (GM) crops.
Genetically modified organisms refer to organisms that have been genetically manipulated or modified. They can be plants,
animals or even microbes.
Genetically modified crops contain and express one or more useful, desirable foreign genes. Such genes incorporated into the
genome of organisms are called transgenes and the plants expressing the products of transgenes are called transgenic plants.

Production of Transgenic plants using Ti Plasmid


The vector used to introduce new genes into plant cells is most often a plasmid from soil bacterium Agrobacterium tumefaciens.
Since the plasmid possesses genes that transfer the DNA from plasmid and gets integrated into host (plant genome), it is called
natural genetic engineer. The plasmid induces tumors in broad leaf plants such as tomato, tobacco, soyabean, etc., hence is also
referred to as tumor inducing plasmid (Ti plasmid).
Plant
chromosomal
DNA

Bacterial genome T-DNA


T-DNA
Ti plasmid
Crown gall

Agrobacterium Transformed plant


tumefaciens cell
Fig. : A. tumefaciens as natural genetic engineer

( )

The transgenic plants are produced for variety of purposes such as they carry desirable traits for disease resistance, insect
resistance and herbicide resistance and improved food quality. Some of them are discussed here.

56 MT BIOLOGY TODAY | AUGUST ‘17


Insect Resistance in Transgenic Plants
• Soil bacterium Bacillus thuringiensis produces proteins that kill certain insects like lepidopterans (tobacco budworm, army
worm), coleopterans and dipterans (flies, mosquitoes).
• Bacteria produces protein crystals containing inactive toxin (protoxins). Hence, these toxins does not harm the bacteria, but
kill insects when they ingest plant parts expressing them. Alkaline pH of alimentary canal of insect, solublises the crystal
and converts inactive protoxins into active toxins.
• Through genetic engineering, these Bt toxin genes were isolated and incorporated into several crop plants as tobacco,
cotton, corn, etc.
• The choice of gene depends upon the crop and its target pest, as most of the toxins are pest or insect group specific.
• Cry genes : In cotton, cry I Ac and cry II Ab provide resistance against cotton bollworms. In corn cry I Ab provides resistance
against corn borer.

Fig.: Mode of action of Bt toxins in insect gut

Pest Resistance in Transgenic Plants


• Many nematodes living as parasites in plants, affect their productivity e.g., Meloidogyne incognita infects the roots of tobacco
plants and greatly reduces its yield.
• To prevent this infection and make plant pest resistant, a novel strategy based on the process of RNA interference is adopted.
• RNA interference (RNAi) is the phenomenon of inhibiting activity of a gene by synthesis of RNA molecules complementary
to mRNA.
• Using Agrobacterium vectors, nematode specific genes are introduced into host plant and it produces both sense and
antisense RNA. The complementary strand of mRNA does not contain sequences that can be translated to produce a
functional protein hence called antisense RNA.
• The antisense RNA and mRNA molecules anneal to form duplex RNA molecules, which block translation of mRNA of affected
gene.

MT BIOLOGY TODAY | AUGUST ‘17 57


• Different steps involved in making tobacco plant resistant
to nematode are as follows :
– Double-stranded RNAs are processed into approximately
21-23 nucleotide RNAs. An RNase enzyme, called Dicer,
cuts the dsRNA molecules (from a virus, transposon or
through transformation) into small interfering RNAs
(siRNAs).
– Each siRNA forms an RNA-induced silencing
complex (RISC) with ribonucleases (distinct from
Dicer).
– The siRNA unwinds and RISC is activated.
– The activated RISC targets complementary mRNA
molecules. The siRNA strands act as guide for RISCs to
identify and cut the mRNA. This destroys the mRNA.
– When mRNA of the parasite is destroyed, no proteins
are synthesised that results in the death of the parasite
(nematode).
– As the transgenic plant expresses antisense RNA,
mRNA of nematode gets silenced and its infestation in
the tobacco plant fails thus these transgenic plants get
protected from the parasite.
Herbicide Resistance in Transgenic Plants
• Transgenic plants resistant to certain biodegradable herbicides have been successfully developed through genetic engineering.
• Herbicides like glyphosate is a broad spectrum herbicide that inhibits the enzyme 5-enolpyruvylshikimate-3-phosphate
synthase (EPSPS). This enzyme is involved in aromatic acid biosynthesis in plants. Thus, the glyphosate kills plant by making
its cells deficient of aromatic amino acids which disrupts their protein synthesis.
Table : Transgenes for herbicide resistance and the mechanisms of their action
Transgene Source organism Confers Mechanism of action/Remarks
resistance to
the herbicide
aroA Agrobacterium sp. strain Glyphosate/ Encodes 5-enolpyruvylshikimate-3-phosphate synthase
CP4 Roundup (EPSPS) enzyme that is insensitive to glyphosate
bar Streptomyces Bialaphos/Basta/ A c e t y l a t i o n o f p h o s p h i n o t h r i c i n ( e n z y m e
hygroscopicus Glufosinate phosphinothricin acetyl tansferase)
bxn Klebsiella ozaene Bromoxynil Converts the herbicide into 3, 5-dibromo-4-hydroxy
benzoic acid
gax Achromobacter sp. strain Glyphosate/ Detoxifies glyphosate by oxidising it into amino
LBAA Roundup ethylphosphonic acid and glyoxylate
Hra, csr1-1, C3 and Arabidopsis thaliana, N. Sulfonylureas and Encode acetolactate synthase (ALS) enzyme that is
ahas3r tabacum, yeast, E. coli imidazolinones insensitive to the herbicides

Transgenic Plant to Reduce Post-Harvest Losses


• Post-harvest and overripening losses have been reduced by introducing genes which prevent deterioration of harvested
products.
• The tomato variety ‘Flavr Savr’ presents an example where expression of a native tomato gene, coding for the enzyme
polygalacturonase has been blocked.
• This enzyme promotes fruit softening by degrading pectin. Non-availability of this enzyme makes fruits of transgenic tomato
variety remain fresh and retain their flavour for much longer than do the fruits of normal tomato varieties. Additionally, these
fruits have superior taste and increased total soluble solids.

58 MT BIOLOGY TODAY | AUGUST ‘17


Transgenic Nutritionally Enhanced Plants
• Certain crops are genetically modified to enhance the nutritional value of the food they produce e.g., vitamin A enriched rice.
• Prof. Ingo Potrykus and Peter Beyer produced genetically engineered rice by introducing three genes associated with
synthesis of carotene.
• It is a transgenic variety of rice (Oryza sativa) which contains good quantities of -carotene (provitamin A- inactive
state of vitamin A). Since the grains (seeds) of the rice are yellow in colour due to -carotene, the rice is commonly called
golden rice.

Fig.: Steps involved in production of golden rice

Transgenic Plants Producing Various Biochemicals


• Plants are the chief source of carbohydrates, e.g., starch, sugar, lipids, proteins and a variety of unique biochemicals.
• Transgenes have been shown to introduce novel branches in the biosynthetic pathways of plants and thereby, to generate
valuable products or to produce new valuable proteins.
• Hirudin (the first commercial biochemical) is a protein that stops blood clotting. The gene encoding hirudin was
chemically synthesised. This gene was then transferred into Brassica napus, where hirudin accumulates in seeds. The hirudin
is then extracted and purified to be used as medicine.
Disadvantages of GM Crops
• GM crops can cause unintended harm to other organisms. A laboratory study on high mortality rate in Monarch butterfly
caterpillars that feed on pollen of Bt corn seemed to support the view point. Monarch caterpillars feed on milkweed plants
that might have pollens of Bt corn blown by wind onto it.
• Use of GM crops can reduce the effectiveness of the pesticides as insects can become resistant to Bt or other crops that
have been genetically modified to produce their own pesticides.
• Another concern is that crop plants engineered for herbicide tolerance and weeds can cross-breed, resulting in the transfer of
the herbicide resistant genes from the crops into the weeds. These “superweeds” would then be herbicide tolerant as well.
• Moreover, GM food can lead to allergies, they can harm bacteria of the alimentary canal, etc.

APPLICATIONS IN MEDICINE
• The recombinant DNA technology has made great impact in the area of healthcare by mass production of safe and more
effective therapeutic drugs.

MT BIOLOGY TODAY | AUGUST ‘17 59


• Now about 30 recombinant therapeutics have been approved for human use, all over the world. In India, 12 of these are
presently being marketed.
• Several of the recombinant proteins, used for treatment of diabetes mellitus (e.g., insulin), dwarfism (e.g., human growth
hormone [hGH]), cancer (e.g., interferons, interleukins, granulocyte macrophage colony stimulating factor), thrombosis (e.g.,
streptokinase) and AIDS (e.g., interferons, granulocyte macrophage colony stimulating factor) has been produced.

Production of Genetically Engineered


Insulin
• Insulin is made of 51 amino acids, arranged in two short
polypeptide chains : chain A with 21 amino acids and
chain B with 30 amino acids, that are linked together
by disulphide bridges. In humans, insulin is synthesised
as a pro-hormone which contains an extra stretch called
the C-peptide with 33 amino acids. This C-peptide is not A
present in the mature insulin, as it is removed during its B
maturation.
• The main challenge for production of insulin using rDNA
techniques was getting insulin assembled into a mature
form.
• In 1983, Eli Lilly an American company prepared two DNA sequences corresponding to A and B chains of human insulin
and introduced them in plasmids of E.coli to produce insulin chains.
• Chains A and B were produced separately, extracted and combined by creating disulfide bonds to form human insulin
(Humulin).
DNA

Recombinant bacteria
Human Introduction of multiply and produce
pancreas cell recombinant DNA human insulin (either
Recombinant peptideA or B depending
Human insulin into a bacterial cell
bacterium upon the gene cloned)
producing gene in the fermentation tank
coding A and
B peptides Fermentation
tank
Annealing by DNA ligase

Recombinant Human insulin is


DNA extracted, purified
Human insulin and packaged into
bottles

Plasmid DNA cut with


Bacterial DNA restriction enzymes

Plasmid
DNA
Bacterium
Fig.: Steps in production of recombinant insulin

Production of Human Growth Hormone


Growth of human body is regulated by human growth hormones (hGH), Somatotropin. Secretion of somatotropin is regulated
by somatotropin releasing hormone and somatostatin or growth inhibiting hormone.
Somatostatin is the first human protein to be produced in E.coli. It is only 14 amino acids long. The gene for somatostatin
was chemically synthesised and expressed as a fusion protein with lac ZX sequence of -galactosidase.
However, the production of somatotropin, being 191 amino acids long, presented a more difficult problem. Therefore, a combination
of artificial gene synthesis and complementary (cDNA) DNA cloning was done.

60 MT BIOLOGY TODAY | AUGUST ‘17


Production of Bovine Growth Hormone
Engineered E.coli produce bovine somatotropin or bovine growth homrone (bGH). This hormone, when injected into cows,
raises milk production as well as beef output. The engineered E.coli produce the enzyme cellulase, which hydrolyses cellulose
and enables animals to utilise the entire feed.

Plasmid Escherichia coli

1
Plasmid is removed and
cut open with restriction
endonuclease. 5
Bacteria producing
bovine somatotropin
are grown on large
3
Somatotropin gene scale.
is inserted into
bacterial plasmid. 4
Plasmid is reintroduced 6
into bacterium. Somatotropin is
removed from
Gene of bacteria and purified.
interest

2
Cow somatotropin
gene is isolated from 7
Bovine somato-
cow cell. tropin is adminis-
tered to cow to
enhance milk
Cow cell production.

Fig.: Bovine somatotropin production

Production of Recombinant Vaccines


• Conventional vaccines consist of attenuated or inactivated pathogens while the gene encoding a critical antigen in a pathogen
has been isolated and expressed in bacterial or animal cells to produce recombinant proteins. These proteins are used as
a vaccine and such vaccines are called recombinant vaccines.
• But usually the entire protein molecule is not necessary for an effective immunisation since the antigenic property is ordinarily
localised in a small part of protein molecules. Therefore, either a constituent polypeptide (in case of heteromultimeric proteins)
or a small part of a polypeptide may be used as a vaccine, such vaccines are known as subunit vaccines.
• Subunit vaccines are produced using recombinant DNA technology for cloning and expressing the appropriate DNA sequences
encoding antigenic polypeptides as its segments in suitable hosts.
• Recombinant vaccines are produced through bacterial fermentation or in animal cell cultures that makes their cost prohibitively
high, thus, preventing their wide use e.g., the oral cholera vaccine consisting of recombinant cholera-toxin B subunit.

DNA Vaccines
• DNA vaccines are composed of a small, circular piece of DNA, called a DNA plasmid that contain genes which code for
proteins of a pathogen. When the vaccine is injected into the host, the inner machinery of the host cells ‘reads’ the DNA,
converts it into proteins from the pathogen. Recognising that the protein is foreign, the cells display them on their surface
to alert the body’s immune system– both helper T-cells, which spur the production of antibodies and killer T-cells. This triggers
a range of immune responses and therefore, DNA vaccines induce wider range of immune responses.

Production of Interferons
• Interferons (IFNs) are members of a large group of proteins called cytokines, which affect a wide range of target cells and
tissues by binding to specific receptors present on the surface of their target cells. Interferons are proteins produced by the
infected cells in response to viral infections. They diffuse to neighbouring cells and trigger a reaction that neutralises the
particular virus and prevent viral infections.

MT BIOLOGY TODAY | AUGUST ‘17 61


• Until recently the only source of interferons was human white blood cells or virus infected human cells grown in tissue
culture.
• Production of human interferon by cloning of genes in colon bacilli was started in 1980 by two American scientists Gilbert
and Weissmann.
• The interferons (particularly IFN- ) are used on a significant scale for the treatment of hepatitis-B. Interferons are also being
tested for the treatment of cancer and other viral diseases including AIDS.

Production of Monoclonal Antibodies


• Monoclonal antibody is a specific antibody produced from a normally short lived antigen-activated B-cell that has been
immortalised by hybridising it with a myeloma cell, by hybrid cell culture technique, known as hybridoma technology.
• In this technique the antigen (against whom the antibodies are required) is injected in rat or mouse.
• After sometime B-lymphocytes from spleen of an animal are isolated and mixed with the myeloma cells (isolated from
cancer cells of bone marrow).
• Antibodies producing B-lymphocytes and myeloma cells fuse together to form hybridoma cells.
• These hybrid cells retain the ability of the B-cells to secrete antibody and the ability of myeloma cell to grow indefinitely.
• The hybridoma cells in culture conditions produce specific and pure (monoclonal) antibodies rapidly.
• These are isolated and purified to be used in the cure of diseases.
• The advantage of monoclonal antibody is that it is derived from a single cell and comprises a uniform breed of antibody,
specific for a single antigen site (epitope).
• In 1984, Kohler and Milstein shared the Nobel Prize in Physiology or Medicine for the development of the monoclonal
antibody technique.

Gene Therapy
• This technique is based on the fact that cells of the body manufacture substances under the guidance of DNA. Thus, by
manipulating the DNA structures, it is possible to control various diseases.

62 MT BIOLOGY TODAY | AUGUST ‘17


• It is a collection of methods that allows correction of a gene defect that has been diagnosed in a child/embryo.
• Correction of a genetic defect involves delivery of a normal gene into the individual or embryo to take over the function of
and compensate for the non-functional gene.

Types of gene therapy

Germ line gene therapy


Involves modification of germ cells i.e., sperms or eggs by introduction of functional genes, which are usually integrated
into their genomes. This mode of therapy is inheritable and is highly effective in counteracting the genetic disorders.

Somatic cell gene therapy


Involves modification of somatic cells by introduction of concerned gene. This is not heritable.
The expression of gene in the body cells however relieves or eliminates symptoms of the disorder temporarily.

Gene therapy for treating SCID


The first gene therapy trials began in 1990 with the treatment of a 4 years old girl, Ashanti DeSilva, who had a genetic disorder
called severe combined immunodeficiency (SCID). SCID is caused by a mutation in the gene encoding the enzyme adenosine
deaminase (ADA). This enzyme is very important for the immune system to function. Because these patients do not have functional
T- lymphocytes, they cannot provide immune responses against invading pathogens.
• As a first step towards gene therapy, lymphocytes are extracted from the bone marrow of the patient and are grown in a
culture outside the body. A functional ADA cDNA (using a retroviral vector) is then introduced into these lymphocytes,
which are reinjected to the patient’s bone marrow.

Normal ADA gene copies are produced by cloning

Packed into a modified retrovirus, where viral genes are replaced by ADA gene

Lymphocytes are isolated from bone marrow of patient and cultured outside (in vitro)

Recombinant retroviruses are used to infect these lymphocytes

Infected lymphocytes express ADA gene

Such lymphocytes are injected back into the patient’s body

Thus, the ADA gene is expressed in the patients and the deficiency of ADA gene gets partially corrected
Flow Chart : Steps of gene therapy for the treatment of SCID
• The drawback of this therapy is that these cells are mortal, hence the patient requires periodic infusion of such genetically
engineered lymphocytes. However, if the therapy is done into cells at early embryonic stages, the ADA deficiency can be
permanently cured.

Molecular Diagnosis of Diseases


• Early diagnosis and understanding the symptoms are very important for effective treatment of disease but early detection
is not possible using conventional methods.
• PCR, ELISA are some of the biotechniques that serve the purpose of early diagnosis.
• PCR can detect mutation even in very low amounts of DNA, as it amplifies DNA by making billions of copies of a DNA
segment in few hours. It is a good technique to detect mutations and identify genetic disorders, especially in detecting HIV
in AIDS patients, mutated genes in cancer patients and other genetic disorders.
• Enzyme linked immunosorbent assay (ELISA) : ELISA is based on the principle of antigen - antibody interaction.
Infection by pathogen can be detected by the presence of antigens such as proteins, glycoproteins, etc., or by detecting the
antibodies synthesised against the pathogen.

MT BIOLOGY TODAY | AUGUST ‘17 63


APPLICATIONS IN INDUSTRY
• The exploitation of microorganisms for industrial production of useful products began as early as 1875 with the production
of alcohol.
• At present, several chemicals, such as lactic acid (1881), amylase (1884), glycerine (1914), citric acid (1923), gluconic acid
(1928), acetic acid, acetone, butanol, a variety of enzymes, vitamins, amino acids and all the antibiotics are produced using
microorganisms.
Production of Enzymes
• In 1874, Christian Hansen prepared pure enzyme by extracting ‘rennet’ from the calf’s stomach.
• The bulk of industrial enzymes are prepared almost exclusively from Gram positive bacteria and fungi.
• Enzymes are purified by precipitation and filtration to remove undesirable metabolites and particulate matter and to improve
stability and standardise activity.
• Different types of specific enzymes used in rDNA technology are lysing enzymes, restriction enzymes, DNA ligases, etc.

Table : Some common industrial enzymes, their sources and uses


Enzyme Source Principal use
a- Amylase Aspergillus oryzae, Bacillus amyloliquefaciens Production of alcohol, preparation of glucose
Bacillus licheniformis syrup
Cellulase Aspergillus sp., Trichoderma koningi Alcohol and glucose production
b-Glucanase Bacillus subtilis, Aspergillus niger -Glucan hydrolysis in brewing
Glucoamylase Aspergillus niger, Bacillus amyloliquifaciens Glucose syrup production from liquefied starch
Glucose isomerase Anthrobacters, Bacillus sp. Isomerisation of glucose into high fructose
syrups
Lactase Saccharomyces fragilis, Hydrolysis of lactose
Kluyveromyces marxiamus Removal of lactose from whey
Lipase Aspergillus sp., Candida lipolytica Cheese and butter preparation, flavour
modification
Pectinase Aspergillus niger Extraction and clarification of fruit juices
Penicillin amidase Bacillus megathenium, Escherichia coli Synthesis of 6-aminopenicillanic acid for
manufacture of semisynthetic antibiotics
Protease (alkaline) Bacillus licheniformis Detergent and leather industries

64 MT BIOLOGY TODAY | AUGUST ‘17


Protease (neutral) Bacillus amyloliquefaciens Fish and meat tenderiser
Protease (acid) Aspergillus niger Digestive aid, substitute for calf rennet
Pullulanase Klebsiella pneumoniae Starch hydrolysis
Streptokinase Hemolytic bacterium, Streptococcus Lysis of blood clots

Production of Antibiotics
• The term antibiotics may be defined as a chemical substance produced by some microorganisms that in small concentration
can kill or inhibit the growth of other harmful microorganisms without affecting the host.
• Antibiotics function either as bactericides (killing bacteria) or bacteriostatic (inhibiting growth of bacteria).
• Suitable strain of microorganism is cultivated on a sterilised nutrient medium provided with optimum pH, aeration, temperature,
antifoaming agent and antibiotic precursor.
• When sufficient antibiotic has diffused into medium, the microorganisms are separated and the antibiotic is extracted from
medium by precipitation, absorption or solvent. It is purified, concentrated and bioassayed before packing.
• The production of newer antibiotics is a never ending process because microbes continuously become resistant to older
antibiotics. S. griseus produces more than 41 antibiotics while Bacillus subtilis can give about 60 antibiotics.

Production of Food and Drinks


• Some of the oldest biotechnology procedures are concerned with the manufacture of various foods and drinks.
• Yeast fermentation or alcoholic fermentation are carried out to produce bread, brewing or malting of sugars from barley
grain to obtain beer and production of other alcoholic drinks such as wine, brandy, rum, whisky, etc., by distillation.
• Microbial fermentation involves large scale fermentation of various microbes capable of producing specific vitamins.
These vitamins are essential for normal health and complete growth of organisms.

Industrial Biotechnology
Recovery of metals
Various microbes are employed to recover valuable metals from low grade ores and from dilute solutions. This
process is called as microbial ore leaching, e.g., Thiobacillus ferroxidans induces leaching of copper as copper
sulphates from ores.

Biotransformation
It is the process by which organic compounds are transformed into other recoverable products by the use of enzymes
present in microbes, plants or animal cells, e.g., biotransformation is used to produce several antibiotics and steroids.
Penicillin G acylase Modified chemically
Penicillin G (Deacylation) 6-aminopenicillanic acid Semisynthetic penicillin + Cephalosporin

Production of biocontrol agents


The microorganisms used to control insects, pests, pathogens and even weeds are considered biocontrol agents. Such
wide strains of bacteria and fungi are developed on large scale in fermenters.

Production of biofertilisers
Microorganisms used to enhance availability of nutrients, viz., nitrogen and phosphorus are produced and made available
commercially, e.g., Rhizobium, Azotobacter, Azospirillum, blue green algae.

Metabolite production
Various metabolites such as acetone, alcohol, antibiotics, enzymes, organic acids, etc., are produced using variety of
microorganisms concerned.

TRANSGENIC ANIMALS
• A transgenic animal contains a foreign gene in its genome introduced by one or the other technique of transfection.

MT BIOLOGY TODAY | AUGUST ‘17 65


Production of Transgenic Animals
• The foreign genes are inserted into the genome of the animal using recombinant DNA technology.

Identification and isolation of desired gene

Selection of a proper vector (usually a virus) or direct gene transfer

Insertion of gene within the vector

Introduction of recombinant vector into the host cells, tissues of embryo or mature individual

Demonstration of integration and expression of foreign gene in transgenic tissue or animal


Flow chart : Steps in production of transgenic animals.

Applications of Transgenic Animals


Production of biological products
Transgenic animals that produce useful biological products can be created by the introduction of the portion of
DNA (or genes) which codes for a particular product. Such as human protein ( -1-antitrypsin) to treat emphysema,
human alpha lactalbumin, etc.

Production of clones
Clones of animals with desirable traits can be produced.

Replacement of defective parts


Defective parts can be replaced with freshly grown parts from own cells or transgenic animals.

Study of physiology and development


Transgenic animals are developed to study how genes are regulated and how they affect normal functions of body and
its development, e.g., study of factors involved in growth.

Testing vaccine safety


Before being used on humans, transgenic mice are used to test safety of newly developed vaccines, e.g., transgenic
mice was used to test safety of polio vaccine.

Study of diseases
Transgenic animals act as models for study and understanding of causes and possible treatments of disease in concern,
e.g., cancer, cystic fibrosis, Alzheimer’s disease.

Examples of Transgenic Animals


Transgenic cattle
• The only successful transfection technique in cattle is microinjection of fertilised ova, which may either be recovered
surgically or may be obtained from ovaries extracted from slaughtered cows and cultured in vitro.
• The chief objectives of transgenic cattle production are as follows:
– increased milk or meat production
– molecular farming
– improved properties and proportions of casein in milk.
– regulate content of lactose and butterfat in milk
– enhanced resistance to viral and bacterial pathogens.
• The name of first transgenic cow is Rosie.

66 MT BIOLOGY TODAY | AUGUST ‘17


Transgenic mice
• Mouse is the most preferred mammal for studies on gene transfer.
• Favourable features of mice as a suitable model are:
– short oestrous cycle and gestation period
– relatively short generation time
– production of several offsprings per pregnancy (i.e., litter)
– convenient in vitro fertilisation
– successful culture of embryos in vitro at least for a period of time
– production and maintenance of ES (embryonic stem) cell lines
– availability of a diverse array of genetic stocks, etc.
• As a result, the techniques of gene transfer and transgenic production have been developed using mice as models.

Ethical Issues
The main bioethical concerns pertaining to biotechnology are briefly mentioned as follows:

Introduction of a transgene from one species into another species violates the ‘integrity
of species’.
Biotechnology may pose unforeseen risks to the environment, including risk to biodiversity.
Thus, it could disturb the existing ecological balance.

Transfer of human genes into animals (and vice-versa) dilutes the concept of ‘humanness’.

Use of animals in biotechnology causes great suffering to them.


Scientists cannot rule out the possibility of other biological damage. It can accidentally
create new infectious agents.

When animals are used for production of pharmaceutical proteins, they are virtually reduced
to the status of a ‘factory’.

It is disrespectful to living beings and only exploits them for the benefit of human beings.

Role of biotechnology in Environment


Biotechnology is applied for processes to protect and restore the quality of environment. It is used to detect, prevent and
remediate the emission of pollutants into the environment in a number of ways. It also makes a significant contribution to
sustainable development.
Some important applications are aimed at:
• processes that make optimal use of natural resources by recycling biomass, recovering energy and minimising waste generation
• use of biotechnological techniques for bioremediation of land and water, waste treatment, soil conservation, reforestation,
land rehabilitation, etc.,
• biodegradation of xenobiotic compounds.

Bioremediation
Bioremediation refers to the process of removal or detoxification of pollutants from contaminated sites, with the use of
biological organisms as bacteria, fungi or plants. Certain bacterial, fungal and algal species capable of accumulating some
toxic inorganic contaminants are developed in the laboratory and introduced at the site. Phytoremediation on the other
hand is use of green plants and their associated microorganisms, soil ammendments to remove both organic and inorganic
contaminants.

GENETICALLY ENGINEERED MICROBES (GEMS)


The microorganism in which a gene or genes have been introduced by r-DNA technology are referred to as genetically engineered
microbes. Such microbes are developed to fulfil specific needs and perform functions such as :

MT BIOLOGY TODAY | AUGUST ‘17 67


• Producing pharmaceutically useful proteins
• Protection of crops from insects (e.g., Clavibacter containing cry gene), fungal diseases (e.g., Pseudomonas expressing
chitinase gene from Serratia), frost damage (e.g., ice mutants of Pseudomonas fluorescens, since ice gene regulates ice
crystal formation at low temperatures), etc.
• Enhancing nitrogen fixation by amplification of nif genes in R. meliloti.

Pseudomonas putida- A superbug


This genetically engineered bacteria was developed by using plasmids from four species of oil eating bacteria and joined
to form recombinant plasmid and introduced into the bacteria. This plasmid conferred the ability to digest the four types
of petro-chemical products i.e., octane, xylene, campher and napthalene.
Hence, it is called a superbug or oil eating bug. It is a potent solution to the problem of soil petro pollution or oil spilling
in sea.

STEPS IN APPROVAL OF GMOs


GMO
ion
Applicat
GMO Registrar

DEPARTMENT
OF
AGRICULTUR
E

Interested parties submit an The GMO application is sent to the GMO The registrar checks if the
application to develop GMO ’s registrar at the department of agriculture application meets the
req uirements of the GMO act
A permit may be issued
GMO GMO
Application for contained use… GMO ADVISORY COMMITTEE
Applicatio Application
n

PERMIT
RE
JE
CT
ISSUED Field trials…
ED
INDE
PEND
EXPE ENT
… or commercial commodities. RTS

An advisory committee evaluates the


executive application. Additional information may be
council Reco req uested from the applicant.
enda mm- Public
GMO do tion
Publi cument Input
Application G Input c
GMO
Appli MO Application
catio
n
DoL L
oH DoH Do
DTI
DEAT
DoA n
DST mendatio
Recom ment
docu
The GMO registrar is informed The executive council evaluates the application
of the decision of the executive and consider the recommendation A recommendation document is
council and implements it. from the advisory committee and public input. prepared by the advisory committee.

New MCQs
1. During the formation of recombinant insulin, maturation of 2. The genetically modified crop of golden rice is a variety
proinsulin into insulin is processed by showing
(a) adding C-peptide to proinsulin (a) insect resistance
(b) adding B-peptide to proinsulin (b) herbicide resistance
(c) removal of C-peptide from proinsulin (c) golden coloured grains with large size
(d) removal of A-peptide from proinsulin. (d) adequate quantities of provitamin A.

68 MT BIOLOGY TODAY | AUGUST ‘17


3. The diagnostic technique of ELISA is based on the principle (a) transgenic food may cause toxicity in humans
of (b) transgenic food may cause allergic reactions in humans
(a) antigen-gene interaction (c) the bacteria present in human alimentary canal may
(b) antigen-antibody interaction become antibiotic resistant
(c) lectin-antibody interaction (d) all of these.
(d) DNA-mRNA interaction. 12. Match column I with column II and select the correct option.
4. Flavr-Savr tomatoes remain fresh for a longer duration due Column I Column II
to the A. Rosie (i) Insect resistant
(a) suppression of activity of polygalacturonase that B. Flavr-Savr tomato (ii) Enriched with vitamin A
inhibits fruit ripening C. Golden rice (iii) Enriched with human
(b) blockage of gene encoding enzyme polygalacturonase -lactalbumin
(c) expression of enzymes that promote pectin degradation D. Bt cotton (iv) Increased shelf life
(d) all of these. (a) A-(ii), B-(iv), C-(i), D-(iii)
(b) A-(i), B-(iii), C-(ii), D-(iv)
5. Which of the given genetically modified crops can be (c) A-(iii), B-(iv), C-(ii), D-(i)
consumed to prevent blindness associated with vitamin A (d) A-(ii), B-(iii), C-(iv), D-(i)
deficiency?
(a) Bt corn (b) Flavr-Savr tomato 13. The gene therapy technique involves
(c) Golden rice (d) Bt brinjal (a) introduction of a foreign gene in a cell
(b) treatment of defective genes with radiation
6. Consider the following statements regarding Bacillus
thuringiensis and select the incorrect ones. (c) replacement of abnormal gene with functional gene
I. Bacillus thuringiensis produce proteins that kill (d) elimination of defective genes.
lepidopteran and coleopteran insects only. 14. Transgenic animals are those animals which
II. The genes cry IAc and cry IAb control cotton bollworms. (a) show resistance to various antibiotics
III. Alkaline pH of insect’s alimentary canal solublises (b) are used for testing vaccine safety before using for
inactive protoxins and convert them into active form. humans
IV. Bt toxin proteins are known as insecticidal proteins. (c) are used in laboratory for study of various biochemical
(a) I and III (b) III and IV processes
(c) I and II (d) I, III and IV (d) none of these.
7. A genetically engineered microorganism used successfully in 15. The natural genetic engineer of plants, capable of
bioremediation of oil spills is transforming plant’s genome is
(a) Bacillus (b) Xanthomonas (a) Agrobacterium tumefaciens
(c) Pseudomonas (d) Agrobacterium. (b) Escherichia coli
8. The transgenic cow ‘Rosie’ was developed by introducing (c) Bacillus thuringiensis
gene for (d) Pseudomonas putida.
(a) -1-antitrypsin (b) human -lactalbumin
(c) -1 antitrypsin (d) cry IAc. Exam Section
9. Hybridomas are formed as a result of fusion of 1. Which kind of therapy was given in 1990 to a four-year-old
girl with adenosine deaminase (ADA) deficiency?
(a) abnormal antibody producing cells with myeloma cells
(a) Gene therapy (b) Chemotherapy
(b) male germ cells with myeloma cells
(c) Immunotherapy (d) Radiation therapy
(c) normal antibody producing cells with myeloma cells (NEET Phase-II 2016)
(d) normal antibody producing cells with ova.
2. Which part of the tobacco plant is infected by Meloidogyne
10. The technique in which sperms or eggs are modified by the incognita?
introduction of functional genes is (a) Stem (b) Root
(a) PCR (b) hybridoma technology (c) Flower (d) Leaf (NEET Phase-I 2016)
(c) ELISA (d) gene therapy. 3. Golden rice is a genetically modified crop plant where the
11. Despite so many GM crops being produced, very few of them incorporated gene is meant for biosynthesis of
are released and cultivated commercially. This is due to the (a) omega 3 (b) vitamin A
fact that (c) vitamin B (d) vitamin C. (AIPMT 2015)

MT BIOLOGY TODAY | AUGUST ‘17 69


4. The introduction of T-DNA into plants involves (a) Bt toxin gene (b) barnase gene
(a) exposing the plants to cold for a brief period (c) lectin gene (d) chitinase gene. (AMU 2015)
(b) allowing the plant roots to stand in water 11. A transgenic food crop which may help in solving the
(c) infection of the plant by Agrobacterium tumefaciens problem of night blindness in developing countries is
(d) altering the pH of the soil, then heat-shocking the (a) Golden rice (b) Flavr-Savr tomatoes
plants. (AIPMT 2015)
(c) Starlink maize (d) Bt soybean. (J & K 2015)
5. Which organism is used to transfer T-DNA?
(a) Streptomyces hygroscopicus 12. The hybridoma cell concept was introduced by
(b) Agrobacterium tumefaciens (a) Kohler and Milstein
(c) Salmonella typhi (b) Schleiden and Schwann
(d) Escherichia coli (AIIMS 2015) (c) Karry Mullis
6. An example of gene therapy is (d) J. Ingenhousz. (UP CPMT 2015)
(a) production of injectable hepatitis B vaccine 13. The first human hormone produced by recombinant DNA
(b) production of vaccines in food crops like potatoes technology is
which can be eaten (a) insulin (b) estrogen
(c) introduction of gene for adenosine deaminase in (c) thyroxin (d) progesterone. (AIPMT 2014)
persons suffering from severe combined immuno 14. Which of the following genes do not occur naturally in
deficiency (SCID) living organisms?
(d) production of test tube babies by artificial insemination (a) Bt genes (b) RNAi genes
and implantation of fertilised eggs. (AIIMS 2015) (c) cry genes
7. Select the wrong statement. (d) Endogenous cytoplasmic defense genes (AIIMS 2014)
(a) Human insulin is being commercially produced from a
15. RNA interference which is employed in making tobacco
transgenic species of Escherichia coli. plant resistant to Meloidogyne incognita is essentially
(b) The genetically modified Bacillus thuringiensis is used involved in
as biopesticide on the commercial scale. (a) preventing the process of replication of DNA
(c) Human protein, alpha-1-antitrypsin is used to treat (b) preventing the process of translation of mRNA
emphysema. (c) preventing the process of splicing of hnRNA
(d) The first transgenic cow, Rosie, produced alpha (d) preventing the process of transcription.
lactalbumin enriched milk. (Karnataka CET 2014)
(e) Bt toxin gene cry IAc controls the corn borer. Assertion & Reason
(Kerala PMT 2015)
The following questions consist of two statements each : assertion
8. The microbial biocontrol agent for butterfly caterpillar is
(A) and reason (R). To answer these questions, mark the correct
(a) Bacillus thuringiensis (b) Saccharomyces alternative as directed below :
(c) Lactobacillus (d) Cyanobacteria (a) If both A and R are true and R is the correct explanation
(e) Staphylococcus. (Kerala PMT 2015) of A.
9. A dicotyledonous plant forms crown gall when (b) If both A and R are true but R is not the correct explanation of
(a) Agrobacterium tumefaciens comes in contact with the A.
plant (c) If A is true but R is false.
(b) Agrobacterium rhizogenes comes in contact with the (d) If both A and R are false.
plant 1. Assertion : GM crops pose risk of causing toxicity and
(c) a specific part of DNA from the Ti plasmid gets allergic reaction in humans.
integrated with the plant chromosome Reason : The transgenes in commercial crops can endanger
(d) a specific part of DNA from the Ri plasmid gets native species.
integrated with the plant chromosome.
2. Assertion : Transgenic tobacco plant is resistant to
(WB JEE 2015)
nematode Meloidogyne incognita.
10. Genetic engineered male sterile crop plants may be Reason : Agrobacterium vectors deliver nematode specific
produced by inserting genes into the host tobacco plant.

70 MT BIOLOGY TODAY | AUGUST ‘17


3. Assertion : Transgenic animals are used to produce Short Answer Type Questions
useful biological products such as -1 antitrypsin to treat
emphysema. 1. (i) polygalacturonase (ii) somatic cell gene therapy
Reason : Transgenic animals are not safe for the production (iii) Genetic Engineering Approval Committee (GEAC)
and consumption of biological products for human use. 2. RNA interference is the phenomenon of inhibiting activity
4. Assertion : Human insulin cannot be synthesised using of a gene by synthesis of RNA molecules complementary to
bacterial cells. mRNA. The complementary strand of mRNA does not contain
sequences that can be translated to produce a functional
Reason : Bacterial genes possess introns.
protein hence called antisense RNA. The antisense RNA and
5. Assertion : The recombinant vaccines consist of critical mRNA molecules anneal to form duplex RNA molecules,
antigen expressed by the gene in bacteria or other microbes. which block translation of mRNA of affected gene.
Reason : The sub-unit vaccines are small part of the The specific genes of nematode are transferred to the host
polypeptide, constituting the entire vaccine, that are tobacco plant using Agrobacterium vectors. Introduction of
effective against immunisation. this DNA produces both sense and antisense RNA in host
cells. The two RNAs being complementary to each other
Short Answer Type Questions forms a dsRNA and thus gets silenced as the protein is not
1. Fill in the blanks. synthesised. In this way, the tobacco plant becomes resistant
to nematode infection.
(i) The shelf life of transgenic tomato is prolonged due to
non-availability of ________ enzyme. 3. The major bioethical concerns regarding the use and
(ii) Introduction of genes into somatic cells to correct the production of transgenic animals are :
defective gene is called _________. (i) Introduction of a transgene from one species into
another species violates the ‘integrity of species’.
(iii) All the decisions regarding the validity and safety of
(ii) Biotechnology may pose unforeseen risks to the
introducing genetically modified organisms are done
environment, including risk to biodiversity. Thus, it
by organisations as ________.
could disturb the existing ecological balance.
2. Write a short note on RNA interference which confers (iii) Transfer of human genes into animals (and vice-versa)
resistance against nematodes in tobacco plant. dilutes the concept of ‘humanness’.
3. Discuss some of the ethical issues concerned with the use (iv) Use of animals in biotechnology causes great suffering
and production of transgenic animals. to them.
4. Illustrate the steps in the production of human insulin using (v) Scientists cannot rule out the possibility of other
a prokaryotic cell with the help of a diagram. biological damage. It can accidentally create new
infectious agents.
ANSWER KEY (vi) When animals are used for production of
pharmaceutical proteins, they are virtually reduced to
New MCQ
the status of a ‘factory’.
1. (c) 2. (d) 3. (b) 4. (b) 5. (c) (vii) It is disrespectful to living beings and only exploits
6. (c) 7. (c) 8. (b) 9. (c) 10. (d) them for the benefit of human beings.
11. (d) 12. (c) 13. (c) 14. (d) 15. (a) 4. The given diagram illustrates the steps involved in the
production of human insulin using a prokaryotic cell :
Exam Section

1. (a) 2. (b) 3. (b) 4. (c) 5. (b)


6. (c) 7. (e) 8. (a) 9. (c) 10. (b)
11. (a) 12. (a) 13. (a) 14. (d) 15. (b)
Assertion & Reason

1. (b) 2. (a) 3. (c) 4. (d) 5. (b)



MT BIOLOGY TODAY | AUGUST ‘17 71


1. Which of the following cannot be inferred from Mendel’s 4. Select the correct statement regarding the phenomenon of
dihybrid cross? co-dominance.
(a) Two new combinations of traits, different from parental (a) In offsprings, the effect of one of the two alleles is
types appear more prominent.
(b) Plants with four different phenotypes were produced in (b) The effect in hybrid is a fine mixture of expression of
F2 generation two alleles.
(c) In F2 generation, the factors for the two characters (c) The alleles show quantitative effect.
assort independently (d) The expressed phenotype is a combination of two
(d) Single trait may be controlled by more than two alleles phenotypes and their alleles.
of a gene
5. Select the correct statements regarding sex determination.
2. Consider the following statements regarding the I. In grasshoppers, the females are homogametic (XX) while,
chromosomal theory of inheritance and select the correct males are heterogametic carrying X and Y chromosomes.
ones. II. In pigeon, the females are heterogametic (ZW) while the
I. Chromosomes as well as genes occur in pairs in diploid males are homogametic (ZZ).
cells. III. In butterflies, the females are homogametic (ZZ) while the
II. The two alleles of a gene pair are located on
males are heterogametic with odd sex chromosome (ZO).
homologous sites of heterologous chromosomes.
IV. In Drosophila, the females are homogametic (XX) and
III. Chromosomes are carriers of Mendelian factors which
males are heterogametic (XY).
segregate and assort independently during meiosis at
(a) I, II and III (b) II, III and IV
the time of gamete formation.
(c) II and IV (d) I, II, III and IV
IV. Pairing and separation of chromosomes during meiosis
leads to segregation of a pair of unit factors. 6. Read the following statements and select the incorrect ones
(a) I, II and IV (b) II and III regarding crossing over.
(c) I, III and IV (d) All of these I. In single crossing over between two non-sister
3. Which of the following represents the phenomenon of chromatids of homologous chromosome pair, two
incomplete linkage? parental types and two recombinant types are formed.
(a) Separation of two genes present on the same II. Crossing over occurs at zygotene stage of prophase I,
chromosome due to crossing over. at four strand stage.
(b) Inheritance of genes in a way that one gene suppresses III. Complementary crossing over forms two cross-over
the expression of other genes. and two non-cross over chromatids.
(c) Expression of heterozygous phenotype is intermediate IV. Genes located closely on chromosome have more
to phenotype of both the parents. chances of forming recombinants.
(d) Genes located on same chromosome always get (a) I, III and IV (b) I, II and III
inherited and expressed together. (c) II, III and IV (d) I and II

72 MT BIOLOGY TODAY | AUGUST ‘17


7. Though Mendel had published his results and conclusions (c) Large sampling size of Mendelian experiments gave
in 1865, yet his work was unnoticed for so many years till greater credibility to the data collected.
1900. The reason for this was mainly because (d) Multiple alleles can be found only after study of
(a) he used statistical methods and mathematical logic for populations.
analysing his experiments 14. If the factors responsible for pod shape and pod colour in a
(b) he failed to reproduce the same results with Hawkweed pea plant, studied by Mendel to understand inheritance of
(c) concept of stable, unblending, discrete units for various two genes were located closely on same chromosome, then
traits did not find acceptance which of the following would not have been explained?
(d) all of these. (a) Law of dominance
8. When two pure line short horned cattle for red (RR) and white (b) Law of segregation
(WW) were cross bred, the individuals of F1 generation were (c) Law of independent assortment
neither red nor white but were roan coloured. This change in (d) Both (b) and (c)
hair colour of cattle is due to the phenomenon of 15. If the mother is heterozygous for disease haemophilia and
(a) co-dominance father suffers from the disease, then what would be the
(b) pleiotropy probability of their son being haemophiliac?
(c) incomplete dominance (a) 100% (b) 50% (c) 25% (d) 75%
(d) dominance. 16. Identify the incorrectly described statements regarding sex
9. Suppose if the recombination frequency between genes linked inheritance.
present on same chromosome, ‘q’ and ‘r’ is 13%, ‘p’ and I. Sex linked traits are inherited together and determined
‘r’ is 2%, ‘q’ and ‘s’ is 4%, ‘q’ and ‘p’ is 15% and ‘r’ and by genes present on sex chromosomes.
‘s’ is 17%. Based on these frequencies, identify the correct II. Sex influenced traits are autosomal traits expressed in
sequence of genes on the chromosome. response to sex hormones in a particular sex only.
(a) p, q, r and s (b) p, r, q and s III. Sex limited traits are expressed as a result of by-products
(c) r, p, q and s (d) s, q, p and r of sex hormones and not due to expression of gene.
10. The genotype of a hybrid plant obtained from a cross can be IV. Haemophilia and colourblindness are sex linked
determined by crossing the plant with diseases in humans.
(a) pure dominant plant at F2 generation (a) II and IV (b) I and III
(b) pure recessive parent at F2 generation (c) II and III (d) III and IV
(c) heterozygous dominant parent at F1 generation 17. Which of the following is correct for the progenies showing
(d) pure recessive parent at F1 generation. incomplete dominance?
11. The presence of an extra X chromosome in genome of a (a) F1 is similar to the dominant parent.
human male indicates (b) There is no mixing of the effect of the two alleles.
(a) Turner’s syndrome (c) The genotypic ratio is similar to phenotypic ratio.
(b) Down syndrome (d) All of these
(c) Klinefelter’s syndrome 18. When pureline plant with violet flowers is crossed with
(d) metamales. pureline plant having white flowers, then all plants of
F1 generation bears violet flowers. What would be the
12. According to the chromosomal theory of linkage proposed proportion of plants with white flowers in generation
by Morgan and Castle the incorrect statement is produced by selfing of F1 plants?
(a) linked genes lie in a linear sequence in chromosome
(b) linked genes may occur on same chromosome 1 1 1 3
(a) (b) (c) (d)
(c) linked genes have a tendency to maintain parental 4 2 3 4
combination of genes 19. A condition in which two different alleles of a gene are
(d) the strength of linkage between genes is directly present in a single diploid cell of an individual is called
proportional to distance between them. (a) homozygous (b) heterozygous
(c) hemizygous (d) dihybrid.
13. Read the following statements and choose the incorrect one.
(a) Recessive allele is influenced in the presence of 20. Which of the following best explains the mechanism of
dominant allele. inheritance with respect to Mendel’s law of segregation
(b) In F2 generation, of a Mendelian monohybrid cross, the and independent assortment?
tall and dwarf traits were identical to their parental (a) A trait or character is inherited independent of the
types and shows blending inheritance. other character.

MT BIOLOGY TODAY | AUGUST ‘17 73


(b) The expression of one trait is modified in the presence 28. Consider the following statements regarding Punnett square
of other trait. and select the correct option.
(c) A trait is always inherited in combination with the other. I. Punnett square was developed by a British geneticist, R.C.
(d) All of these Punnett.
21. Select the correctly matched pair. II. It is a mathematical equation to calculate the probability
(a) Sickle cell anaemia - X chromosome of all possible genotypes of offspring in a genetic cross.
(b) Haemophilia - Y chromosome III. Punnett squares may act as standard tools used by
(c) Down syndrome - 21st chromosome genetic counsellors to determine the appearance of a
(d) Colourblindness - 7th chromosome genetic disease in the progenies.
22. Read the given statements and select the correct option. IV. Punnett square can be used to study only monohybrid
Statement A : Y-linked disorders are rare. crosses.
Statement B : Y chromosome do not contain many genes V. Punnett square can be effectively used to understand the
and these disorders are inherited from father to son. independent segregation of the two pairs of genes during
(a) Both statements A and B are true and B is the correct meiosis.
explanation of A. (a) I and IV (b) II and V
(b) Both statements A and B are true but B is not the (c) I, III and V (d) All of these
correct explanation of A. 29. Which cross helps to increase useful traits in a population?
(c) Statement A is true but statement B is false. (a) Back cross (b) Reciprocal cross
(d) Both statements A and B are false. (c) Test cross (d) All of these
23. If the child has blood group O, then which of the following
30. If a normal man marries a woman, carrier for colourblindness
would not be the blood groups of parents?
then what are the chances of their child being colourblind?
(a) A and B (b) A and A
(a) 25% (b) 50% (c) 75% (d) 33%
(c) AB and B (d) B and O
31. Match the column I with column II and III and select the
24. How many Barr bodies will be observed in an individual
correct option.
with genotype 44+XXX?
Column I Column II Column III
(a) 3 (b) 2
A. Albinism (p) Decreased (i) Autosomal,
(c) 1 (d) 4
synthesis of recessive
25. Read the following statements and select the correct ones. globin chain genetic
I. Linkage chromosome maps depict the exact physical disorder
distance amongst genes. B. Thalassemia (q) Absence of (ii) Urine turns
II. The sequence of genes in a chromosome can be homogenetisic black on
determined by chromosome maps. oxidase enzyme exposure to air
III. Frequency of linkage increases with the increase in the C. Sickle cell (r) Absence of (iii) Replacement
distance but decreases with increase in age. anaemia enzyme of glutamic
IV. Chromosome maps can predict the result of breeding tyrosinase acid with valine
experiments. D. Alkaptonuria (s) Homozygous (iv) Adequate
(a) I and III (b) III and IV recessive genes haemoglobin
(c) II and IV (d) I, II, III and IV not generated
26. The phenomenon, in which an allele of one gene suppresses in blood
the activity of an allele of another gene, is known as (a) A-(q)-(iii), B-(p)-(iv), C-(r)-(ii), D-(s)-(i)
(a) epistasis (b) dominance (b) A-(r)-(i), B-(p)-(iv) C-(s)-(iii), D-(q)-(ii)
(c) suppression (d) inactivation. (c) A-(p)-(i), B-(r)-(iv), C-(q)-(iii), D-(s)-(ii)
(d) A-(r)-(i), B-(q)-(iv), C-(s)-(ii), D-(p)-(iii)
27. Consider the following three cases with respect to sex
determination in Drosophila and select the correct option. 32. Which of the given statements best explains the Mendel’s
law of segregation?
P.
X =1 Q.
X >1 R
X = 0.5 (a) Only one allele of a pair expresses itself in a hybrid and
A A A prevents expression of the other allele.
P Q R (b) Alleles of a pair separate from each other during
(a) Female Metafemale Male gamete formation.
(b) Male Female Metafemale (c) Alleles of two pairs get randomly rearranged in
(c) Female Male Intersex offspring during fertilisation.
(d) Metafemale Female Male (d) Both (b) and (c).

74 MT BIOLOGY TODAY | AUGUST ‘17


33. Read the given statements and select the correct option. III. Absence of menstruation
Statement A : According to Bridges, in Drosophila, both IV. Cardiovascular abnormalities
Y-chromosome and autosomes are responsible for fertility These traits indicate that the female suffers from
and sex determination in males. (a) Down syndrome
Statement B : In Drosophila, autosomes are responsible (b) Turner’s syndrome
for feminine characteristics. (c) Klinefelter’s syndrome
(a) Both statements A and B are true and B is the correct (d) thalassemia.
explanation of A. 38. The hologynic type of inheritance refers to transfer of traits
(b) Both statements A and B are true but B is not the from
correct explanation of A. (a) father to son
(c) Statement A is true but statement B is false. (b) father to grandson through his daughter
(d) Both statements A and B are false. (c) mother to daughter
34. Select the incorrectly matched pair. (d) mother to granddaughter through her son.
(a) Monohybrid cross - 3:1 39. Select the correct statement regarding nondisjunction of
(b) Dihybrid cross - 9:3:3:1 chromosomes.
(c) Incomplete dominance - 1:2:1 (a) Nondisjunction refers to failure of synapsed
(d) Test cross - 1:3:1 homologous chromosomes to separate during
35. Consider the following statements regarding chromosomal metaphase I of meiosis.
aberrations and select the incorrect ones. (b) Nondisjunction was first discovered by Morgan in XX
I. Chromosomal aberrations refers to changes in number chromosomes in Drosophila.
and arrangement of genes in the chromosomes. (c) The initial nonseparation of synapsed chromosomes
II. Aneuploidy refers to the condition in an individual that occurs in meiocytes is called secondary
where the genome comprises of extra or fewer nondisjunction.
chromosomes than its normal number. (d) White eyed female when crossed with red eyed males
III. Heteroploidy occurs due to non-disjunction of produced a number of abnormal flies, i.e., XXY, XXX,
chromosomes of homologous pair. XYY due to secondary nondisjunction.
IV. Nullisomy is a type of hyperploidy in which one 40. From the given features, identify the type of cross being
chromosome is devoid of its homologue. performed.
(a) II and III (b) IV only I. The cross is both sided in which female of one type is
(c) II only (d) I, III and IV crossed with male of the second type and vice versa.
36. Barr body in humans refers to II. It allows to study inheritance of two or more traits.
(a) heterochromatin in both male and female cells III. It can differentiate between nuclear and cytoplasmic
(b) Y chromosomes in somatic cells of male inheritance as well as sex linked and autosomal
(c) inactive X chromosome in somatic cells of female inheritance.
(d) euchromatin in female cells only. (a) Test cross
37. A female shows certain characteristics as (b) Monohybrid cross
I. Short stature and narrow hips (c) Reciprocal cross
II. Undeveloped ovaries and breasts (d) Back cross

41. From the given pattern of gene inheritance for a particular IV. Females show traits only if their father and mother
trait, identify the type of gene associated with it. both possess it.
I. Trait appears more often in males as compared to (a) Autosomal recessive (b) Sex-linked recessive
females. (c) Autosomal dominant (d) Sex-linked dominant
II. Trait rarely appears in both father and son. 42. In maize, coloured endosperm (C) is dominant over
III. Females when homozygous for such trait, passes it to colourless (c) and full endosperm (S) is dominant over
all her sons. shrunken (s). When a dihybrid of F1 generation was test

MT BIOLOGY TODAY | AUGUST ‘17 75


crossed, it resulted in four phenotypes in given percentages. (c) tall and dwarf in 3:1 ratio
Coloured and full - 48% (d) all dwarf.
Coloured and shrunken - 5% 47. The linkage map of a chromosome of Drosophila has 45
Colourless and full - 7% linkage groups, with yellow body (y) gene at one end and
Colourless and shrunken - 48% bobbed hair (b) gene at its other end.
Based on the above data, the distance between the two The recombination frequency between these two genes ‘y’
non-allelic genes is deduced to be and ‘b’ will be
(a) 48 units (b) 11 units (a) 45% (b) 50% (c) 50% (d) 100%.
(c) 7 units (d) 5 units.
48. Consider the given pedigree analysis for a disease and identify
43. Consider the given pedigree analysis of the family members
the type of disease.
(comprising three generations) for the trait fused ear lobe.
This autosomal recessive trait appears in the progeny only
when its homozygous recessive (aa).
Aa aa
I
1 2

II (a) Sex linked recessive


X 3 4 5 Y 6 (b) Autosomal recessive
(c) Autosomal dominant
(d) Sex linked dominant
III 49. A geneticist infers that the two alleles for coat colour in
7 8 9 10 11
cattle show incomplete dominance. The best method to
Based on the above facts, identify the correct genotype of obtain evidence in support of his hypothesis is performing
individuals X and Y. (a) a test cross of homozygous recessive cattle to unknown
X Y genotype
(a) Aa aa (b) selfing of F1 cattle to obtain 1:2:1 ratio
(b) Aa AA (c) a cross with true breeding cattle of different colour to
(c) AA Aa obtain an intermediate phenotype in the F1
(d) aa Aa (d) a reciprocal cross where the sex of cattle of each coat
colour is reversed.
44. In a monohybrid cross between pea plants homozygous
for the gene of starch synthesis, i.e., BB and bb ‘B’ being 50. A colourblind woman, marries a normal man. What
dominant allele produces large and round starch grains proportion of his sons and daughters will be colourblind?
seeds while ‘b’ results in smaller and round starch grains Sons Daughters
seeds. The resulting heterozygotes produce intermediate (a) 50% 50%
type of seed grains. This is a case of (b) 100% –
(a) co-dominance (b) incomplete dominance (c) 75% 25%
(c) pleiotropism (d) multiple allelism. (d) 25% 25%
45. A child with abnormal characteristics was diagnosed with ANSWER KEY
a genetic disorder. On analysis of his genotype showed -
45+XY chromosomes. 1. (d) 2. (c) 3. (a) 4. (d) 5. (c)
The genetic disorder in concern can be identified as 6. (c) 7. (d) 8. (a) 9. (b) 10. (b)
(a) Turner’s syndrome (b) Down syndrome 11. (c) 12. (d) 13. (b) 14. (c) 15. (b)
(c) Klinefelter’s syndrome (d) Patau’s syndome. 16. (c) 17. (c) 18. (a) 19. (b) 20. (a)
21. (c) 22. (a) 23. (c) 24. (b) 25. (c)
46. A dwarf pea plant was treated with gibberellin and thus
26. (a) 27. (a) 28. (c) 29. (a) 30. (a)
became tall. This treated pea plant was then crossed with a
31. (b) 32. (b) 33. (d) 34. (d) 35. (b)
homozygous tall pea plant. The results in F2 generation are
36. (c) 37. (b) 38. (c) 39. (d) 40. (c)
expected to be
41. (b) 42. (b) 43. (b) 44. (b) 45. (b)
(a) all tall
46. (c) 47. (b) 48. (b) 49. (c) 50. (b)
(b) 50% tall and 50% dwarf


76 MT BIOLOGY TODAY | AUGUST ‘17


4 Class XII

T his specially designed column enables students to self analyse their


extent of understanding of specified chapters. Give yourself four
marks for correct answer and deduct one mark for wrong answer.
Self check table given at the end will help you to check your
readiness.
• Evolution
• Human Health and Disease

Total Marks : 160 Time Taken : 40 Min.


1. Select the incorrectly matched pair of organisms and their A B C
description. (a) Stomach Merozoites RBCs
(a) Neopalina – Connecting link between annelida and (b) Salivary glands Merozoites Liver
arthropoda (c) Salivary glands Sporozoites RBCs
(b) Archaeopteryx – Missing link between birds and (d) Stomach Sporozoites RBCs
reptiles 5. The theory which states that life reached earth from some
(c) Seymouria – Missing link between amphibians and unknown part of the universe in the form of spores is known
reptiles as ___(i)____. It was proposed by___(ii)____.
(d) Ornithorhynchus – Connecting link between reptiles (i) (ii)
and mammals (a) cosmozoic theory Georges Cuvier
2. Identify the option that correctly indicates the sequence in (b) biogenesis theory Preyer
which the organisms first evolved on earth. (c) theory of Panspermia Richter
I. Aerobic photoautotrophs (d) naturalistic theory Preyer
II. Anaerobic chemoheterotrophs 6. The type of speciation found in flightless grasshoppers,
III. Anaerobic photoautotrophs occurring at the edge of parent species range is called
IV. Anaerobic chemoautotrophs (a) sympatric speciation
(a) II → I → IV → III (b) I → III → IV → II (b) parapatric speciation
(c) IV → II → I → III (d) II → IV → III → I (c) quantum speciation
(d) allopatric speciation.
3. Read the following statements regarding humoral immunity
and select the incorrect one. 7. The type of immunity obtained by administration of anti-
(a) It does not provide immunity against cancer. tetanus serum to a recipient is called
(b) Antibodies that circulate in the blood and lymph form (a) artificial active immunity
(b) natural passive immunity
its main component.
(c) natural active immunity
(c) It defends the body against all kinds of pathogens.
(d) artificial passive immunity.
(d) It does not get activated following an organ transplant
in a patient. 8. Select the mismatched pair.
(a) Kala-azar – Glossina palpalis
4. Refer to the given table showing stages in life cycle of (b) Elephantiasis – Culex mosquito
Plasmodium and select the correct option for A, B and C. (c) Bubonic plague – Rat flea
Host Stage Site (d) Malaria – Anopheles mosquito
Anopheles mosquito Ookinete A 9. Which of the following statements about Stanley Miller and
Man B Liver Urey’s experiment are correct?
I. The mixture used for the experiment contained CH4,
Man Gametocytes C H2, NH3 and water vapour.

MT BIOLOGY TODAY | AUGUST ‘17 77


II. The apparatus was airtight and had a reducing 13. Which one of the following statements is incorrect?
environment. (a) The biogenetic law stating ‘Ontogeny repeats
III. Simple organic compounds such as alanine, aspartic phylogeny‘ was given by Von Baer.
acid and suberin were formed. (b) Auricular muscles in human beings are vestigial.
(a) I and II (b) II and III
(c) Tooth buds and gill clefts found in bird embryos are
(c) I and III (d) I, II and III
temporary embryonic structures.
10. Match column I with column II and select the correct option (d) Spiny and scaly anteaters show adaptive convergence.
from the codes given below.
Column I Column II 14. Given flow chart shows the chronological order of evolution
A. Age of fishes (i) Carboniferous Period of human from early to recent. Identify the missing links A,
B. Origin of mammals (ii) Devonian Period B and C from options given below
C. Age of herbs (iii) Jurassic Period
Dryopithecus Ramapithecus A
D. Age of amphibians (iv) Triassic Period
E. Origin of angiosperms (v) Quaternary Period
(a) A-(ii) B-(i) C-(iii) D-(iv) E-(v) Homo erectus
(b) A-(iii) B-(ii) C-(i) D-(iv) E-(v)
(c) A-(ii) B-(iv) C-(v) D-(i) E-(iii) Homo sapiens sapiens C B
(d) A-(i) B-(ii) C-(iv) D-(v) E-(iii) (a) A – Homo habilis, B – Australopithecus,
11. Refer to the given table showing differences between C – Cro-magnon man
allopatric speciation and sympatric speciation. (b) A – Australopithecus, B – Neanderthal man,
Allopatric speciation Sympatric speciation C – Homo habilis
(i) It occurs in an isolated It occurs from a segment (c) A – Homo habilis, B – Cro-magnon man,
population. within a population. C – Homo sapiens fossilis
(ii) The barrier is ecological The barrier is physical. (d) A – Australopithecus, B – Neanderthal man,
and genetic. C – Cro-magnon man
(iii) There are chances of Chances of breakdown
15. Mohit was suffering from high fever, pain in stomach and
breakdown of isolating of isolating mechanism
mechanism. are rare. eyes, rashes on the body, bleeding from nose, frequent
(iv) Speciation is rapid. Speciation is slow. vomiting and excessive thirst. A blood test suggested by his
doctor diagnosed him with
(v) Barriers to interspecific Barriers to interspecific
(a) chikungunya (b) amoebiasis
crossings are fewer. crossings are more
(c) dengue fever (d) viral hepatitis.
pronounced.
16. Consider the following statements about antibodies and
Which of the above cited differences are not correct?
select the correct option stating which ones are true (T) and
(a) (i), (iii) and (iv) (b) (ii), (iii) and (v)
which ones are false (F).
(c) (ii) and (iv) (d) (i) and (iv)
(i) All immunoglobulins are antibodies but all antibodies
12. Which of the following is a source for psychotropic drug?
are not immunoglobulins.
(a) (b)
(ii) IgA is abundant in colostrum and is passed on to the
newborn baby to impart active immunity.
(iii) IgG and IgM participate in complement activation.
(iv) Antigen-antibody complex is formed by binding of
specific antigen to Fab part in the variable region of the
antibody.
(c) (d)
(i) (ii) (iii) (iv)
(a) T F T F
(b) F F T T
(c) T T F F
(d) F F T F

78 MT BIOLOGY TODAY | AUGUST ‘17


17. Read the following postulates regarding a theory of evolution 23. Darwin’s theory of natural selection could not explain
and identify the scientist who gave them. (i) vestigial organs
I. Discontinuous variations are the raw material of (ii) discontinuous variations
evolution. (iii) differential reproduction
II. Accumulation of variations produce a new species. (iv) speciation
III. Evolution is a discontinuous process. (v) over specialisation of certain organs.
(a) Hugo de Vries (b) Alfred Wallace (a) (i), (ii) and (iii) (b) (ii), (iii) and (iv)
(c) Lamarck (d) Darwin (c) (i), (ii), (iii) and (v) (d) (i), (ii) and (v)
18. The mouthparts of some insects such as mosquitoes and 24. Given below are the different barriers that belong to non-
butterflies are example of ________ organs which results specific immunity. Select the option that correctly segregates
from _______ evolution. them under physical and physiological barriers respectively.
(a) analogous, divergent (i) Mucous membranes (ii) Sebum
(b) homologous, divergent (iii) Vaginal secretions (iv) Lysozyme
(c) homologous, convergent
(v) Cerumen (vi) Stratum corneum
(d) analogous, convergent
(vii) Cilia of the pharynx (viii) Bile
19. Select the correct statements for the drug having the Physical Physiological
following molecular structure. (a) (i), (ii), (iii), (vii) (iv), (v), (vi), (viii)
OH (b) (i), (vi), (vii) (ii), (iii), (iv), (v), (viii)
(c) (iii), (v), (vi) (i), (ii), (iv), (vii), (viii)
(d) (ii), (iii), (v), (vi) (i), (iv), (vii), (viii)
HO 25. Hardy-Weinberg principle is applicable to a population only
under conditions when there is
I. It is obtained from Cannabis.
(a) a gene flow in the population
II. It causes hallucinations and affects the cerebrum along
(b) elimination of certain alleles in a population
with sense organs.
(c) absence of mutations in genes or chromosomes of the
III. Even slight excess of this drug can cause death.
(a) I and III (b) I and II population
(c) I, II and III (d) II and III (d) variations in the gene frequencies over several
generations of the concerned population.
20. Select the correct statement regarding cell-mediated immune
system. 26. Identify the correct pair of carcinomas.
(a) The cell mediated immunity involves production of (a) Osteoma and adenocarcinoma
(b) Lipoma and melanoma
both B-cells and T-cells of immune system.
(c) Glioma and osteoma
(b) The effector functions of cell mediated immunity (d) Melanoma and adenocarcinoma
involves cytolysis and lymphokine production.
27. Select the option that correctly fills any four blanks in the
(c) Its regulatory functions include increased production of
given paragraph.
antibodies.
The primary lymphoid organs are bone marrow and
(d) All of these __(i)__. The __(ii)__ mature in the bone marrow and migrate
21. Select the mismatched pair. to secondary lymphoid organs via blood and __(iii)__. Here
(a) Alcoholism – Mallory Weiss Syndrome, Pancreatitis they undergo proliferation and __(iv)__. T-lymphocytes help
(b) Toxoid – Tetanus vaccine, Diphtheria vaccine B-lymphocytes to produce __(v)__ which regulate __(vi)__
(c) Homologous organs – Thorn of Bougainvillea, Tendril immunity.
of Cucurbita. (a) (i) thymus, (iii) lymph, (iv) differentiation, (vi) humoral
(d) Connecting link – Seymouria, Ichthyostega (b) (i) spleen, (ii) B-lymphocytes, (iv) activation,
(v) immunoglobulins
22. According to Oparin’s hypothesis, the earliest non-living (c) (i) thymus, (iii) interstitial fluid, (iv) perforins,
structures that led to the formation of living cells were called (vi) humoral
(a) microspheres (b) proteinoids (d) (i) lymph nodes, (ii) T-lymphocytes, (v) antibodies,
(c) coacervates (d) stromatolites. (vi) cellular

MT BIOLOGY TODAY | AUGUST ‘17 79


28. Select the incorrectly described species. (a) retrogressive metamorphosis
(a) Monotypic – A species without any distinct variety. (b) atavism
species (c) progressive metamorphosis
(b) Sibling – Two species that are morphologically (d) divergent evolution.
species almost identical and can interbreed
freely. 33. Of the below mentioned functions, which of these are
(c) Allopatric – A species occurring in an exclusive performed by complement proteins?
species geographical area. (a) Cytolysis (b) Inflammation
(d) Allochronic – Species that do not belong to the (c) Phagocytosis (d) All of these
species same time periods. 34. Consider the following statements and select the correct
29. Read the given statements and select the correct option. option stating which ones are true (T) and which ones are
Statement A : Chronic alcoholism can lead to cirrhosis. false (F).
Statement B : Continuous intake of alcohol leads to (i) Marsupials of Australia follow the same biogeographical
hepatocyte destruction and collagen deposition in liver. evolutionary trend as Darwin’s finches.
(a) Both statements A and B are correct and B is the (ii) The considerable change in allele frequency that is
correct explanation of A. more marked in small populations is called genetic
(b) Both statements A and B are correct but B is not the drift.
correct explanation of A. (iii) Major histocompatibility complex responsible for
(c) Statement A is correct but B is incorrect. lymphocyte recognition are encoded by genes present
(d) Both statements A and B are incorrect. on chromosome 8.
(iv) Benign tumours are cancerous and show metastasis.
30. Which one of the following correctly describes the
(i) (ii) (iii) (iv)
transmission and symptoms of the respective diseases?
(a) T F F T
Disease Transmission Symptoms
(a) Influenza Droplet Sore throat, sneezing, (b) T T T F
infection headache (c) T T F F
(b) Yellow fever Rat flea Severe pain in joints, (d) F F T F
yellowing of skin, 35. Read the following statements regarding autoimmunity and
chilliness select the correct ones.
(c) Cholera Sputum of Frequent white stool I. Autoimmunity is the property of immune system
patient and vomiting wherein it fails to recognise ‘self’ and ‘non-self’ cells
(d) Botulism Wound Double vision, leading to malfunction of its system.
infection vomiting, blood and II. During autoimmune disorder, the C-reactive proteins,
an important component of immune system decreases
mucus in stool
significantly.
31. Read the given statements and select the incorrect one. III. Autoimmunity may also get triggered by decreased
(a) Balancing selection favours average sized individuals helper T-cell and increased suppressor T-cell functions.
while eliminates small sized individuals. IV. Autoimmune disorders are more common in women
(b) During progressive selection, the population changes than in men.
towards one particular direction. (a) I and IV (b) II and IV
(c) Diversifying selection is opposite of stabilising selection (c) I and III (d) II and III
and is rare in nature. 36. Genetic drift can be best explained as
(d) Progressive selection maintains a mean value from (a) sum total of all genes and alleles in a population
generation to generation. (b) addition of new alleles to local gene pool of host
32. A normal Citrus leaf shows jointed winged petiole where population
the lamina is separated from wing petiole by means of a (c) drastic change in allele frequency when population
constriction. But at times, the leaf may show two additional size is very small
lateral leaflets, thus making the leaf trifoliate. This describes (d) change in gene frequency due to repeated mating for
the phenomenon of certain selected traits.
Contd. on Page No. 84
80 MT BIOLOGY TODAY | AUGUST ‘17
BIO -GRAM
Gel Filtration Chromatography
Chromatography is a biophysical technique that is used for the separation, identification, and purification of
the components in a mixture for qualitative and quantitative analysis. Biomolecules can be purified based
on their size and shape, total charge, hydrophobic groups present on the surface, and binding capacity with
the stationary phase. There are two basic techniques of chromatography based on type of stationary bed:
plane chromatography and column chromatography. Column chromatography is one of the most common
methods of protein purification. It may be of various types viz., adsorption, partition, ion exchange, affinity,
gel filtration. Gel filtration or molecular/size exclusion or molecular sieve chromatography is a type of column
chromatography that separates molecules on the basis of their molecular size.

The basic components of column chromatography are stationary phase; composed of either a solid phase or a layer of a liquid adsorbed on the
surface of a solid support. It is chosen according to the analytes to be separated and mobile phase; composed of liquid components that complement
the stationary phase and maintains constant rate of flow into the column. The general setup of gel filtration chromatography is described below.

Injector Inlet
Delivers test Provides entry to eluting solvent Data recorder
sample to the top and sample into the column. Chromatograph and analyser 75 Kd
of the column.
Detector 125 Kd

Amount of analyte
Syringe Measures visible or UV absorbance. 250 Kd
to load Quantification is achieved on the
sample basis of peak area coupled with an
Pump appropriate calibration graph. The
Column area of each peak is proportional to
Filling the chromato- the amount of analyte. Use of mass or Volume eluted
graphy column with NMR spectrometer as detector provides
matrix is referred to as structural identification of molecules.
“column packing”. The
packed matrix is called Determination of molecular weight
the “bed”. Once the effluent volume is determined for the given sample i.e., protein,
its distribution coefficient Kd is calculated. Kd refers to the distribution of
Outlet solute particle (protein) between inner and outer solvent.
Solvent/Elution
buffer The effluent from the Kd = 0, when solute molecule is large, Kd = 1 if solute is small and
(Mobile phase) column is led into two penetrate gel pores and Kd is between 0 and 1 for intermediate sized
streams i.e., continuous proteins.
monitor and collecting Kd of standard proteins of known molecular weights are plotted against
system. the log of their molecular weight. By placing the Kd of unknown protein
Fraction collector on this plot, molecular weight of unknown protein can be determined.
Collects the separated analytes. They are
programmed for collection of a definite
Sample Elution buffer
volume of the effluent in each tube
Small volume of sample having different size of It is the mobile phase that flows
before a new tube is placed in position
analytes may be injected directly into the column through the matrix and out of the
automatically.
surface with the help of a syringe or capillary tube. column.

Solvent/ Solvent
Principle
When a sample having molecules of different Elution
buffer
size is placed on the top of such an equilibrated Small molecules enter
column, the larger molecules pass through the Frit the aqueous spaces
interstitial spaces between the beads. This is It is a membrane or porous within beads
because the pores of the gel have a smaller disk that supports and
retains the matrix in the Polymer bead
diameter than what is needed for the large column but allows water
molecules to enter. Large molecules, therefore, and dissolved solutes to
move down the column with little resistance pass. Sample
and will come first in the effluent. The small
molecules, however, can enter the pores and
are thereby effectively removed from the stream Column packing Large molecules
of the eluting solvent. These molecules are thus Packing of inert heteroporous cannot enter
retarded and will then pass through the column cross linked polymeric gels or beads
beads (stationary phase)which Elution spout
at slower rate, hence appear latter in effluent. Elution
Thus, a sample passing through a gel filtration are allowed to attain equilibrium
with a solvent (mobile phase). Flow direction
column will separate based on molecular size:
the big ones will elute first , middle sized proteins It is usually carried out under a
will elute in the middle and the smallest ones Solvent Separated protein constant hydrostatic pressure
will elute last. Fig.: Principle of gel filtration chromatography. to achieve a constant flow rate.

MT BIOLOGY TODAY | AUGUST ‘17 81


BIOLOGY
OLYMPIAD PROBLEMS
1. A few statements regarding the sexual and asexual modes of 4. A typical graph obtained for an enzyme catalysed reaction
reproduction in plants are given. that takes place in a human body is shown below.
(i) In sexual reproduction, progeny are genetically Vmax
different from each other.

Velocity
(ii) In asexual mode of reproduction, progeny are genetically
identical to each other but different from the parent.
(iii) Sexual reproduction is more conducive for driving
evolution. [Substrate]
(iv) A minor change in the habitat may adversely affect all Which of the following correctly represents the same
offspring derived by asexual reproduction. reaction in which no enzyme is used? Broken line represents
(v) A bisexual plant grown in isolation is always incapable the enzyme catalysed reaction for comparison.
of sexual reproduction.
Vmax Vmax
Which of these statements are correct?
(a) (i), (iii) and (iv) (b) (i), (ii) and (v)
Velocity

Velocity
(c) (iii), (iv) and (v) (d) (ii), (iii) and (iv) (INBO 2017) (a) (b)
2. Two ecological pyramids are drawn below. If ‘I‘ represents
aquatic and ‘II‘ represents terrestrial ecosystem, which of [Substrate] [Substrate]
the following statements are true?
Vmax Vmax
Velocity
Velocity

I II
(c) (d)
(i) Pyramid ‘I‘ is based on biomass.
(ii) Pyramid ‘II‘ is based on numbers. [Substrate] [Substrate]
(iii) The unshaded box in pyramid ‘I‘ represents zooplankton (INBO 2015)
and in pyramid ‘II‘ it represents insects.
5. The amount of DNA present per cell during a nuclear division
(iv) Neither ‘I‘ nor ‘II‘ can be based on energy.
is represented as a bar diagram below.
(a) (ii) and (iii) only (b) (i) and (iii) only
DNA content →

(c) (i), (ii) and (iv) only (d) (i), (ii), (iii) and (iv) 4C
(INBO 2016)
3. The support staff in a Zoology lab mixed up the description 2C
charts of the following organisms. C
X : Rotifer Y : Sea anemone Z : Spider W X Y Z
Description charts: What phase are represented by X and Y?
I. Coelomate with segmented body. (a) X - Prophase I, Y - S phase
II. Pseudocoelomate with alimentary canal and head (b) X - Prophase I, Y - Prophase II
having ciliated crown. (c) X - Metaphase II, Y - Prophase II
III. Diploblastic with gastrovascular cavity. (d) X - Anaphase II, Y - Telophase I (NSEB 2014-15)
Description charts that match X, Y and Z respectively are: 6. When treated with proteases, the extract of which of the
(a) I, II and III (b) II, III and I endocrine glands would lose its hormonal influence?
(c) III, I and II (d) I, III and II (a) Ovary (b) Pineal gland
(NSEB 2015-16) (c) Pituitary gland (d) Adrenal cortex (INBO 2014)

82 MT BIOLOGY TODAY | AUGUST ‘17


7. Water potential ( ) plays important role in water absorption The map distance between ‘s’ and ‘e’ genes will be
and conduction from soil to leaf. Under which condition the (a) 87.7 map units (b) 50.2 map units
process will go on smoothly? (c) 12.3 map units (d) 49.8 map units.
(a) atmosphere < leaf < root < soil (INBO 2013)
(b) atmosphere > leaf > root > soil SOLUTIONS
(c) atmosphere = leaf = root = soil
1. (a) : Progenies obtained through asexual reproduction are
(d) atmosphere < leaf = root > soil (NSEB 2013-14)
genetically identical to the parents as well as to each other.
8. A short fragment of DNA was analysed for various mutations. A bisexual plant contains flowers with both male and female
When comparison between normal and mutated types was reproductive organs. Such plants when grown in isolation
done, following gel pattern was observed for two individuals A may undergo self pollination if they have proper devices or
and B. Most probable type of mutation in these individuals is contrivances and hence undergo sexual reproduction.
Control A B
2. (d) : Pyramid I is spindle-shaped pyramid of biomass in
aquatic ecosystem and it can be represented as follows:
Large fishes
Small fishes
Zooplanktons

Phytoplanktons
Here, the biomass of phytoplankton is smaller than that of
zooplankton because of lower reproductive potential and
(a) deletion in A and inversion in B.
longevity.
(b) deletion in A and duplication in B.
(c) inversion in A and deletion in B. Pyramid II represents spindle like pyramid of number in a
(d) duplication in A and deletion in B. (NSEB 2013 - 14) tree ecosystem as given below :

9. Effect of some compounds (present in partially digested Falcons


food) on pancreatic secretion is depicted in the bar graph. Birds
Compounds 1, 2 and 3 represent Insects
Tree

Here a single large producer tree provides nourishment


Levels of enzymes
to several insects. These insects may be the food of
comparatively small population of birds those are eaten by
Levels of NaHCO3
very small number of eagles or falcons.
The pyramid of energy is always upright, therefore, pyramid
I and II cannot be based on energy.
Control 1 2 3
3. (b)
(a) 1. Acid 2. Fat 3. Salt
4. (c) : In absence of enzyme, reaction is not catalysed and
(b) 1. Salt 2. Peptone 3. Fat
hence rate of reaction or velocity increases slowly in
(c) 1. Acid 2. Fat 3. Peptone
comparison to catalysed reaction with the increase in
(d) 1. Pepsin 2. Acid 3. Fat (INBO 2013)
substrate concentration.
10. A cross between wild-type fruitfly Drosophila melanogaster 5. (b) : The amount of DNA in each diploid cell is 2C. During
[normal bristles and grey body (s+s, e+e)] with a mutant fly S phase, DNA replicates and the amount of DNA per cell
[short bristles and ebony body (ss, ee)] produces following doubles i.e., increases to 4C. Hence, when cell enters M
progenies: phase and undergoes meiosis I, the amount of DNA in
Normal bristles with grey body – 537 flies prophase I will be 4C. After completion of meiosis I DNA
Short bristles with ebony body – 542 flies will reduce to 2C because meiosis I is a reductional division.
Normal bristles with ebony body – 76 flies Therefore, in prophase II of meiosis II the amount of DNA
Short bristles with grey body – 75 flies would be 2C.

MT BIOLOGY TODAY | AUGUST ‘17 83


6. (c) : Pituitary hormones are protein hormones. When individual B and normal DNA fragment which suggests that
they are treated with proteases or proteolytic enzymes, A is larger than normal fragment due to duplication.
hydrolysis of their peptide bonds occurs that causes loss of 9. (c) : Pancreatic secretion contains different enzymes for
their hormonal influence. the digestion of starch, proteins, peptones, fats and sodium
7. (a) : Water potential ( w) helps in the movement of water bicarbonate to neutralise hydrochloric acid of the chyme
within systems. Water flows from higher water potential (food coming from stomach). Bar graph 1 shows high level
to lower water potential. Therefore, to maintain smooth of NaHCO3 which is the effect of high acid concentration.
absorption and conduction of water the condition should Bar graph 2 and 3 show high levels of enzymes due to high
concentration of fat and peptone.
be atmosphere < leaf < root < soil.
10. (c) : Progenies having normal bristles with ebony body and
8. (d) : Gel electrophoresis separates fragments of DNA, RNA
short bristles with grey body are recombinants. Hence, the
or protein on the basis of their size. The smaller fragment
recombination frequency can be calculated as -
moves farther than larger fragment. In the given gel pattern
DNA fragment of individual B travelled greater distance Recombinant frequency = Recombinants × 100%
Total offspring
than the DNA fragment of individual A and normal DNA
= 76 + 75 / 1230 = 0.123% or 12.3%
fragment. It means DNA fragment of B is shorter than
Since 1 map unit = 1%
other two which suggests deletion in DNA. Similarly,
DNA fragment of A travelled lesser distance than that of So, the map distance between s and e gene = 12.3 map unit.

Contd. from Page No. 80
events that occur by chance in nature and random
4 Class XII
mutations in organism
(b) interspecific struggle for existence gives rise to survival
37. Match column I with column II and select the correct option. of the fittest
(There can be more than one match for items in column I). (c) strong selection pressure in a population having
Column I Column II different forms results in one form, permanently
A. Natural selection (i) Rubella replacing the other
B. Autoimmune disease (ii) Whooping cough (d) stabilising selection does not favour evolution.
C. Viral disease (iii) Glucose 6-phosphate
39. Given below are the various events that occur in a person,
dehydrogenase deficiency
during AIDS, from the time of first exposure to virus. Choose
D. Bacterial disease (iv) Multiple sclerosis
the option with the correct sequence.
E. Spirochaetal disease (v) Rhinitis
I. Viral DNA gets incorporated in the genome of host cell.
(vi) Sickle cell anaemia
II. Reverse transcriptase enzyme produces viral DNA in
(vii) Leprosy
host cell.
(viii) Pernicious anaemia
III. Viral RNA is produced by infected cell.
(ix) Syphilis
IV. Virus attacks helper T-lymphocytes.
(x) Gonorrhoea
(a) IV → II → III → I (b) II → I → III → IV
(a) A-(v), (vi); B-(iii), (iv); C-(viii), (ix); D-(vii), (x); E-(i), (ii)
(c) II → III → I → IV (d) IV → I → II → III
(b) A-(iv), (viii); B-(v), (vi); C-(i), (ii); D-(vii), (ix); E-(iii), (x)
(c) A-(iv), (vi); B-(iii), (viii); C-(i), (x); D-(v), (vii); E-(ii), (ix) 40. Which of the following is not true regarding amphetamines?
(d) A-(iii), (vi); B-(iv), (viii); C-(i), (v); D-(ii), (vii), (x); E-(ix) (a) It is used in the dope test for athletes.
(b) It is a CNS stimulator.
38. Lederberg’s replica plating experiment provided an evidence
(c) It is obtained from Lophophora williamsii.
in support of hypothesis that proves
(d) It does not remove fatigue and cause wakefulness.
(a) evolution is a random probability process based on
Key is published in this issue. Search now! 

Check your score! If your score is


> 90% EXCELLENT WORK ! You are well prepared to take the challenge of final exam.

No. of questions attempted …… 90-75% GOOD WORK ! You can score good in the final exam.
No. of questions correct …… 74-60% SATISFACTORY ! You need to score more next time.
Marks scored in percentage …… < 60% NOT SATISFACTORY! Revise thoroughly and strengthen your concepts.

84 MT BIOLOGY TODAY | AUGUST ‘17


Readers can send their responses at editor@mtg.in or post us with complete address by 25th of every month to win exciting prizes.
Winners' names will be published in next issue.

ACROSS
1. Inorganic inclusions found in some bacteria that 1 2
help them align along the geomagnetic lines. (12)
3 4 5 6 7 8 9
3. Sternum of a bird, to which the powerful flight
muscles are attached. (4) 10 11 12
6. Sepals that are modified hairy processes in
13 14
some plants. (6)
10. The female reproductive part that dilates under 15
the influence of relaxin during child birth. (6) 16 17
13. The process of fermentation brought about by
yeast extract. (7) 18

15. The complete sequence of developmental 19 20


stages of biotic succession. (4)
17. A triangular region of urinary bladder present
between the openings of ureters and urethra. (7) 21

18. Study of different appearances of a plant with 22 23 24 25


the changing seasons. (9)
26
19. The point of attachment between the
homologous chromosomes where crossing over 27
occurs. (7)
28
23. Modified maxillary teeth of snakes. (5)
24. An example of club fungi that produces thick 29 30
walled black coloured spores. (4)
26. A protein and carbohydrate deficiency disease 8. The animals that are active during dusk. (8)
resulting in impaired growth of infants. (8) 9. The sharp and stiff multicellular epidermal outgrowths found in
27. Acrosomal enzyme that digests the zona pellucida of egg. (7) some plants. (8)
28. The space between the two pleural sacs. (11) 11. The pentapeptides produced by nerve cells that influence
perception of pleasure and pain. (11)
29. Hole forming proteins secreted by cytotoxic T-cells to destroy
12. Enamel forming cells that disappear before tooth eruption. (11)
foreign cells. (9)
14. The process of removal of introns from hnRNA. (8)
30. Black powdery substance produced by incomplete combustion of 16. An enzyme used to cut dsRNA molecule into small interfering RNAs.
fuel. (4) (5)
DOWN 20. A single long process of cyton that carries nerve impulse away
2. Plants that require low intensity of light for growth. (10) from neuron towards the brain. (4)
4. An extract produced from Claviceps purpurea from which LSD is 21. Little sacs of synovial fluid that act as cushions in some joints. (6)
obtained. (5) 22. The unbranched underground stems of Colocasia used for vegetative
5. An insoluble, fibrous protein that is responsible for blood clotting. (6) propagation. (5)
7. The special starch storing structures present in the chloroplasts of 25. A popular Japanese food made from fermented soybean milk. (4)
Spirogyra. (9) 

MT BIOLOGY TODAY | AUGUST ‘17 85


86 MT BIOLOGY TODAY | AUGUST ‘17

Você também pode gostar